由买买提看人间百态

boards

本页内容为未名空间相应帖子的节选和存档,一周内的贴子最多显示50字,超过一周显示500字 访问原贴
Parenting版 - 分数应用题
相关主题
求科普: 奥数 vs. Math Olympiad vs. Math Count vs. Math circle vs. 超前学数学学校早晚要教的东西,早早学会了又如何?
9岁, 如何报名考amc10AMC 8 成绩 2016
算不算有数学天分请教小孩数学教育问题
[葩论] 论“有限小灶策略”小学数学书推荐?
给初中学生家长的一些建议 (转载)我来说一下天才的鉴定方法
小学数学上超前班有什么好处吗?[华盛顿邮报] 9 - 14 岁是数学兴趣的关键时期?
小学数学有比赛项目吗怎么推小学数学好?小学一年级的孩子。
请教:怎样说服孩儿他爸多关心孩子的学习?请问,AMC 8应该从几年级开始练习呀?
相关话题的讨论汇总
话题: 数学话题: 竞赛话题: moems话题: aime话题: amc
进入Parenting版参与讨论
1 (共1页)
d**********h
发帖数: 2795
1
几十年前,四年级之前的数学难点是四则混合运算,主要讲究细心
五年级数学的难点是分数应用题
美国这里几年级开始这个?为啥没人讨论?
还是美国不教这个思路,直接进入代数方程思路?
我觉得分数应用题的难点在:有一个未知的东西(也就是要求的那个),一切题面描述都
围绕这个未知展开,但是因为不可以列方程,所以必须倒着想。如果描述太复杂,脑子
就不够用了,所以有时候需要作图。
例如,一堆糖,小妹吃了三分之一,还剩六个,那么问:吃之前一共有多少糖?
要是代数方程,直接写
一堆 减去 一堆 乘以 三分之一 等于 六
分数应用题的思路不允许事先把“一堆”虚拟出来,要在脑子里把逻辑解套,直接写成
“显式”表达
六 除以 (一 减去 三分之一)
这个解套过程在几个进水管,放水管同时工作时就困难了
专家们展开说说。。。
如果说分数应用题和方程是两种思路,两种方法,我觉得在方程之前必须把分数思路巩
固好。一旦接触了简单直观的方程思维,分数思维就废了
s***n
发帖数: 1280
2
你说的例子算two-step equation。学校里面应该是七年级开始讲,直接上代数方法。
简单点的分数应用题,六年级教。
非代数的方法要求概念更清楚,实际上比代数方法难,美国这边学校里面不教,但低年
级的GT可能教。

【在 d**********h 的大作中提到】
: 几十年前,四年级之前的数学难点是四则混合运算,主要讲究细心
: 五年级数学的难点是分数应用题
: 美国这里几年级开始这个?为啥没人讨论?
: 还是美国不教这个思路,直接进入代数方程思路?
: 我觉得分数应用题的难点在:有一个未知的东西(也就是要求的那个),一切题面描述都
: 围绕这个未知展开,但是因为不可以列方程,所以必须倒着想。如果描述太复杂,脑子
: 就不够用了,所以有时候需要作图。
: 例如,一堆糖,小妹吃了三分之一,还剩六个,那么问:吃之前一共有多少糖?
: 要是代数方程,直接写
: 一堆 减去 一堆 乘以 三分之一 等于 六

d*******e
发帖数: 863
3
Bar Model
t*******r
发帖数: 22634
4
我不熟悉五年级学校数学的难点。
但就这道题而言,我觉得是典型的 arithmetic 阶段的 whole-part-part 的模式题。
(或者 whole-part-rest?我不熟悉学校具体用词)
个人觉得 pre-algebra 的 2-stage read-forward work-backward problem,是指非严
格固定模式的 (指可以有模式但不是上面那种严格固定模式)。严格固定模式应该不算。
(另外我对小学数学术语不熟悉,用词就是了意思)。
个人看法,我不常检查娃的作业,我说的不一定对。

【在 d**********h 的大作中提到】
: 几十年前,四年级之前的数学难点是四则混合运算,主要讲究细心
: 五年级数学的难点是分数应用题
: 美国这里几年级开始这个?为啥没人讨论?
: 还是美国不教这个思路,直接进入代数方程思路?
: 我觉得分数应用题的难点在:有一个未知的东西(也就是要求的那个),一切题面描述都
: 围绕这个未知展开,但是因为不可以列方程,所以必须倒着想。如果描述太复杂,脑子
: 就不够用了,所以有时候需要作图。
: 例如,一堆糖,小妹吃了三分之一,还剩六个,那么问:吃之前一共有多少糖?
: 要是代数方程,直接写
: 一堆 减去 一堆 乘以 三分之一 等于 六

w******o
发帖数: 726
5
这些关键的数学知识美国小学基本不教,老师也糊涂。很可气。V

【在 s***n 的大作中提到】
: 你说的例子算two-step equation。学校里面应该是七年级开始讲,直接上代数方法。
: 简单点的分数应用题,六年级教。
: 非代数的方法要求概念更清楚,实际上比代数方法难,美国这边学校里面不教,但低年
: 级的GT可能教。

w******o
发帖数: 726
6
你说的没错,大多数美国孩子的数学就是被太早引入代数费掉的。没有了数学感觉。

【在 d**********h 的大作中提到】
: 几十年前,四年级之前的数学难点是四则混合运算,主要讲究细心
: 五年级数学的难点是分数应用题
: 美国这里几年级开始这个?为啥没人讨论?
: 还是美国不教这个思路,直接进入代数方程思路?
: 我觉得分数应用题的难点在:有一个未知的东西(也就是要求的那个),一切题面描述都
: 围绕这个未知展开,但是因为不可以列方程,所以必须倒着想。如果描述太复杂,脑子
: 就不够用了,所以有时候需要作图。
: 例如,一堆糖,小妹吃了三分之一,还剩六个,那么问:吃之前一共有多少糖?
: 要是代数方程,直接写
: 一堆 减去 一堆 乘以 三分之一 等于 六

d**********h
发帖数: 2795
7
看来我要自己给他补补如何“绕着玩儿的想”
美国不教,可能有他们的考虑,只是我们不知道。
欧美的教育理念和前苏联经常不一致,而我们师承苏俄,重理论轻应用

【在 w******o 的大作中提到】
: 这些关键的数学知识美国小学基本不教,老师也糊涂。很可气。V
b***d
发帖数: 2695
8
可是数学,没有理论基础,就不好应用之

【在 d**********h 的大作中提到】
: 看来我要自己给他补补如何“绕着玩儿的想”
: 美国不教,可能有他们的考虑,只是我们不知道。
: 欧美的教育理念和前苏联经常不一致,而我们师承苏俄,重理论轻应用

t*******r
发帖数: 22634
9
纠结 pre-algebra multiple step word problem 的话,直接上 AMC8,1985 到 2015
总共 30 年,30*25 = 750 道题。不用再纠结了。

【在 b***d 的大作中提到】
: 可是数学,没有理论基础,就不好应用之
d**********h
发帖数: 2795
10
上网做了两题amc8和10
太容易了,to我
下周开始试试折磨我娃 :)

2015

【在 t*******r 的大作中提到】
: 纠结 pre-algebra multiple step word problem 的话,直接上 AMC8,1985 到 2015
: 总共 30 年,30*25 = 750 道题。不用再纠结了。

相关主题
小学数学上超前班有什么好处吗?学校早晚要教的东西,早早学会了又如何?
小学数学有比赛项目吗AMC 8 成绩 2016
请教:怎样说服孩儿他爸多关心孩子的学习?请教小孩数学教育问题
进入Parenting版参与讨论
t*******r
发帖数: 22634
11
折磨你应该用 AIME,从最后一道往前做。
// run

【在 d**********h 的大作中提到】
: 上网做了两题amc8和10
: 太容易了,to我
: 下周开始试试折磨我娃 :)
:
: 2015

t*******r
发帖数: 22634
12
如果你的目的是 pre-algebra 并且还想避免代数的话,我个人建议不要做 AMC10,因
为 AMC10 里面应该很多代数题。
不过你自己做一套 AMC10 就知道了。

【在 d**********h 的大作中提到】
: 上网做了两题amc8和10
: 太容易了,to我
: 下周开始试试折磨我娃 :)
:
: 2015

Y********d
发帖数: 1478
13
潮水的毛病果然有传染性:)

【在 d**********h 的大作中提到】
: 上网做了两题amc8和10
: 太容易了,to我
: 下周开始试试折磨我娃 :)
:
: 2015

d**********h
发帖数: 2795
14
是这样的。
其实我现在的目的就是让孩子有个感觉,从读懂问题开始的感觉,我娃脑子不转玩儿(
我目前感觉)
半年为期,
如果读题有了感觉,再说解题思路
目前没有任何追求解题的想法

【在 t*******r 的大作中提到】
: 如果你的目的是 pre-algebra 并且还想避免代数的话,我个人建议不要做 AMC10,因
: 为 AMC10 里面应该很多代数题。
: 不过你自己做一套 AMC10 就知道了。

d*****n
发帖数: 1956
15
还有方程,现在国内5年级就教了,但是解方程的方法是等式两边加上或减去同一个数
,等式不变。我们那时是初一引入正负数的概念后用移项来解方程的。
这个分数的问题我也注意到了,这边都是直来直去,国内的都要拐好几个弯儿。比如我
教孩子分数乘法要先约分再相乘,他非要先相乘再约分,为此和我不止一次闹矛盾。
他们教的求最大公约和最小公倍的方法也和我们当年不一样。
有心按国内的课本教,但是每天回来还要做饭收拾之类的,有时候力不从心。
l*****8
发帖数: 16949
16
相对AMC, MOEMS难多了。AMC主要是覆盖的内容多。其实就如潮水给起的名字一样,基
本都是计算题。
娃的学校12月要让他们去参加一个数学竞赛。我看了一下历年的样题,基本都是计算题
(比AMC更加"计算”),所谓难的地方就是用文字绕人。LP和娃都抱怨我给她做的MOEMS
太难了,不符合考情。我正在纠结是否继续我的MOEMS大计还是屈从于考试,让她多做
计算题。
t*******r
发帖数: 22634
17
我觉得 MOEMS 的问题不仅仅是难,而且跟学校数学不直接 align。而且需要我更多的
engagement。
另一方面,MOEMS 的好处是用 list & check 就可以解题,不需要多少先验的数学知识
,娃可以自己探索。
因为我家的主力方向不是参加数学竞赛,而是帮助学校数学 (参见我旁边的“有限小灶
”)。另外我的想法并不是避免代数,而是快速通过 pre-algebra 而进入代数。所以我
用 MOEMS 搞了一年 list & check 以及把 math foundation 介绍完以后,我就改成
AMC8,目标就是快速进入 AMC10 阶段。
我个人觉得 MOEMS 更适合能将来参加数学竞赛拿奖的。实话实说。

MOEMS

【在 l*****8 的大作中提到】
: 相对AMC, MOEMS难多了。AMC主要是覆盖的内容多。其实就如潮水给起的名字一样,基
: 本都是计算题。
: 娃的学校12月要让他们去参加一个数学竞赛。我看了一下历年的样题,基本都是计算题
: (比AMC更加"计算”),所谓难的地方就是用文字绕人。LP和娃都抱怨我给她做的MOEMS
: 太难了,不符合考情。我正在纠结是否继续我的MOEMS大计还是屈从于考试,让她多做
: 计算题。

t*******r
发帖数: 22634
18
但另一方面,对于 dragonbreath (龙吸) 和想在 pre-algebra 挖深挖透的,我觉得
可以考虑推迟 AMC10,先把 MOEMS 挖深挖大的战略。
各家情况不一样。
另外你如果想当前的某种竞赛,那就要跟当前的某种竞赛 align。
我们没有在 10 年级以前参加竞赛的打算,所以我们目前不需要跟任何竞赛直接 align
。我们主要战略就是 “有限小灶”。

MOEMS

【在 l*****8 的大作中提到】
: 相对AMC, MOEMS难多了。AMC主要是覆盖的内容多。其实就如潮水给起的名字一样,基
: 本都是计算题。
: 娃的学校12月要让他们去参加一个数学竞赛。我看了一下历年的样题,基本都是计算题
: (比AMC更加"计算”),所谓难的地方就是用文字绕人。LP和娃都抱怨我给她做的MOEMS
: 太难了,不符合考情。我正在纠结是否继续我的MOEMS大计还是屈从于考试,让她多做
: 计算题。

t*******r
发帖数: 22634
19
移项不是基本的,基本的是两边加上等值的东东。
但这些不是主要纠结,主要的纠结,是你说的五年级的 “baby algebra” 并没有真正
引入 algebra 的 symbolic system,所以实际上还是瓢虫阶段。
而我的观点是,baby algebra 就当 spiral 玩玩就好。而一旦进入真正的 algebra 之
后,要尽可能马不停蹄地快速越过 pre-algebra 阶段。
后来黄小木给出的 Concrete Operation 阶段 到 Formal Operation 阶段 理论,我觉
得是对我的观点的理论支持。真正的 algebra 的本质,是数学学习过程中第一次上
Formal Operation 的认知阶段,而 baby algebra 不算 Formal Operation 阶段。

【在 d*****n 的大作中提到】
: 还有方程,现在国内5年级就教了,但是解方程的方法是等式两边加上或减去同一个数
: ,等式不变。我们那时是初一引入正负数的概念后用移项来解方程的。
: 这个分数的问题我也注意到了,这边都是直来直去,国内的都要拐好几个弯儿。比如我
: 教孩子分数乘法要先约分再相乘,他非要先相乘再约分,为此和我不止一次闹矛盾。
: 他们教的求最大公约和最小公倍的方法也和我们当年不一样。
: 有心按国内的课本教,但是每天回来还要做饭收拾之类的,有时候力不从心。

t*******r
发帖数: 22634
20
另外做 AMC8 应该对 MOEMS 也有帮助,应该不矛盾,如果你家小时数够花的话。
我娃上半年做了几套 AMC8 以后,她说她开始觉得 MOEMS 比以前容易了一些。
当然后我们这里 MOEMS 俱乐部很快就结束了,我们小时数不够也不打算回头再搞
MOEMS 了,所以没有后续的信息。

MOEMS

【在 l*****8 的大作中提到】
: 相对AMC, MOEMS难多了。AMC主要是覆盖的内容多。其实就如潮水给起的名字一样,基
: 本都是计算题。
: 娃的学校12月要让他们去参加一个数学竞赛。我看了一下历年的样题,基本都是计算题
: (比AMC更加"计算”),所谓难的地方就是用文字绕人。LP和娃都抱怨我给她做的MOEMS
: 太难了,不符合考情。我正在纠结是否继续我的MOEMS大计还是屈从于考试,让她多做
: 计算题。

相关主题
小学数学书推荐?怎么推小学数学好?小学一年级的孩子。
我来说一下天才的鉴定方法请问,AMC 8应该从几年级开始练习呀?
[华盛顿邮报] 9 - 14 岁是数学兴趣的关键时期?数学教育 一家之言 番外篇
进入Parenting版参与讨论
t*******r
发帖数: 22634
21
“比如我 教孩子分数乘法要先约分再相乘,他非要先相乘再约分,为此和我不止一次
闹矛盾。”
这个我觉得先在教分数乘法的同时,同时也把 unique prime factorization 的概念给
教了,那就不会闹矛盾了。
因为这玩意儿本质是一回事,unique 保证了本质是一回事。而实战上愿意 “按部就班
” 还是 “乱中取胜”,视情况灵活掌握就完事了。

【在 d*****n 的大作中提到】
: 还有方程,现在国内5年级就教了,但是解方程的方法是等式两边加上或减去同一个数
: ,等式不变。我们那时是初一引入正负数的概念后用移项来解方程的。
: 这个分数的问题我也注意到了,这边都是直来直去,国内的都要拐好几个弯儿。比如我
: 教孩子分数乘法要先约分再相乘,他非要先相乘再约分,为此和我不止一次闹矛盾。
: 他们教的求最大公约和最小公倍的方法也和我们当年不一样。
: 有心按国内的课本教,但是每天回来还要做饭收拾之类的,有时候力不从心。

t*******r
发帖数: 22634
22
外加所有分数都可以分解成 a natural number multiple a unit fraction, 其中的
unit fraction 是 multiplicative inverse of a natural number,的概念。
这样彻底没有争议或矛盾了。

【在 t*******r 的大作中提到】
: “比如我 教孩子分数乘法要先约分再相乘,他非要先相乘再约分,为此和我不止一次
: 闹矛盾。”
: 这个我觉得先在教分数乘法的同时,同时也把 unique prime factorization 的概念给
: 教了,那就不会闹矛盾了。
: 因为这玩意儿本质是一回事,unique 保证了本质是一回事。而实战上愿意 “按部就班
: ” 还是 “乱中取胜”,视情况灵活掌握就完事了。

t*******r
发帖数: 22634
23
还需要 the uniqueness of multiplicative inverse。否则也不行。
这好像还真是不少活。nothing easy for 普通娃,是钢钢的。



【在 t*******r 的大作中提到】
: 外加所有分数都可以分解成 a natural number multiple a unit fraction, 其中的
: unit fraction 是 multiplicative inverse of a natural number,的概念。
: 这样彻底没有争议或矛盾了。

l*****8
发帖数: 16949
24
你说的对。可能我高估了孩子的能力,同时低估了MOEMS的难度。AMC8只要是她学过的
内容,大部分都能自己figure out.但是后面的大概10题内容超过了她的范围,所以听
你提到MOEMS,我就买了几本书,觉得既然是小学内容,难度应该低于AMC,我自己没看就
丢给她了。结果发现难度超过AMC一个数量级。那些分数题,不用方程解起来真心不容
易。
我家跳舞花时间太多,小时数肯定是不够的。我现在纠结的就是继续素质培养还是应付
考试。

【在 t*******r 的大作中提到】
: 另外做 AMC8 应该对 MOEMS 也有帮助,应该不矛盾,如果你家小时数够花的话。
: 我娃上半年做了几套 AMC8 以后,她说她开始觉得 MOEMS 比以前容易了一些。
: 当然后我们这里 MOEMS 俱乐部很快就结束了,我们小时数不够也不打算回头再搞
: MOEMS 了,所以没有后续的信息。
:
: MOEMS

s***n
发帖数: 1280
25
MOEMS包括小初中。初中已经讲了方程,我感觉那些分数题可能就是要大家用方程解。
我儿子一直跟我说MOEMS比AMC 8容易。不过他做MOEMS更容易粗心错。

【在 l*****8 的大作中提到】
: 你说的对。可能我高估了孩子的能力,同时低估了MOEMS的难度。AMC8只要是她学过的
: 内容,大部分都能自己figure out.但是后面的大概10题内容超过了她的范围,所以听
: 你提到MOEMS,我就买了几本书,觉得既然是小学内容,难度应该低于AMC,我自己没看就
: 丢给她了。结果发现难度超过AMC一个数量级。那些分数题,不用方程解起来真心不容
: 易。
: 我家跳舞花时间太多,小时数肯定是不够的。我现在纠结的就是继续素质培养还是应付
: 考试。

l*****8
发帖数: 16949
26
哦,我是从第二册开始做的,里面EM是分开的,我只让她做E的,里面就有好多这种不
用方程很难的题。她没学过方程,简单的类似鸡兔同笼的能做出来,再复杂点就不行了
。还有一个难的地方是MOEMS不是选择题,没法猜。

【在 s***n 的大作中提到】
: MOEMS包括小初中。初中已经讲了方程,我感觉那些分数题可能就是要大家用方程解。
: 我儿子一直跟我说MOEMS比AMC 8容易。不过他做MOEMS更容易粗心错。

t*******r
发帖数: 22634
27
如果用 AMC 题集是用于学习目的而不是备赛目的,一般建议不要先看选择支 (个别题
目例外)。
我开始时都让娃遮掉选择支,不过太麻烦所以现在靠娃自觉了。不过需要我 review 时
,除非必要,不先看选择支,解出数字后再对选择支。
当然如果你近期如果有数学竞赛备赛的话,另说。

【在 l*****8 的大作中提到】
: 哦,我是从第二册开始做的,里面EM是分开的,我只让她做E的,里面就有好多这种不
: 用方程很难的题。她没学过方程,简单的类似鸡兔同笼的能做出来,再复杂点就不行了
: 。还有一个难的地方是MOEMS不是选择题,没法猜。

j*p
发帖数: 780
28
哦俺刚知道MOEMS不是选择题。以前只知道参加了会有一trophy,或是gold pin之类的
东西。

【在 l*****8 的大作中提到】
: 哦,我是从第二册开始做的,里面EM是分开的,我只让她做E的,里面就有好多这种不
: 用方程很难的题。她没学过方程,简单的类似鸡兔同笼的能做出来,再复杂点就不行了
: 。还有一个难的地方是MOEMS不是选择题,没法猜。

t*******r
发帖数: 22634
29
AIME & above 的数学竞赛都不是选择题,只有 AMC 这种大众喜闻乐见的计算题级别的
,才是选择题。// run

【在 j*p 的大作中提到】
: 哦俺刚知道MOEMS不是选择题。以前只知道参加了会有一trophy,或是gold pin之类的
: 东西。

t*******r
发帖数: 22634
30
MOEMS 的方程题,因为数字很小,所以可以直接用 guess & check 的策略。有助于兴
趣。
但我觉得 guess & check 太多意义不大,所以我给娃讲解的都是用 reduction /
deduction 或者偶尔 induction。数字小更容易直接拿积木 / 纸牌 来演示。
但后来我发现,我这积木纸牌瓢虫阵,虽然成功的避免了代数符号系统,但这瓢虫阵对
娃思维的要求,好像不比代数低。Too demanding。这也是我后来放弃 MOEMS,转成
AMC8,期望尽快打好 pre-algebra 的基础尽快上 algebra AMC10。

【在 l*****8 的大作中提到】
: 哦,我是从第二册开始做的,里面EM是分开的,我只让她做E的,里面就有好多这种不
: 用方程很难的题。她没学过方程,简单的类似鸡兔同笼的能做出来,再复杂点就不行了
: 。还有一个难的地方是MOEMS不是选择题,没法猜。

相关主题
小学升初中,儿子数学跳级的一点经验9岁, 如何报名考amc10
有明天考AMC 8的吗?算不算有数学天分
求科普: 奥数 vs. Math Olympiad vs. Math Count vs. Math circle vs. 超前学数学[葩论] 论“有限小灶策略”
进入Parenting版参与讨论
n********h
发帖数: 13135
31
不是参加就有trophy. 学校组织的 moems是多次考试成绩累积在一起。如果全班第一名
,才有trophy,同时总成绩排在national top 的才有pin.比如我娃学校的结果来看,
一道不错才能gold pin. 错一道题silver pin. 多过一道,啥也没有

【在 j*p 的大作中提到】
: 哦俺刚知道MOEMS不是选择题。以前只知道参加了会有一trophy,或是gold pin之类的
: 东西。

l*****8
发帖数: 16949
32
你娃的学校很牛啊。

【在 n********h 的大作中提到】
: 不是参加就有trophy. 学校组织的 moems是多次考试成绩累积在一起。如果全班第一名
: ,才有trophy,同时总成绩排在national top 的才有pin.比如我娃学校的结果来看,
: 一道不错才能gold pin. 错一道题silver pin. 多过一道,啥也没有

l*****8
发帖数: 16949
33
其实猜猜也不错,能培养数字的感觉。不要说这些小孩子题,就是真的做数学研究,很
多时候也是连猜带蒙的。

【在 t*******r 的大作中提到】
: MOEMS 的方程题,因为数字很小,所以可以直接用 guess & check 的策略。有助于兴
: 趣。
: 但我觉得 guess & check 太多意义不大,所以我给娃讲解的都是用 reduction /
: deduction 或者偶尔 induction。数字小更容易直接拿积木 / 纸牌 来演示。
: 但后来我发现,我这积木纸牌瓢虫阵,虽然成功的避免了代数符号系统,但这瓢虫阵对
: 娃思维的要求,好像不比代数低。Too demanding。这也是我后来放弃 MOEMS,转成
: AMC8,期望尽快打好 pre-algebra 的基础尽快上 algebra AMC10。

n********h
发帖数: 13135
34
好像拿不拿pin是靠个人在national的排名。拿trophy是班级名次。娃学校拿trophy的
两娃,是满分,剩下的拿silver pin也不多。不算多牛

【在 l*****8 的大作中提到】
: 你娃的学校很牛啊。
j*p
发帖数: 780
35
确实牛。👍🏻
我们这trophy是年级第一,好学校一道不错才能得有可能。
今年学校报道gold pin 是national top 2% 23-25;silver pin 10% 18-22。

【在 l*****8 的大作中提到】
: 你娃的学校很牛啊。
s***n
发帖数: 1280
36
是的。一学年考五次,每次五道题,奖看总分。全班/team第一有个trophy。满分有个
Medal。Gold pin是national top 2%,Silver pin是 national top 10%。 Gold pin 和
Silver pin的 cutoff 分数每年会不一样。去年是23-25 Gold pin,18-22 Silver
pin。

【在 n********h 的大作中提到】
: 不是参加就有trophy. 学校组织的 moems是多次考试成绩累积在一起。如果全班第一名
: ,才有trophy,同时总成绩排在national top 的才有pin.比如我娃学校的结果来看,
: 一道不错才能gold pin. 错一道题silver pin. 多过一道,啥也没有

n********h
发帖数: 13135
37
我们这么错一道题就只能silver pin了。

【在 j*p 的大作中提到】
: 确实牛。👍🏻
: 我们这trophy是年级第一,好学校一道不错才能得有可能。
: 今年学校报道gold pin 是national top 2% 23-25;silver pin 10% 18-22。

t*******r
发帖数: 22634
38
猜猜是不错,但在小时数有限、也不是不会猜的前提下,我担心搞太多也可能过犹不及。
因为我觉得 “代数猜” vs “算术猜”,不完全是一回事。或者用黄小木的说法,
Formal Operation vs Concrete Operation。
或者说 “算术猜” 是猜数字,“代数猜” 是猜 reduction / induction。

【在 l*****8 的大作中提到】
: 其实猜猜也不错,能培养数字的感觉。不要说这些小孩子题,就是真的做数学研究,很
: 多时候也是连猜带蒙的。

t*******r
发帖数: 22634
39
这个 2% / 10% 是参加的人里面的 2% / 10%,远远好于一般意义上的 National Top
的百分比。
我觉得拿 pin 的都可以考虑将来准备数学竞赛。



【在 s***n 的大作中提到】
: 是的。一学年考五次,每次五道题,奖看总分。全班/team第一有个trophy。满分有个
: Medal。Gold pin是national top 2%,Silver pin是 national top 10%。 Gold pin 和
: Silver pin的 cutoff 分数每年会不一样。去年是23-25 Gold pin,18-22 Silver
: pin。

l*****8
发帖数: 16949
40
啊,一次只考5题啊。我没拎清行情,一次让娃做10-15题,难怪娃抱怨。



【在 s***n 的大作中提到】
: 是的。一学年考五次,每次五道题,奖看总分。全班/team第一有个trophy。满分有个
: Medal。Gold pin是national top 2%,Silver pin是 national top 10%。 Gold pin 和
: Silver pin的 cutoff 分数每年会不一样。去年是23-25 Gold pin,18-22 Silver
: pin。

相关主题
[葩论] 论“有限小灶策略”小学数学有比赛项目吗
给初中学生家长的一些建议 (转载)请教:怎样说服孩儿他爸多关心孩子的学习?
小学数学上超前班有什么好处吗?学校早晚要教的东西,早早学会了又如何?
进入Parenting版参与讨论
t*******r
发帖数: 22634
41
silver pin 很不错了,我觉得你们可以考虑数学竞赛。
我觉得这些竞赛的一个很大的好处,就是可以估计 National Top Percentiles,这样
就可以及时动态上调或下调策略。不至于到 end of day 时突然被 surprise 或者血本
无归。
其实俺也就是看着去年娃的 MOEMS 成绩很差,离开任何 pin 都很远。但学校的
computer adaptive test 的成绩 within expectation。所以就下调下一年度的预期和
目标,慢慢调整到放弃数学竞赛的想法。(当然这也得慢慢来,不能一下子 shock 娃)。

【在 n********h 的大作中提到】
: 我们这么错一道题就只能silver pin了。
t*******r
发帖数: 22634
42
题目左边应该有建议时间的,你多半急着没看。
guess & check 花不少时间的。cut down 时间后的估测不会正确。你应该去重新估测
一次。

【在 l*****8 的大作中提到】
: 啊,一次只考5题啊。我没拎清行情,一次让娃做10-15题,难怪娃抱怨。
:
: 和

l*****8
发帖数: 16949
43
我看到了,觉得时间给得太多了,就直接忽略了。看来是没把自己放在孩子的鞋子里。

【在 t*******r 的大作中提到】
: 题目左边应该有建议时间的,你多半急着没看。
: guess & check 花不少时间的。cut down 时间后的估测不会正确。你应该去重新估测
: 一次。

t*******r
发帖数: 22634
44
大部分时间还是按照冰箱贴的要求,才比较顺手。
或者干脆换个不同的冰箱贴走不同的道路。
大幅度修改冰箱贴不是那么容易的。

【在 l*****8 的大作中提到】
: 我看到了,觉得时间给得太多了,就直接忽略了。看来是没把自己放在孩子的鞋子里。
n********h
发帖数: 13135
45
和我家娃没啥关系。数学竞赛道路上挤满了老中,已经下不去脚了。就冲错一题就
silver pin了,基本可以洗洗睡了。

)。

【在 t*******r 的大作中提到】
: silver pin 很不错了,我觉得你们可以考虑数学竞赛。
: 我觉得这些竞赛的一个很大的好处,就是可以估计 National Top Percentiles,这样
: 就可以及时动态上调或下调策略。不至于到 end of day 时突然被 surprise 或者血本
: 无归。
: 其实俺也就是看着去年娃的 MOEMS 成绩很差,离开任何 pin 都很远。但学校的
: computer adaptive test 的成绩 within expectation。所以就下调下一年度的预期和
: 目标,慢慢调整到放弃数学竞赛的想法。(当然这也得慢慢来,不能一下子 shock 娃)。

t*******r
发帖数: 22634
46
但 AIME 错三题还是能进 AMO qualifiers 吧。也不至于洗洗睡了。

【在 n********h 的大作中提到】
: 和我家娃没啥关系。数学竞赛道路上挤满了老中,已经下不去脚了。就冲错一题就
: silver pin了,基本可以洗洗睡了。
:
: )。

j*p
发帖数: 780
47
差一分也说明不了什么。我们学校有一位六年级得perfect score,但AMC 8四到八年级
school winner还是我家孩子。:-)
和另外一位ID说的一样,今年23-25似乎都应该是gold pin。

★ 发自iPhone App: ChineseWeb 1.0.3

【在 n********h 的大作中提到】
: 和我家娃没啥关系。数学竞赛道路上挤满了老中,已经下不去脚了。就冲错一题就
: silver pin了,基本可以洗洗睡了。
:
: )。

t*******r
发帖数: 22634
48
先进来 cong 一个。

【在 j*p 的大作中提到】
: 差一分也说明不了什么。我们学校有一位六年级得perfect score,但AMC 8四到八年级
: school winner还是我家孩子。:-)
: 和另外一位ID说的一样,今年23-25似乎都应该是gold pin。
:
: ★ 发自iPhone App: ChineseWeb 1.0.3

s***n
发帖数: 1280
49
MOEMS的影响力比较小。很多竞赛娃不玩这个,所以从它估计national top
percentiles偏差会比较大。
另外小初中数学竞赛和高中数学竞赛的考察的侧重点不一样。初中偏抽象思维,高中偏
逻辑思维。初中竞赛好的孩子未必高中竞赛好。这种情况不少见。
小初中阶段很多孩子的逻辑思维还在发展,变数比较大。
一句话,MOEMS的national top percentiles对高中竞赛的参考意义不太。

)。

【在 t*******r 的大作中提到】
: silver pin 很不错了,我觉得你们可以考虑数学竞赛。
: 我觉得这些竞赛的一个很大的好处,就是可以估计 National Top Percentiles,这样
: 就可以及时动态上调或下调策略。不至于到 end of day 时突然被 surprise 或者血本
: 无归。
: 其实俺也就是看着去年娃的 MOEMS 成绩很差,离开任何 pin 都很远。但学校的
: computer adaptive test 的成绩 within expectation。所以就下调下一年度的预期和
: 目标,慢慢调整到放弃数学竞赛的想法。(当然这也得慢慢来,不能一下子 shock 娃)。

Y********d
发帖数: 1478
50

同祝贺,还有头家和北滩家娃!
看到聪明好学娃忍不住喜欢,算不算职业病?
让你家大娃加油哩!让她哪天拿个trophy拿个pin,阿姨跟她爸说情,她可以多看几集
fiction:-)

【在 t*******r 的大作中提到】
: 先进来 cong 一个。
相关主题
AMC 8 成绩 2016我来说一下天才的鉴定方法
请教小孩数学教育问题[华盛顿邮报] 9 - 14 岁是数学兴趣的关键时期?
小学数学书推荐?怎么推小学数学好?小学一年级的孩子。
进入Parenting版参与讨论
t*******r
发帖数: 22634
51
其实我曾经在 MOEMS 上花了不少时间。我一度对 MOEMS 非常推崇,因为 MOEMS 的题
的数字都很小,题又很有趣,因此可以在算术到前代数阶段,不使用符号系统 (
symbolic system),避免引入代数,用瓢虫阵和基于集合论图论的思想来解题。
但是,正如黄小木同学的 Concrete Operation 到 Formal Operation 的阶段论,我虽
然避免了代数和任何符号系统,但这个瓢虫阵,实际上还是无法避免其背后 Formal
Operation 的本质,对娃而言,这可能不比代数简单,可能还不如先学代数。
但是另一方面,我似乎觉得,从 Concrete Operation 到 Formal Operation 到
Beyond Formal Operation,年龄似乎不是唯一的因素,而 practice 和 repetition
可能影响也很大。因为 Formal Operation 在日常生活中几乎不咋出现,小时数不够的
话,一把年纪可能也是白搭。

【在 l*****8 的大作中提到】
: 我看到了,觉得时间给得太多了,就直接忽略了。看来是没把自己放在孩子的鞋子里。
t*******r
发帖数: 22634
52
我娃她要想拿 trophy,她自然会奔那个方向。
Trophy 不是我谈判专家的谈判标的,我更重要的小时数都谈判不下来,trophy 我还是
算了不放上谈判桌了。

【在 Y********d 的大作中提到】
:
: 同祝贺,还有头家和北滩家娃!
: 看到聪明好学娃忍不住喜欢,算不算职业病?
: 让你家大娃加油哩!让她哪天拿个trophy拿个pin,阿姨跟她爸说情,她可以多看几集
: fiction:-)

n********h
发帖数: 13135
53
我说了和我家娃没啥关系。因为学校组织的,每个学生都参加,所以知道点儿情况。

【在 j*p 的大作中提到】
: 差一分也说明不了什么。我们学校有一位六年级得perfect score,但AMC 8四到八年级
: school winner还是我家孩子。:-)
: 和另外一位ID说的一样,今年23-25似乎都应该是gold pin。
:
: ★ 发自iPhone App: ChineseWeb 1.0.3

t*******r
发帖数: 22634
54
全员参加的话,这得建多少个 team 的说。

【在 n********h 的大作中提到】
: 我说了和我家娃没啥关系。因为学校组织的,每个学生都参加,所以知道点儿情况。
n********h
发帖数: 13135
55
Trophy这是可遇不可求。我娃学校有几个数学牛娃,而且基本不会分在一个班里。剩下
的,trophy基本无望

【在 t*******r 的大作中提到】
: 我娃她要想拿 trophy,她自然会奔那个方向。
: Trophy 不是我谈判专家的谈判标的,我更重要的小时数都谈判不下来,trophy 我还是
: 算了不放上谈判桌了。

n********h
发帖数: 13135
56
一个班一个team.数学完蛋娃也要被迫参加当分母。不过不收额外报名费,不用家长接
送,不用准备服装,比当钢琴比赛的分母强。

【在 t*******r 的大作中提到】
: 全员参加的话,这得建多少个 team 的说。
t*******r
发帖数: 22634
57
属实。

【在 n********h 的大作中提到】
: Trophy这是可遇不可求。我娃学校有几个数学牛娃,而且基本不会分在一个班里。剩下
: 的,trophy基本无望

t*******r
发帖数: 22634
58
你们比较狠。我们是 invitation-basis 的,总共好像只有两个队。还是早上一大早在
上学前干活,我看到不少娃是梦游的姿势走入教室。
不过这个 MOEMS 本来就不在学校正规教程里。也不能也不该喧宾夺主。无可厚非的。
为初中数学分 track 的 Math Placement & Placement Test 才是老师和家长们盯着的
玩意儿。Priority is the King。从这方面而言,我对小学基本满意。

【在 n********h 的大作中提到】
: 一个班一个team.数学完蛋娃也要被迫参加当分母。不过不收额外报名费,不用家长接
: 送,不用准备服装,比当钢琴比赛的分母强。

Y********d
发帖数: 1478
59

发现黄小木同学在这个楼里被你点名肯定过三次了,红着脸高兴一把 :-P
不过那个理论是Piaget老先生的,不是我自己的。当然,现学现卖的本事我这个书呆子
是有的。
昨天看到一本书上说,我们的大脑主要是三个因素决定的。有了适当的食物,睡眠和刺
激,自然的力量就可以保证大脑的基本结构得到良好发展;而人最后会表现出什么样子
,基因起到了很大的作用,尤其在性格脾气方面。但是,最近几十年大脑扫描科学的发
展,有了一个颠覆脑神经科学的惊人发现,那就是我们的大脑是有弹性的,或者说可以
重塑的。而且大脑的物理变化在我们的一生中一直在发生着,而不是像我们从前假定的
那样,只发生在儿童时期。
(这个让我想起了你以前讲到过“大脑灰质剪枝理论”,但是应该也不算矛盾。一个是
从redundant到remaining;另一个是从remaining到remolding)
那么什么东西重塑我们的大脑呢?经验。即便在很老的时候,我们的经验依旧可以改变
大脑的物理结构:大脑神经元在受到外界刺激时被激活--〉神经元之间有连接并形成通
路--〉每一个特定通路是如何被激活的决定我们的思维活动--〉多个神经元被同时激活
后可以在她们之间产生新的通路--〉加以时日,这些通路就会导致大脑的重新连接。这
个过程不仅对孩子和少年成立,而且发生在我们每一个人的一生之中。
因此,在大脑的基本结构和天生的性格脾气之外,父母其实是可以通过给孩子提供特定
的经验,以帮助孩子的大脑更有弹性和更加完善。
这个应该与你上面的那段话一致:
“我似乎觉得,从 Concrete Operation 到 Formal Operation 到 Beyond Formal
Operation,年龄似乎不是唯一的因素,而 practice 和 repetition 可能影响也很大
。因为 Formal Operation 在日常生活中几乎不咋出现,小时数不够的话,一把年纪可
能也是白搭。”
你的practice 和 repetition应该就是经验的意思了。小时数不够就是通路还没有完全
形成。
算不算为你的直觉提供点理论支持?
你觉得我要是写个上面这样的读书笔记有意思吗?
要是你也觉得没有意思,我就不写了,码字也挺烦的说。

【在 t*******r 的大作中提到】
: 其实我曾经在 MOEMS 上花了不少时间。我一度对 MOEMS 非常推崇,因为 MOEMS 的题
: 的数字都很小,题又很有趣,因此可以在算术到前代数阶段,不使用符号系统 (
: symbolic system),避免引入代数,用瓢虫阵和基于集合论图论的思想来解题。
: 但是,正如黄小木同学的 Concrete Operation 到 Formal Operation 的阶段论,我虽
: 然避免了代数和任何符号系统,但这个瓢虫阵,实际上还是无法避免其背后 Formal
: Operation 的本质,对娃而言,这可能不比代数简单,可能还不如先学代数。
: 但是另一方面,我似乎觉得,从 Concrete Operation 到 Formal Operation 到
: Beyond Formal Operation,年龄似乎不是唯一的因素,而 practice 和 repetition
: 可能影响也很大。因为 Formal Operation 在日常生活中几乎不咋出现,小时数不够的
: 话,一把年纪可能也是白搭。

P******e
发帖数: 1325
60
这本书的书名什么?听起来很有意思。不知道成年人的大脑能被重塑的程度怎样?
另外,经验的作用,是指自己的经验积累吧?父母的传授跟教科书有什么不同?
支持你写读书笔记,会有很多人有兴趣讨论的。

★ 发自iPhone App: ChineseWeb 1.0.2
★ 发自iPhone App: ChineseWeb 1.0.2

【在 Y********d 的大作中提到】
:
: 发现黄小木同学在这个楼里被你点名肯定过三次了,红着脸高兴一把 :-P
: 不过那个理论是Piaget老先生的,不是我自己的。当然,现学现卖的本事我这个书呆子
: 是有的。
: 昨天看到一本书上说,我们的大脑主要是三个因素决定的。有了适当的食物,睡眠和刺
: 激,自然的力量就可以保证大脑的基本结构得到良好发展;而人最后会表现出什么样子
: ,基因起到了很大的作用,尤其在性格脾气方面。但是,最近几十年大脑扫描科学的发
: 展,有了一个颠覆脑神经科学的惊人发现,那就是我们的大脑是有弹性的,或者说可以
: 重塑的。而且大脑的物理变化在我们的一生中一直在发生着,而不是像我们从前假定的
: 那样,只发生在儿童时期。

相关主题
请问,AMC 8应该从几年级开始练习呀?有明天考AMC 8的吗?
数学教育 一家之言 番外篇求科普: 奥数 vs. Math Olympiad vs. Math Count vs. Math circle vs. 超前学数学
小学升初中,儿子数学跳级的一点经验9岁, 如何报名考amc10
进入Parenting版参与讨论
t*******r
发帖数: 22634
61
“但是,最近几十年大脑扫描科学的发 展,有了一个颠覆脑神经科学的惊人发现,那
就是我们的大脑是有弹性的,或者说可以 重塑的。而且大脑的物理变化在我们的一生
中一直在发生着,而不是像我们从前假定的 那样,只发生在儿童时期。”
我倾向于支持这个说法。原因是我时而不时觉得我娃缺乏数学里最最基本的 instinct
。但后来想了 n 久以后,觉得我自己的 instinct 很可能不是 born 而是 acquired。

【在 Y********d 的大作中提到】
:
: 发现黄小木同学在这个楼里被你点名肯定过三次了,红着脸高兴一把 :-P
: 不过那个理论是Piaget老先生的,不是我自己的。当然,现学现卖的本事我这个书呆子
: 是有的。
: 昨天看到一本书上说,我们的大脑主要是三个因素决定的。有了适当的食物,睡眠和刺
: 激,自然的力量就可以保证大脑的基本结构得到良好发展;而人最后会表现出什么样子
: ,基因起到了很大的作用,尤其在性格脾气方面。但是,最近几十年大脑扫描科学的发
: 展,有了一个颠覆脑神经科学的惊人发现,那就是我们的大脑是有弹性的,或者说可以
: 重塑的。而且大脑的物理变化在我们的一生中一直在发生着,而不是像我们从前假定的
: 那样,只发生在儿童时期。

t*******r
发帖数: 22634
62
我觉得 AMC8 的第 15 题到第 25 题,即使娃做不出来,只要娃能慢慢理解,我觉得也
算 ready 值得试着慢慢做,对/读答案。
我觉得后面的题目的好处在于,出现娃学校里没有见过的模式,容易发现概念上的漏洞
。因为探讨 why I am unable to deduction/induction for this new problem 是发
现概念漏洞的利器。
当然这种情况不作速度和正确率的要求。

【在 l*****8 的大作中提到】
: 你说的对。可能我高估了孩子的能力,同时低估了MOEMS的难度。AMC8只要是她学过的
: 内容,大部分都能自己figure out.但是后面的大概10题内容超过了她的范围,所以听
: 你提到MOEMS,我就买了几本书,觉得既然是小学内容,难度应该低于AMC,我自己没看就
: 丢给她了。结果发现难度超过AMC一个数量级。那些分数题,不用方程解起来真心不容
: 易。
: 我家跳舞花时间太多,小时数肯定是不够的。我现在纠结的就是继续素质培养还是应付
: 考试。

l*****8
发帖数: 16949
63
很多内容没学过,比如勾股弦定理,完全没法下手(她不可能天才到琢磨出这个定理吧
。)。我现在担心的是上太多太难太超前的东西是否会打击她的积极性。

【在 t*******r 的大作中提到】
: 我觉得 AMC8 的第 15 题到第 25 题,即使娃做不出来,只要娃能慢慢理解,我觉得也
: 算 ready 值得试着慢慢做,对/读答案。
: 我觉得后面的题目的好处在于,出现娃学校里没有见过的模式,容易发现概念上的漏洞
: 。因为探讨 why I am unable to deduction/induction for this new problem 是发
: 现概念漏洞的利器。
: 当然这种情况不作速度和正确率的要求。

d**********h
发帖数: 2795
64
我觉得是这样。
题海除了强化,另一个重要功用就是“补漏”
牛娃不用补漏,人家举一反三,触类旁通
一般娃就差些,要点到了,才知道,才明白
当然不开窍娃,就没得说了,大家一起修炼,父母练心性,娃儿练脑瓜儿
其实这个练题型的方法也像训练机器人,题型分的越细,规律越清晰,越容易follow,
最后娃就机器人化了

【在 t*******r 的大作中提到】
: 我觉得 AMC8 的第 15 题到第 25 题,即使娃做不出来,只要娃能慢慢理解,我觉得也
: 算 ready 值得试着慢慢做,对/读答案。
: 我觉得后面的题目的好处在于,出现娃学校里没有见过的模式,容易发现概念上的漏洞
: 。因为探讨 why I am unable to deduction/induction for this new problem 是发
: 现概念漏洞的利器。
: 当然这种情况不作速度和正确率的要求。

s***n
发帖数: 1280
65
是的。要遁序渐进。现在这个阶段,AMC 8只是帮你大概了解孩子的相对水平。孩子的
数学教育还是应该按照本来的次序。

【在 l*****8 的大作中提到】
: 很多内容没学过,比如勾股弦定理,完全没法下手(她不可能天才到琢磨出这个定理吧
: 。)。我现在担心的是上太多太难太超前的东西是否会打击她的积极性。

t*******r
发帖数: 22634
66
勾股定理可以先教非代数证明,mathsisfuns.com 上应该有证明。
上 algebra 概念后可以教利用代数的证明,更直截了当。
教完以后其实娃也不会证明,但是会认可勾股定理。
顺便还可以说明 sqrt(2) 是无理数,但同时是落在数轴上的点,慢慢建立实数的概念。
其实我昨晚又跟娃对攻实数的概念,因为她有题不理解也不会琢磨。
这些我都不期望娃能会,只是让娃知道数学并不仅仅是 follow the routine。

【在 l*****8 的大作中提到】
: 很多内容没学过,比如勾股弦定理,完全没法下手(她不可能天才到琢磨出这个定理吧
: 。)。我现在担心的是上太多太难太超前的东西是否会打击她的积极性。

l*****8
发帖数: 16949
67
这个太深了点,现在真没有那么多时间。慢慢来吧。

念。

【在 t*******r 的大作中提到】
: 勾股定理可以先教非代数证明,mathsisfuns.com 上应该有证明。
: 上 algebra 概念后可以教利用代数的证明,更直截了当。
: 教完以后其实娃也不会证明,但是会认可勾股定理。
: 顺便还可以说明 sqrt(2) 是无理数,但同时是落在数轴上的点,慢慢建立实数的概念。
: 其实我昨晚又跟娃对攻实数的概念,因为她有题不理解也不会琢磨。
: 这些我都不期望娃能会,只是让娃知道数学并不仅仅是 follow the routine。

t*******r
发帖数: 22634
68
也不会成为机器人。昨晚我跟娃对攻到最后,我问娃,“Can't you make a thought
experiment which can testify whether a person (or a creature) understand
real number (or natural number, negative number, fraction)”。
娃的第一个回答是,战战兢兢地说。。。 “if he (or she) say he understand real
number?”

【在 d**********h 的大作中提到】
: 我觉得是这样。
: 题海除了强化,另一个重要功用就是“补漏”
: 牛娃不用补漏,人家举一反三,触类旁通
: 一般娃就差些,要点到了,才知道,才明白
: 当然不开窍娃,就没得说了,大家一起修炼,父母练心性,娃儿练脑瓜儿
: 其实这个练题型的方法也像训练机器人,题型分的越细,规律越清晰,越容易follow,
: 最后娃就机器人化了

t*******r
发帖数: 22634
69
我在 MOEMS 的时候跟娃搞过那个勾股定理好玩版。都玩到剪纸拼图的程度。
反正 MOEMS 有趣味,那时没有其他压力,娃又有的是时间浪费来瞎玩。

【在 l*****8 的大作中提到】
: 这个太深了点,现在真没有那么多时间。慢慢来吧。
:
: 念。

s***n
发帖数: 1280
70
我觉得数学的逻辑思维能力类似智商,是一半看天生(可能是大半),一半看后天培训。
不过和人的记忆能力不同。天生的记忆能力能在学龄前就表现出来,但天生的逻辑思维
能力要到青少年期才完全解码显示。逻辑方面早慧的孩子可以很早就理解比较复杂的数
学概念,但也有不少晚慧的孩子,他们在逻辑思维方面的优势会比较晚才显现。某种意
义上,这就是静淌派等的东西。

instinct

【在 t*******r 的大作中提到】
: “但是,最近几十年大脑扫描科学的发 展,有了一个颠覆脑神经科学的惊人发现,那
: 就是我们的大脑是有弹性的,或者说可以 重塑的。而且大脑的物理变化在我们的一生
: 中一直在发生着,而不是像我们从前假定的 那样,只发生在儿童时期。”
: 我倾向于支持这个说法。原因是我时而不时觉得我娃缺乏数学里最最基本的 instinct
: 。但后来想了 n 久以后,觉得我自己的 instinct 很可能不是 born 而是 acquired。

相关主题
9岁, 如何报名考amc10给初中学生家长的一些建议 (转载)
算不算有数学天分小学数学上超前班有什么好处吗?
[葩论] 论“有限小灶策略”小学数学有比赛项目吗
进入Parenting版参与讨论
t*******r
发帖数: 22634
71
其实我觉得 MOEMS 对帮助娃建立 proof 的概念挺好的,因为数字小,又有很多容易做
到 proof 的每一步都非常 obvious (从娃的角度) 的 discrete problem。缺点是需要
父母较多的 engage,也就是在娃 guess & check 之后,父母用瓢虫阵法 work out
the results,并且在瓢虫阵的每一步的 deduction,都做到对娃是 so obvious that
娃 have no question of that.
有时候我觉得这个是我搞 MOEMS 最大的收获了。至少以后娃就不会 question the
concept of proof,重点可以转到 How do we work it out?

【在 s***n 的大作中提到】
: 我觉得数学的逻辑思维能力类似智商,是一半看天生(可能是大半),一半看后天培训。
: 不过和人的记忆能力不同。天生的记忆能力能在学龄前就表现出来,但天生的逻辑思维
: 能力要到青少年期才完全解码显示。逻辑方面早慧的孩子可以很早就理解比较复杂的数
: 学概念,但也有不少晚慧的孩子,他们在逻辑思维方面的优势会比较晚才显现。某种意
: 义上,这就是静淌派等的东西。
:
: instinct

d**********h
发帖数: 2795
72
我看到的几年前的说法是:研究发现有些日本人37/38岁的时候大脑还在发育。不知道为
啥说日本人,也可能做调查的是个日本小组。
大脑发育有先有后,自然之道。而且也是教育的基础,也是decay的生理基础
我想近四十大脑还发育而没有停滞,可能真的不容易。
其实,生生死死是个自然规律,该死不死可能就有癌变的危险了(当然,天才除外) :)

instinct

【在 t*******r 的大作中提到】
: “但是,最近几十年大脑扫描科学的发 展,有了一个颠覆脑神经科学的惊人发现,那
: 就是我们的大脑是有弹性的,或者说可以 重塑的。而且大脑的物理变化在我们的一生
: 中一直在发生着,而不是像我们从前假定的 那样,只发生在儿童时期。”
: 我倾向于支持这个说法。原因是我时而不时觉得我娃缺乏数学里最最基本的 instinct
: 。但后来想了 n 久以后,觉得我自己的 instinct 很可能不是 born 而是 acquired。

t*******r
发帖数: 22634
73
应该是 “变化” 而不是 “发育” -- neuroplasticity,看起来像智慧生物为了适应
环境变化,又不能很容易地再长条胳膊出来,的折中。
不过从适应环境变化的角度看,USMC 型 比 日不落帝国型,稍稍占优。

【在 d**********h 的大作中提到】
: 我看到的几年前的说法是:研究发现有些日本人37/38岁的时候大脑还在发育。不知道为
: 啥说日本人,也可能做调查的是个日本小组。
: 大脑发育有先有后,自然之道。而且也是教育的基础,也是decay的生理基础
: 我想近四十大脑还发育而没有停滞,可能真的不容易。
: 其实,生生死死是个自然规律,该死不死可能就有癌变的危险了(当然,天才除外) :)
:
: instinct

t*******r
发帖数: 22634
74
spiral 一次勾股定理证明,两小时咋也搞定了。再忙也不缺两小时其实。
而且勾股定理属于 “一招鲜,吃遍天” 的级别的,这种级别的 定理/公理,整个中小
学阶段也没多少。
我觉得如果不是为了速度正确率去比赛,而是慢慢琢磨学习的话,也不需要那么按部就
班嘛。也可以 acquire knowledge on-demand 嘛。
要我说一句,同志们要在与题目和蒋军的斗争中学习,(大手一挥),千里跃进大别山,
在乱中取胜。(机要员同志) 烦转交小平同志去办 。 -- 毛泽东

【在 l*****8 的大作中提到】
: 这个太深了点,现在真没有那么多时间。慢慢来吧。
:
: 念。

s***n
发帖数: 1280
75
对于普通娃,乱中取胜的概率比较小,乱成浆糊的机会比较大。

【在 t*******r 的大作中提到】
: spiral 一次勾股定理证明,两小时咋也搞定了。再忙也不缺两小时其实。
: 而且勾股定理属于 “一招鲜,吃遍天” 的级别的,这种级别的 定理/公理,整个中小
: 学阶段也没多少。
: 我觉得如果不是为了速度正确率去比赛,而是慢慢琢磨学习的话,也不需要那么按部就
: 班嘛。也可以 acquire knowledge on-demand 嘛。
: 要我说一句,同志们要在与题目和蒋军的斗争中学习,(大手一挥),千里跃进大别山,
: 在乱中取胜。(机要员同志) 烦转交小平同志去办 。 -- 毛泽东

t*******r
发帖数: 22634
76
我觉得主要还是 “乱中取胜” 需要有相应的 infrastructure 的支持,否则的确更容
易出现的情况是被敌人包了饺子。

【在 s***n 的大作中提到】
: 对于普通娃,乱中取胜的概率比较小,乱成浆糊的机会比较大。
Y********d
发帖数: 1478
77

The Whole Brain Child,版上的hcww网友推荐的。
确实很有意思,主要从解释大脑如何工作的角度来讲具体的养娃策略。
是个好问题。但是书上没有定量的结论。我也颇好奇。
书里没有区分具体的经验获得方式(就我读到的部分而言),而是强调父母给予孩子经
验,比如帮助孩子重现危机发生的经过,可以让娃的头脑得到更好的整合,即平衡感情
和理智,小脑和大脑。
主要是我没有若水姐姐那种三言两语概括的能力,估计写出来会又臭又长。你要是有兴
趣不妨一起读啊?倒时候开个楼大家一起讨论吧。那个书不厚,就171页,还有mp3和
kindle版本。

【在 P******e 的大作中提到】
: 这本书的书名什么?听起来很有意思。不知道成年人的大脑能被重塑的程度怎样?
: 另外,经验的作用,是指自己的经验积累吧?父母的传授跟教科书有什么不同?
: 支持你写读书笔记,会有很多人有兴趣讨论的。
:
: ★ 发自iPhone App: ChineseWeb 1.0.2
: ★ 发自iPhone App: ChineseWeb 1.0.2

Y********d
发帖数: 1478
78

书上说,a person's brain isn't considered fully developed until she reaches
her mid-twenties.
到底记忆力和逻辑思维能力在具体什么年龄分别完全显现出来,我也一直很好奇。这本
书里没有讲到(就我读过的部分而言),你读到过什么资料吗?
所以,有些地方的小学里,三年级就进行所谓智力测试的分班考试,然后根据成绩分成
慢班快班,从此走上完全不同的轨道,让人想想就觉得很愤怒。

【在 s***n 的大作中提到】
: 我觉得数学的逻辑思维能力类似智商,是一半看天生(可能是大半),一半看后天培训。
: 不过和人的记忆能力不同。天生的记忆能力能在学龄前就表现出来,但天生的逻辑思维
: 能力要到青少年期才完全解码显示。逻辑方面早慧的孩子可以很早就理解比较复杂的数
: 学概念,但也有不少晚慧的孩子,他们在逻辑思维方面的优势会比较晚才显现。某种意
: 义上,这就是静淌派等的东西。
:
: instinct

Y********d
发帖数: 1478
79

看来还有机会抓住最后的几年努力冲刺,变得更加聪明 :)
聪明是没戏了,根据书上的说法,变得更加resilient and integrated.

【在 d**********h 的大作中提到】
: 我看到的几年前的说法是:研究发现有些日本人37/38岁的时候大脑还在发育。不知道为
: 啥说日本人,也可能做调查的是个日本小组。
: 大脑发育有先有后,自然之道。而且也是教育的基础,也是decay的生理基础
: 我想近四十大脑还发育而没有停滞,可能真的不容易。
: 其实,生生死死是个自然规律,该死不死可能就有癌变的危险了(当然,天才除外) :)
:
: instinct

Y********d
发帖数: 1478
80

是滴,百科全书哥哥,书上用的词就是 plastic :)
感觉你的USMC型更加resilient; 日不落帝国型更加integrated;都很重要啊!

【在 t*******r 的大作中提到】
: 应该是 “变化” 而不是 “发育” -- neuroplasticity,看起来像智慧生物为了适应
: 环境变化,又不能很容易地再长条胳膊出来,的折中。
: 不过从适应环境变化的角度看,USMC 型 比 日不落帝国型,稍稍占优。

相关主题
请教:怎样说服孩儿他爸多关心孩子的学习?请教小孩数学教育问题
学校早晚要教的东西,早早学会了又如何?小学数学书推荐?
AMC 8 成绩 2016我来说一下天才的鉴定方法
进入Parenting版参与讨论
Y********d
发帖数: 1478
81
看到这两条,我实在是忍不住笑出来了。
其实一个是觉得学校教育相当稳扎稳打的提供了基础设施,父母给来点锦上添花,塞外
飞仙。
另一个是觉得美帝这些学校教育怎么够格,父母要帮助举一反三,触类旁通。
这个事情上,潮水到有点top-down,而头到有点bottom-up的意思。
预祝两军最终胜利会师阿尔卑斯山!

【在 t*******r 的大作中提到】
: 我觉得主要还是 “乱中取胜” 需要有相应的 infrastructure 的支持,否则的确更容
: 易出现的情况是被敌人包了饺子。

t*******r
发帖数: 22634
82
其实我的看法是:
如果把 Formal Operation 和 Beyond Formal Operation 所需要的 infrastructure,
比喻为一张 decision graph。
那么,这张 decision graph 的 leaf node,才是 school system testable &
verifiable 的,non-leaf node 不行。
所以从这个角度,school system 的 repetitive training,就是先不管三七二十一一
条一条 decision path 给 build 起来,因为每条 decision path 的起点和终点都是
leaf node,这样的好处显然是 trackable & accountable。而且再不济也能练出个人
肉计算器/人肉积分器,在过去的肉算年代,再不济也能当个售货员珠算冠军。
但缺点我觉得也是明显的,也就是这么一大堆 repetitive training 出来的 decision
paths,能否某日突然幡然醒悟,自发自觉形成 decision graph 级的大张战略形势图
,老实说,基本看造化,偶然性狠大。

【在 Y********d 的大作中提到】
: 看到这两条,我实在是忍不住笑出来了。
: 其实一个是觉得学校教育相当稳扎稳打的提供了基础设施,父母给来点锦上添花,塞外
: 飞仙。
: 另一个是觉得美帝这些学校教育怎么够格,父母要帮助举一反三,触类旁通。
: 这个事情上,潮水到有点top-down,而头到有点bottom-up的意思。
: 预祝两军最终胜利会师阿尔卑斯山!

d**********h
发帖数: 2795
83
你说的没错,但是不是美国需要的。天朝在建国初期的一穷二白,到改开之后的快速发
展都急需科技人才,工程师之类的。
但是,美国的人才需求不同,教育的侧重也不同。以前不是说嘛,米国对中国的最大出
口是好莱坞电影。
所以美国不需要那么多数学人才,连分母都不需要。
所以,米国很注重人才分流,也如我听说的德国注重技工学校教育一样。


decision

【在 t*******r 的大作中提到】
: 其实我的看法是:
: 如果把 Formal Operation 和 Beyond Formal Operation 所需要的 infrastructure,
: 比喻为一张 decision graph。
: 那么,这张 decision graph 的 leaf node,才是 school system testable &
: verifiable 的,non-leaf node 不行。
: 所以从这个角度,school system 的 repetitive training,就是先不管三七二十一一
: 条一条 decision path 给 build 起来,因为每条 decision path 的起点和终点都是
: leaf node,这样的好处显然是 trackable & accountable。而且再不济也能练出个人
: 肉计算器/人肉积分器,在过去的肉算年代,再不济也能当个售货员珠算冠军。
: 但缺点我觉得也是明显的,也就是这么一大堆 repetitive training 出来的 decision

Y********d
发帖数: 1478
84


decision
读你的帖子一定要google在手啊,我还要特地去学习一下decision graph和leaf node。
如果我理解正确的话,你这个帖子算是正式定义了repetitive training,这个事情我
也一直想问你但是又忘了问的,所以在你的定义里, repetitive training就是起点和
终点都确定的决策路径,而学校正规的重复训练就是在这条路径上让孩子反复走?
这个狭义的定义我基本同意,而且觉得相当形象。
但是总觉得和广义的重复训练有所不同,因为我们常说举一反三,触类旁通,也许就是
引导娃在走过很多次起点和终点都确定的决策路径之后,有没有其他引向non-leaf
node的决策路径?这个确实需要引导的,需要父母给予孩子经历的,或者就是你说的乱
中取胜?在很多次乱中取胜的经历之后,娃终于哪一天大脑的通路终于形成,也就是新
的decision path出来了?很多条新的decision path编织成战略级别的decision graph?
我能不能以后做你的官方发言人? :-)
不过,老实讲,这也就是常规学校考试的弊端,问太多close-ended problem了。以后
的社会里,孩子需要面对的是越来越多open-ended problem,甚至,更重要的是寻找
problem的能力。

【在 t*******r 的大作中提到】
: 其实我的看法是:
: 如果把 Formal Operation 和 Beyond Formal Operation 所需要的 infrastructure,
: 比喻为一张 decision graph。
: 那么,这张 decision graph 的 leaf node,才是 school system testable &
: verifiable 的,non-leaf node 不行。
: 所以从这个角度,school system 的 repetitive training,就是先不管三七二十一一
: 条一条 decision path 给 build 起来,因为每条 decision path 的起点和终点都是
: leaf node,这样的好处显然是 trackable & accountable。而且再不济也能练出个人
: 肉计算器/人肉积分器,在过去的肉算年代,再不济也能当个售货员珠算冠军。
: 但缺点我觉得也是明显的,也就是这么一大堆 repetitive training 出来的 decision

s***n
发帖数: 1280
85
先推荐一本书:NurtureShock - New Thinking About Children。里面分十个主题介绍
了近十几年教育学研究和心理学研究的进展。很多研究进展都有别于大家早前的认识,
所以说shock。比如这本书就提到那些在Kindergarten就考上GT的孩子只有29%能通过三
年级的GT考试。而那些初中的GT生很多在高中还是GT。换言之,在K阶段起跑赢了的孩
子很多一进到途中跑阶段就落后了。我想很多教育政策制定者可能知道这个结论 - 我
附近几个County的GT筛选在小初高阶段各有一次,晚慧娃有机会翻身。
记忆力和逻辑思维能力成熟的年龄是我根据个人观察 + 书本知识 + Wiki + 网上讨论
有个大致的猜测,很不科学和严谨。比如记忆力,细分的话,主要有长期记忆力和短期
记忆力,前者学龄后基本没什么变化,后者学龄后可以线性发展到青少年期。在记忆力
方面有天赋的孩子在学龄前会比较惹眼,比如那些背诗认字小神童。但这些孩子的理解
力/抽象思维能力,广义创造力/逻辑思维能力并不一定比其他孩子突出,所以他们中的
很多人成年后智力会泯然众人。
逻辑思维能力的发展我提到的那本书有点讨论。我印象是说逻辑思维一般到初中阶段才
大致成熟 - 这是为什么需要逻辑思维的代数和几何一般要到高中后才讲;这也是为什
么初中的GT生和高中的GT生相关性比较好。某种意义上这是静淌派的理论基础 - 在孩
子逻辑思维能力还没ready的时候,不要太着急的往前推。
BTW: 我印象中20多岁才成熟的那部分脑子是和判断力有关。那个结论好像是保险公司
和租车公司"歧视"26岁以下顾客的理论基础。

reaches

【在 Y********d 的大作中提到】
:
: 是
: decision
: 读你的帖子一定要google在手啊,我还要特地去学习一下decision graph和leaf node。
: 如果我理解正确的话,你这个帖子算是正式定义了repetitive training,这个事情我
: 也一直想问你但是又忘了问的,所以在你的定义里, repetitive training就是起点和
: 终点都确定的决策路径,而学校正规的重复训练就是在这条路径上让孩子反复走?
: 这个狭义的定义我基本同意,而且觉得相当形象。
: 但是总觉得和广义的重复训练有所不同,因为我们常说举一反三,触类旁通,也许就是
: 引导娃在走过很多次起点和终点都确定的决策路径之后,有没有其他引向non-leaf

t*******r
发帖数: 22634
86
个人看法,这个首先还不是 open-ended vs close-ended problem,首先是 deduction
vs induction。
或者说,“按部就班” 的本质,是更侧重于 deduction。而 “乱中取胜” / “在与
蒋军的斗争中学习” 的本质,是更侧重于 induction。个人看法。

node。

【在 Y********d 的大作中提到】
:
: 是
: decision
: 读你的帖子一定要google在手啊,我还要特地去学习一下decision graph和leaf node。
: 如果我理解正确的话,你这个帖子算是正式定义了repetitive training,这个事情我
: 也一直想问你但是又忘了问的,所以在你的定义里, repetitive training就是起点和
: 终点都确定的决策路径,而学校正规的重复训练就是在这条路径上让孩子反复走?
: 这个狭义的定义我基本同意,而且觉得相当形象。
: 但是总觉得和广义的重复训练有所不同,因为我们常说举一反三,触类旁通,也许就是
: 引导娃在走过很多次起点和终点都确定的决策路径之后,有没有其他引向non-leaf

t*******r
发帖数: 22634
87
根据大脑灰质剪枝理论,我觉得也不能排除过多的 repetitive routine work 会
decrease neuroplasticity,也就是说,我觉得不能排除 acquired 因子。

【在 s***n 的大作中提到】
: 先推荐一本书:NurtureShock - New Thinking About Children。里面分十个主题介绍
: 了近十几年教育学研究和心理学研究的进展。很多研究进展都有别于大家早前的认识,
: 所以说shock。比如这本书就提到那些在Kindergarten就考上GT的孩子只有29%能通过三
: 年级的GT考试。而那些初中的GT生很多在高中还是GT。换言之,在K阶段起跑赢了的孩
: 子很多一进到途中跑阶段就落后了。我想很多教育政策制定者可能知道这个结论 - 我
: 附近几个County的GT筛选在小初高阶段各有一次,晚慧娃有机会翻身。
: 记忆力和逻辑思维能力成熟的年龄是我根据个人观察 + 书本知识 + Wiki + 网上讨论
: 有个大致的猜测,很不科学和严谨。比如记忆力,细分的话,主要有长期记忆力和短期
: 记忆力,前者学龄后基本没什么变化,后者学龄后可以线性发展到青少年期。在记忆力
: 方面有天赋的孩子在学龄前会比较惹眼,比如那些背诗认字小神童。但这些孩子的理解

Y********d
发帖数: 1478
88

赞有料有趣!
找了一下那本Nurture Shock - New Thinking About Children,在amazon上和我看的
那本the whole brain child是一起推荐的。要不你抽空写个读书笔记,参加版上的【
看书养娃】活动?
我很期待的 :-)

【在 s***n 的大作中提到】
: 先推荐一本书:NurtureShock - New Thinking About Children。里面分十个主题介绍
: 了近十几年教育学研究和心理学研究的进展。很多研究进展都有别于大家早前的认识,
: 所以说shock。比如这本书就提到那些在Kindergarten就考上GT的孩子只有29%能通过三
: 年级的GT考试。而那些初中的GT生很多在高中还是GT。换言之,在K阶段起跑赢了的孩
: 子很多一进到途中跑阶段就落后了。我想很多教育政策制定者可能知道这个结论 - 我
: 附近几个County的GT筛选在小初高阶段各有一次,晚慧娃有机会翻身。
: 记忆力和逻辑思维能力成熟的年龄是我根据个人观察 + 书本知识 + Wiki + 网上讨论
: 有个大致的猜测,很不科学和严谨。比如记忆力,细分的话,主要有长期记忆力和短期
: 记忆力,前者学龄后基本没什么变化,后者学龄后可以线性发展到青少年期。在记忆力
: 方面有天赋的孩子在学龄前会比较惹眼,比如那些背诗认字小神童。但这些孩子的理解

d**********h
发帖数: 2795
89
多谢解读,潮水兄的确需要一个小秘 :)
中国老话说:书读百遍其意自现。这是一个规律,凡是有点启发性教育的引导,哪怕是
一点点,不创新都难。到了最后,严格因循老路反而困难了。
比如练字,颜筋柳骨的五年十年下来,怎么都会有点自我特色。
一个事情千万遍重复之后,总会偷个懒取个巧之类的。
举一反三触类旁通则是褒义描述

node。

【在 Y********d 的大作中提到】
:
: 赞有料有趣!
: 找了一下那本Nurture Shock - New Thinking About Children,在amazon上和我看的
: 那本the whole brain child是一起推荐的。要不你抽空写个读书笔记,参加版上的【
: 看书养娃】活动?
: 我很期待的 :-)

t*******r
发帖数: 22634
90
但分母更多吧,比如范进?

【在 d**********h 的大作中提到】
: 多谢解读,潮水兄的确需要一个小秘 :)
: 中国老话说:书读百遍其意自现。这是一个规律,凡是有点启发性教育的引导,哪怕是
: 一点点,不创新都难。到了最后,严格因循老路反而困难了。
: 比如练字,颜筋柳骨的五年十年下来,怎么都会有点自我特色。
: 一个事情千万遍重复之后,总会偷个懒取个巧之类的。
: 举一反三触类旁通则是褒义描述
:
: node。

相关主题
[华盛顿邮报] 9 - 14 岁是数学兴趣的关键时期?数学教育 一家之言 番外篇
怎么推小学数学好?小学一年级的孩子。小学升初中,儿子数学跳级的一点经验
请问,AMC 8应该从几年级开始练习呀?有明天考AMC 8的吗?
进入Parenting版参与讨论
d**********h
发帖数: 2795
91
赞总结,我也需要消化一下,和我们家娃印证一下 :)

【在 s***n 的大作中提到】
: 先推荐一本书:NurtureShock - New Thinking About Children。里面分十个主题介绍
: 了近十几年教育学研究和心理学研究的进展。很多研究进展都有别于大家早前的认识,
: 所以说shock。比如这本书就提到那些在Kindergarten就考上GT的孩子只有29%能通过三
: 年级的GT考试。而那些初中的GT生很多在高中还是GT。换言之,在K阶段起跑赢了的孩
: 子很多一进到途中跑阶段就落后了。我想很多教育政策制定者可能知道这个结论 - 我
: 附近几个County的GT筛选在小初高阶段各有一次,晚慧娃有机会翻身。
: 记忆力和逻辑思维能力成熟的年龄是我根据个人观察 + 书本知识 + Wiki + 网上讨论
: 有个大致的猜测,很不科学和严谨。比如记忆力,细分的话,主要有长期记忆力和短期
: 记忆力,前者学龄后基本没什么变化,后者学龄后可以线性发展到青少年期。在记忆力
: 方面有天赋的孩子在学龄前会比较惹眼,比如那些背诗认字小神童。但这些孩子的理解

d**********h
发帖数: 2795
92
看你如何定义分母了,无论三角形还是钟形函数,巅峰的总是少数,无论主观如何努力
,大数原理是不变的。
人才分流至少让曲线(分母)不是太难看罢了

【在 t*******r 的大作中提到】
: 但分母更多吧,比如范进?
Y********d
发帖数: 1478
93

deduction
让我想起有一次和芸芸姐姐私信,讨论思维方式的问题。当时网上有一道数学题,你和
大伙都去做了,我的评论如下:
“我的思路会和XXX差不多,从具体到抽象,或者说归纳,但是潮水可以直接抽象,或
者说演绎。我其实欣赏的是他这种思考的习惯。你看他在网上有时自说自话的一段又一
段,不管别人听不听得懂,我就觉得特别有意思。”
可你这会儿说的,
“按部就班” 的本质,是更侧重于 deduction。而 “乱中取胜” / “在与蒋军的斗
争中学习” 的本质,是更侧重于 induction。
让我完全lost了!

【在 t*******r 的大作中提到】
: 个人看法,这个首先还不是 open-ended vs close-ended problem,首先是 deduction
: vs induction。
: 或者说,“按部就班” 的本质,是更侧重于 deduction。而 “乱中取胜” / “在与
: 蒋军的斗争中学习” 的本质,是更侧重于 induction。个人看法。
:
: node。

t*******r
发帖数: 22634
94
我觉得还存在一个方向,就是利用 beyond formal 来 guide,或者说 induction 来
guide。
beyond formal / induction 的一个例子:
离散问题通常想到用 decision tree 建模,是因为 natural numbers 实际上是反映
counting,折射的是我们这个 cosmos 的 causality property。
而连续问题通常想到用 number line / analytic geometry 建模,是因为 real
numbers 实际上是反映 measuring,折射的是我们这个 cosmos 的 spatial property。
上面这两个,很难说是 formal operation,也不能实际用来解题,但其的确提供一种
隐约的 guidance。我个人感觉是属于 beyond formal。

node。

【在 Y********d 的大作中提到】
:
: deduction
: 让我想起有一次和芸芸姐姐私信,讨论思维方式的问题。当时网上有一道数学题,你和
: 大伙都去做了,我的评论如下:
: “我的思路会和XXX差不多,从具体到抽象,或者说归纳,但是潮水可以直接抽象,或
: 者说演绎。我其实欣赏的是他这种思考的习惯。你看他在网上有时自说自话的一段又一
: 段,不管别人听不听得懂,我就觉得特别有意思。”
: 可你这会儿说的,
: “按部就班” 的本质,是更侧重于 deduction。而 “乱中取胜” / “在与蒋军的斗
: 争中学习” 的本质,是更侧重于 induction。

t*******r
发帖数: 22634
95
“乱中取胜” 试着解 AMC8 第 15 到 第 25 题,需要试着解从来没见过没教过的题。
所以需要 induction。

【在 Y********d 的大作中提到】
:
: deduction
: 让我想起有一次和芸芸姐姐私信,讨论思维方式的问题。当时网上有一道数学题,你和
: 大伙都去做了,我的评论如下:
: “我的思路会和XXX差不多,从具体到抽象,或者说归纳,但是潮水可以直接抽象,或
: 者说演绎。我其实欣赏的是他这种思考的习惯。你看他在网上有时自说自话的一段又一
: 段,不管别人听不听得懂,我就觉得特别有意思。”
: 可你这会儿说的,
: “按部就班” 的本质,是更侧重于 deduction。而 “乱中取胜” / “在与蒋军的斗
: 争中学习” 的本质,是更侧重于 induction。

d**********h
发帖数: 2795
96
嗯,这个理论,我的浅显理解是我妈经常说的:大脑越用越聪明
其实也reasonable,大脑的一个部分和其他部分比,如果经常锻炼,得到更多氧气,养
料,肯定会“肥大”,如我们锻炼肌肉。这有点拉马克的用进废退学说的意思。
按照弗洛伊德的说法就是,心理能在此区域做的心理工比别处多,这一块所掌管的记忆
,性格,反应速度等就比较深刻敏感。

【在 Y********d 的大作中提到】
:
: deduction
: 让我想起有一次和芸芸姐姐私信,讨论思维方式的问题。当时网上有一道数学题,你和
: 大伙都去做了,我的评论如下:
: “我的思路会和XXX差不多,从具体到抽象,或者说归纳,但是潮水可以直接抽象,或
: 者说演绎。我其实欣赏的是他这种思考的习惯。你看他在网上有时自说自话的一段又一
: 段,不管别人听不听得懂,我就觉得特别有意思。”
: 可你这会儿说的,
: “按部就班” 的本质,是更侧重于 deduction。而 “乱中取胜” / “在与蒋军的斗
: 争中学习” 的本质,是更侧重于 induction。

Y********d
发帖数: 1478
97
小秘要吃饭去了,这厢先别过了:-)
看来读书笔记是一定要写了,能够钓出一条弗洛伊德大鱼也不错!

【在 d**********h 的大作中提到】
: 嗯,这个理论,我的浅显理解是我妈经常说的:大脑越用越聪明
: 其实也reasonable,大脑的一个部分和其他部分比,如果经常锻炼,得到更多氧气,养
: 料,肯定会“肥大”,如我们锻炼肌肉。这有点拉马克的用进废退学说的意思。
: 按照弗洛伊德的说法就是,心理能在此区域做的心理工比别处多,这一块所掌管的记忆
: ,性格,反应速度等就比较深刻敏感。

d**********h
发帖数: 2795
98
我觉得潮水兄的解释是这样的:
前面的按部就班是因为已经有了冰箱贴,只要按照预定方案,对每个题中的特色稍作调
整(演绎deduction),可解
对后面的,基本no idea的新题型,无法简单归类到前面的任何一个冰箱贴 就只能期望
乱中取胜,是期望。对于新题型,deduction不成了,只好寄希望于归纳induction。
这有点像认识论里的分析和综合。

【在 Y********d 的大作中提到】
: 小秘要吃饭去了,这厢先别过了:-)
: 看来读书笔记是一定要写了,能够钓出一条弗洛伊德大鱼也不错!

t*******r
发帖数: 22634
99
赞同这个看法。

【在 d**********h 的大作中提到】
: 我觉得潮水兄的解释是这样的:
: 前面的按部就班是因为已经有了冰箱贴,只要按照预定方案,对每个题中的特色稍作调
: 整(演绎deduction),可解
: 对后面的,基本no idea的新题型,无法简单归类到前面的任何一个冰箱贴 就只能期望
: 乱中取胜,是期望。对于新题型,deduction不成了,只好寄希望于归纳induction。
: 这有点像认识论里的分析和综合。

s***n
发帖数: 1280
100
同意。尤其在孩子大脑发育时。所以某些时候跳出学校的框框是必要的。因材施教吗。

【在 d**********h 的大作中提到】
: 嗯,这个理论,我的浅显理解是我妈经常说的:大脑越用越聪明
: 其实也reasonable,大脑的一个部分和其他部分比,如果经常锻炼,得到更多氧气,养
: 料,肯定会“肥大”,如我们锻炼肌肉。这有点拉马克的用进废退学说的意思。
: 按照弗洛伊德的说法就是,心理能在此区域做的心理工比别处多,这一块所掌管的记忆
: ,性格,反应速度等就比较深刻敏感。

相关主题
求科普: 奥数 vs. Math Olympiad vs. Math Count vs. Math circle vs. 超前学数学[葩论] 论“有限小灶策略”
9岁, 如何报名考amc10给初中学生家长的一些建议 (转载)
算不算有数学天分小学数学上超前班有什么好处吗?
进入Parenting版参与讨论
s***n
发帖数: 1280
101
这是做到什么程度的问题,不是做不做的问题了。
Repetitive routine work是对基础概念的hand-on experience。它对基础概念的形成
和巩固是很有帮助的。 美国的小学数学教育不太重视Repetitive routine work,使很
多孩子在抽象思维方面的基础训练不够,影响了以后在数学方面的发展。
这就好比瞎子摸象。一个瞎子是自己花了点时间把象摸了个遍,一个瞎子是听老师把象
的样子讲了一遍,我觉得前者脑子里的印"象“会更深些。如果把两个认知手段结合起
来,那瞎子对象的认识就比较完全和巩固了。
我不知道你说的acquired 因子指什么方面。Repetitive routine work是acquire知识
的手段,不是用来acquire能力的。Repetitive non-routine work 可以acquire能力。

【在 t*******r 的大作中提到】
: 根据大脑灰质剪枝理论,我觉得也不能排除过多的 repetitive routine work 会
: decrease neuroplasticity,也就是说,我觉得不能排除 acquired 因子。

Y********d
发帖数: 1478
102

感觉你比我更胜任潮水官方发言人(俗称小秘)一职,我主动撤销job application。
下次你们发言,我在下面拍照提问鼓掌好了:-)

【在 d**********h 的大作中提到】
: 我觉得潮水兄的解释是这样的:
: 前面的按部就班是因为已经有了冰箱贴,只要按照预定方案,对每个题中的特色稍作调
: 整(演绎deduction),可解
: 对后面的,基本no idea的新题型,无法简单归类到前面的任何一个冰箱贴 就只能期望
: 乱中取胜,是期望。对于新题型,deduction不成了,只好寄希望于归纳induction。
: 这有点像认识论里的分析和综合。

Y********d
发帖数: 1478
103

赞更进一步,形象地深入地定义repetitive training.

【在 s***n 的大作中提到】
: 这是做到什么程度的问题,不是做不做的问题了。
: Repetitive routine work是对基础概念的hand-on experience。它对基础概念的形成
: 和巩固是很有帮助的。 美国的小学数学教育不太重视Repetitive routine work,使很
: 多孩子在抽象思维方面的基础训练不够,影响了以后在数学方面的发展。
: 这就好比瞎子摸象。一个瞎子是自己花了点时间把象摸了个遍,一个瞎子是听老师把象
: 的样子讲了一遍,我觉得前者脑子里的印"象“会更深些。如果把两个认知手段结合起
: 来,那瞎子对象的认识就比较完全和巩固了。
: 我不知道你说的acquired 因子指什么方面。Repetitive routine work是acquire知识
: 的手段,不是用来acquire能力的。Repetitive non-routine work 可以acquire能力。

s***n
发帖数: 1280
104
谢解释。理解潮水的帖子是一种能力 - 很多时候我没有。
我本来以为大英帝国 vs. 美国海军陆战队,按部就班 vs. 乱中取胜是指学习的策略 -
是花时间打牢基础的旧知识点再攻下一个基础知识点呢,还是通过做非基础题快速学
习新知识点和解题技巧。 我的看法是对绝大多数孩子,按部就班是重要的,是根本。
难怕你想乱中取胜,也需要按部就班地建起扎实的infrastructure基础。没有强大的海
外基地(大英帝国)做靠山,再牛逼的海军陆战队也只敢在家旁边横。除非时间非常有限
或为了应试,我会花更多的时间去按部就班地建基地。乱中取胜更适合普通娃应试型学
习和牛蛙早期超前学习。
根据你的解释,潮水的大英帝国 vs. 美国海军陆战队,按部就班 vs. 乱中取胜其实是
指考试做题的策略 - deduction vs. induction。是的,一般而言,容易题用
deduction的步骤多些,难题竞赛题用induction的步骤多些。但大多数时候,解难题两
者都需要:用deduction把起点推前点,用induction把终点拉近点,两者会师了,题就
解出来了。这个时候把两者分裂看说哪个更重要,其实意义不大。不过我觉得对大部分
普通娃和部分牛娃,按部就班的学好基础知识,可以把很多induction的步骤变成
deduction,降低题目的难度。新题型毕竟还得绕着基础知识点,所谓万变不离其宗。
deduction触角长的话,就可以把会师点大幅往终点推,可以把induction难题变成容易
题。有些牛蛙induction能力强。但哪怕是对这些娃,巩固下基础知识,练练deduction
也不是坏事。

【在 d**********h 的大作中提到】
: 我觉得潮水兄的解释是这样的:
: 前面的按部就班是因为已经有了冰箱贴,只要按照预定方案,对每个题中的特色稍作调
: 整(演绎deduction),可解
: 对后面的,基本no idea的新题型,无法简单归类到前面的任何一个冰箱贴 就只能期望
: 乱中取胜,是期望。对于新题型,deduction不成了,只好寄希望于归纳induction。
: 这有点像认识论里的分析和综合。

t*******r
发帖数: 22634
105
我觉得,实践上 repetitive routine work 方面做何种程度,目前没有真正意义上的
scientific study。实践操作而言,我觉得 (1) 能执行下来,(2) 能满足需要,我觉
得就差不多了。每个娃不一样,每家的需求也不一样,这很难给个一刀切的标准。
另外我前面说的 acquire 的意思,是说 neuroplasticity 也可能有 acquired 因子。

【在 s***n 的大作中提到】
: 这是做到什么程度的问题,不是做不做的问题了。
: Repetitive routine work是对基础概念的hand-on experience。它对基础概念的形成
: 和巩固是很有帮助的。 美国的小学数学教育不太重视Repetitive routine work,使很
: 多孩子在抽象思维方面的基础训练不够,影响了以后在数学方面的发展。
: 这就好比瞎子摸象。一个瞎子是自己花了点时间把象摸了个遍,一个瞎子是听老师把象
: 的样子讲了一遍,我觉得前者脑子里的印"象“会更深些。如果把两个认知手段结合起
: 来,那瞎子对象的认识就比较完全和巩固了。
: 我不知道你说的acquired 因子指什么方面。Repetitive routine work是acquire知识
: 的手段,不是用来acquire能力的。Repetitive non-routine work 可以acquire能力。

t*******r
发帖数: 22634
106
对于一场考试而言,我个人认为永远是 deduction 更有效,因为是 (1) 考试的 close
-ended problem 的 nature,(2) 考试是事前给定 limited scope 并且 testable,(3
) 考试是测试 acquired knowledge 而不是 acquiring knowledge。
但另一方面,我觉得 induction 的能力是个 life skill。虽然不能被考试所测试,但
对大部分人 life quality 而言,induction 最少最少也能让 life 更轻松些。
或者另一个角度,induction 是人类几万年来跟 mother nature 混的时候进化出来的
。光 deduction 对 mother nature 不太有效。另外电脑目前还不能 induction。
Arguably,我个人 unscientific 地猜测,induction 也可能是 consciousness 的
basis 之一。

-

【在 s***n 的大作中提到】
: 谢解释。理解潮水的帖子是一种能力 - 很多时候我没有。
: 我本来以为大英帝国 vs. 美国海军陆战队,按部就班 vs. 乱中取胜是指学习的策略 -
: 是花时间打牢基础的旧知识点再攻下一个基础知识点呢,还是通过做非基础题快速学
: 习新知识点和解题技巧。 我的看法是对绝大多数孩子,按部就班是重要的,是根本。
: 难怕你想乱中取胜,也需要按部就班地建起扎实的infrastructure基础。没有强大的海
: 外基地(大英帝国)做靠山,再牛逼的海军陆战队也只敢在家旁边横。除非时间非常有限
: 或为了应试,我会花更多的时间去按部就班地建基地。乱中取胜更适合普通娃应试型学
: 习和牛蛙早期超前学习。
: 根据你的解释,潮水的大英帝国 vs. 美国海军陆战队,按部就班 vs. 乱中取胜其实是
: 指考试做题的策略 - deduction vs. induction。是的,一般而言,容易题用

t*******r
发帖数: 22634
107
或者说,acquired 意思,unscientifically speaking:
(1) Neuroplasticity maybe (more or less) the opposite of strong & stable
neuro pathways.
(2) However, neuroplasticity certainly doesn't mean no neuro pathways. (That
called retarded). Neuroplasticity maybe the outcome of many many related
weaker & less stable neuro pathways, or even form a correlated graph between
those less stable pathways, instead of small number of strong & stable
pathways.
(3) If the above 2 guesswork are true, then it's plausible that for a known
routine environment, neuroplasticity certainly less effective / efficient.
However, when and only when under a constantly changing environment with
level of unpredictables, neuroplasticity has its advantage.
(4) If the above guesswork is also true, then it's plausible that different
environment may have different impact of brain's develope / retain / refine
neuroplasticity. Or in another word, “acquired”.



【在 t*******r 的大作中提到】
: 我觉得,实践上 repetitive routine work 方面做何种程度,目前没有真正意义上的
: scientific study。实践操作而言,我觉得 (1) 能执行下来,(2) 能满足需要,我觉
: 得就差不多了。每个娃不一样,每家的需求也不一样,这很难给个一刀切的标准。
: 另外我前面说的 acquire 的意思,是说 neuroplasticity 也可能有 acquired 因子。

t*******r
发帖数: 22634
108
或者打个群众喜闻乐见的比方,“程控电话交换机” vs “Internet + TCP/IP 数据包
”。
当然,IP 协议阶段还是比较容易丢失数据包,要发展到更高一层的 TCP 协议阶段,才
能满足大多数实用上对 reliability / efficiency 以及 user-friendly 的要求。

That
between
known

【在 t*******r 的大作中提到】
: 或者说,acquired 意思,unscientifically speaking:
: (1) Neuroplasticity maybe (more or less) the opposite of strong & stable
: neuro pathways.
: (2) However, neuroplasticity certainly doesn't mean no neuro pathways. (That
: called retarded). Neuroplasticity maybe the outcome of many many related
: weaker & less stable neuro pathways, or even form a correlated graph between
: those less stable pathways, instead of small number of strong & stable
: pathways.
: (3) If the above 2 guesswork are true, then it's plausible that for a known
: routine environment, neuroplasticity certainly less effective / efficient.

t*******r
发帖数: 22634
109
或者进一步说,neuroplasticity,这看起来是个 quantity outweigh quality 的事儿
,但其本质上是 variety outweigh superiority 的事儿。
如果从这个角度看,repetitive training 的长处是 superiority,也就是说,fast,
precise, effective & efficient。但缺点是,在给定总小时数的情况下,less
variety。

That
between
known

【在 t*******r 的大作中提到】
: 或者说,acquired 意思,unscientifically speaking:
: (1) Neuroplasticity maybe (more or less) the opposite of strong & stable
: neuro pathways.
: (2) However, neuroplasticity certainly doesn't mean no neuro pathways. (That
: called retarded). Neuroplasticity maybe the outcome of many many related
: weaker & less stable neuro pathways, or even form a correlated graph between
: those less stable pathways, instead of small number of strong & stable
: pathways.
: (3) If the above 2 guesswork are true, then it's plausible that for a known
: routine environment, neuroplasticity certainly less effective / efficient.

d**********h
发帖数: 2795
110
翻译出来就是:样样通样样松vs。一招鲜吃遍天 :)
好读书不求甚解 vs。韦编三绝
这个谈论是不会有切实可行的指导意义的,最终无非是加权平均
而且效果无参考比较

,

【在 t*******r 的大作中提到】
: 或者进一步说,neuroplasticity,这看起来是个 quantity outweigh quality 的事儿
: ,但其本质上是 variety outweigh superiority 的事儿。
: 如果从这个角度看,repetitive training 的长处是 superiority,也就是说,fast,
: precise, effective & efficient。但缺点是,在给定总小时数的情况下,less
: variety。
:
: That
: between
: known

相关主题
小学数学上超前班有什么好处吗?学校早晚要教的东西,早早学会了又如何?
小学数学有比赛项目吗AMC 8 成绩 2016
请教:怎样说服孩儿他爸多关心孩子的学习?请教小孩数学教育问题
进入Parenting版参与讨论
d*******e
发帖数: 863
111
请问一下这个 MOEMS 的书在哪儿买的?有兴趣买一本儿看看。
还有这个主题信息量挺大的,但是没有提 MathCounts。有人有经验对比一下这个和
AMC8 以及 MOEMS 的吗?主要是好像 MathCounts 网上有免费的题库,免费的啊!

MOEMS

【在 l*****8 的大作中提到】
: 相对AMC, MOEMS难多了。AMC主要是覆盖的内容多。其实就如潮水给起的名字一样,基
: 本都是计算题。
: 娃的学校12月要让他们去参加一个数学竞赛。我看了一下历年的样题,基本都是计算题
: (比AMC更加"计算”),所谓难的地方就是用文字绕人。LP和娃都抱怨我给她做的MOEMS
: 太难了,不符合考情。我正在纠结是否继续我的MOEMS大计还是屈从于考试,让她多做
: 计算题。

t*******r
发帖数: 22634
112
“最终无非是加权平均”
-- 这就是各家自己的冰箱贴。
“而且效果无参考比较”
-- 不能 repetitive training 的 non-routine problem 都这
德行。教育自家娃是一个没法 repetitive training 的事儿,
当然,如果能生同卵 100+ 胞胎,另说。。。

【在 d**********h 的大作中提到】
: 翻译出来就是:样样通样样松vs。一招鲜吃遍天 :)
: 好读书不求甚解 vs。韦编三绝
: 这个谈论是不会有切实可行的指导意义的,最终无非是加权平均
: 而且效果无参考比较
:
: ,

t*******r
发帖数: 22634
113
MathCounts 是不是初中水平的?

【在 d*******e 的大作中提到】
: 请问一下这个 MOEMS 的书在哪儿买的?有兴趣买一本儿看看。
: 还有这个主题信息量挺大的,但是没有提 MathCounts。有人有经验对比一下这个和
: AMC8 以及 MOEMS 的吗?主要是好像 MathCounts 网上有免费的题库,免费的啊!
:
: MOEMS

t*******r
发帖数: 22634
114
瞎子摸象是个 close-ended problem,如果 open-end it 的话,比如装一个下面这玩
意儿,那就可以直接看美女,不用摸。
t*******r
发帖数: 22634
115
其实我觉得这个还是有可行的指导意义的,但这个变成反过来的,变成了战术的
可行性来决定战略目标。
或者这么说,在目测将来进入州大不困难的前提下,对于超过学校加课后班要求
的无论是 repetitive training 还是 千里跃进大别山training 还是
whatsoever training,本质上已经是属于 “锦上添花” 而不是 “雪中送炭”
的范畴。
从上面这个角度看,是 “另起炉灶” 还是 “有限小灶”,本质上,是取决于
愿意派多少个师。而愿意派多少个师,是政治家之间在国会的妥协的产物,
而不是前线军事指挥官的决定。
或者这么说,李将军手上只有七个师,他就只能搞七个师能搞的磁性战术。
而不能像虎肉一样搞上四百个师,架人梯都能跨过海峡、踏平高雄。

【在 d**********h 的大作中提到】
: 翻译出来就是:样样通样样松vs。一招鲜吃遍天 :)
: 好读书不求甚解 vs。韦编三绝
: 这个谈论是不会有切实可行的指导意义的,最终无非是加权平均
: 而且效果无参考比较
:
: ,

d**********h
发帖数: 2795
116
潮水兄,就过嘴瘾
刘帅当年大吼:两军相逢勇者胜。又曰“摸摸xxx(此处省去十字)”。哪像您现在。。。
战略战术都重要,但是战略更重要,所谓主帅无能累死三军。
当前状况是:父母无能累死孩子
太祖说,战略上要藐视敌人,战术上要重视敌人
当前状况:战略上坚信人人可以藤校,战术上要拼了老命推娃。

【在 t*******r 的大作中提到】
: 其实我觉得这个还是有可行的指导意义的,但这个变成反过来的,变成了战术的
: 可行性来决定战略目标。
: 或者这么说,在目测将来进入州大不困难的前提下,对于超过学校加课后班要求
: 的无论是 repetitive training 还是 千里跃进大别山training 还是
: whatsoever training,本质上已经是属于 “锦上添花” 而不是 “雪中送炭”
: 的范畴。
: 从上面这个角度看,是 “另起炉灶” 还是 “有限小灶”,本质上,是取决于
: 愿意派多少个师。而愿意派多少个师,是政治家之间在国会的妥协的产物,
: 而不是前线军事指挥官的决定。
: 或者这么说,李将军手上只有七个师,他就只能搞七个师能搞的磁性战术。

t*******r
发帖数: 22634
117
七个师的事儿,是在于其他的师可以干别的。我家大娃实在闲得无聊,遂同意自学点音
乐,至少可以打发点无聊的时光。
当然,对于反正不来支票的兴趣爱好,“乱中取胜”战术的长期费效比,是不是更值得
t*******r
发帖数: 22634
118
但杜鲁门就只给七个师,咋整。麦克阿瑟不服的后果大伙儿都看见了不是?就只能
fade away 了。其实也就是 decay 的漂亮说法。

【在 d**********h 的大作中提到】
: 潮水兄,就过嘴瘾
: 刘帅当年大吼:两军相逢勇者胜。又曰“摸摸xxx(此处省去十字)”。哪像您现在。。。
: 战略战术都重要,但是战略更重要,所谓主帅无能累死三军。
: 当前状况是:父母无能累死孩子
: 太祖说,战略上要藐视敌人,战术上要重视敌人
: 当前状况:战略上坚信人人可以藤校,战术上要拼了老命推娃。

d**********h
发帖数: 2795
119
这个说的点子上了。
在座诸位,谁家娃以后会为生计发愁,nobody。
大家现在讨论个啥?不就是不浪费资源,最大化效果,当然没有手里四百个师只用七个
的道理。
克劳塞维兹说:战略决战的时候,一切兵力都要用上,不要浪费。拿破仑滑铁卢的时候
,那个傻X元帅带着三分之一的兵力在梦游。。。
潮帅,前车之鉴啊

【在 t*******r 的大作中提到】
: 七个师的事儿,是在于其他的师可以干别的。我家大娃实在闲得无聊,遂同意自学点音
: 乐,至少可以打发点无聊的时光。
: 当然,对于反正不来支票的兴趣爱好,“乱中取胜”战术的长期费效比,是不是更值得
: ?

d**********h
发帖数: 2795
120
你娃抵几个师,没人知道。
香料越是碾压越是芬芳,代价是粉身碎骨
所以大家找个平均,所谓的主观心理价位,既芬芳又不扭曲自己
又是一个不可定量评估的东西罢了

【在 t*******r 的大作中提到】
: 但杜鲁门就只给七个师,咋整。麦克阿瑟不服的后果大伙儿都看见了不是?就只能
: fade away 了。其实也就是 decay 的漂亮说法。

相关主题
小学数学书推荐?怎么推小学数学好?小学一年级的孩子。
我来说一下天才的鉴定方法请问,AMC 8应该从几年级开始练习呀?
[华盛顿邮报] 9 - 14 岁是数学兴趣的关键时期?数学教育 一家之言 番外篇
进入Parenting版参与讨论
t*******r
发帖数: 22634
121
但孙子也说了,不战而屈人之兵,乃上上策。
当然,后果就是成为孙子而不是老子。

【在 d**********h 的大作中提到】
: 这个说的点子上了。
: 在座诸位,谁家娃以后会为生计发愁,nobody。
: 大家现在讨论个啥?不就是不浪费资源,最大化效果,当然没有手里四百个师只用七个
: 的道理。
: 克劳塞维兹说:战略决战的时候,一切兵力都要用上,不要浪费。拿破仑滑铁卢的时候
: ,那个傻X元帅带着三分之一的兵力在梦游。。。
: 潮帅,前车之鉴啊

t*******r
发帖数: 22634
122
美帝二战建了多少个师?但战后立即大规模复员。这不矛盾。

【在 d**********h 的大作中提到】
: 这个说的点子上了。
: 在座诸位,谁家娃以后会为生计发愁,nobody。
: 大家现在讨论个啥?不就是不浪费资源,最大化效果,当然没有手里四百个师只用七个
: 的道理。
: 克劳塞维兹说:战略决战的时候,一切兵力都要用上,不要浪费。拿破仑滑铁卢的时候
: ,那个傻X元帅带着三分之一的兵力在梦游。。。
: 潮帅,前车之鉴啊

d**********h
发帖数: 2795
123
可以不战而屈人之兵的战略家,古今中外有几个,大部分是外强中干的天机老人罢了
潮水兄,真要有这个层次的战略家,前知五百年,后知五百载,娃就真不愁了
不过,您的计划是符合您自己战略目标的,good!

【在 t*******r 的大作中提到】
: 但孙子也说了,不战而屈人之兵,乃上上策。
: 当然,后果就是成为孙子而不是老子。

s***n
发帖数: 1280
124
MOEMS的书Amazon就有。
不象AMC 8和MOEMS允许小学生参加考试,MathCounts只能6-8年级参加。这里参加讨论
的几个家长,娃最大也就刚入六年级吧,所以大家这方面的讨论少点。
就影响力来说,MathCounts应该算是最有影响的初中数学竞赛 - 总统会接见冠军队和
个人赛前三,还有些组织赞助scholarship。AMC 8算是比较官方的数学竞赛。基本上初
中竞赛娃,这两个赛是一定参加的。MOEMS的影响力比这两个小。
MathCounts是以团体赛为主。一个学校出一只4个学生组成的校队参加团体赛和个人赛
,另外6个学生只参加个人赛。各个队先参加Chapter地区比赛,出线的校队和个人参加
州赛。州赛个人前四名组成州队参加全国赛。
MathCounts题如其名,侧重速算和数感。难度和AMC 8差不多,但题目都是填空题。比
赛形式多样些,有两轮甚至可以用计算器。州赛和全国赛有抢答部分,有些州甚至用抢
答部分定最后名次。

【在 d*******e 的大作中提到】
: 请问一下这个 MOEMS 的书在哪儿买的?有兴趣买一本儿看看。
: 还有这个主题信息量挺大的,但是没有提 MathCounts。有人有经验对比一下这个和
: AMC8 以及 MOEMS 的吗?主要是好像 MathCounts 网上有免费的题库,免费的啊!
:
: MOEMS

d**********h
发帖数: 2795
125
赞科普

【在 s***n 的大作中提到】
: MOEMS的书Amazon就有。
: 不象AMC 8和MOEMS允许小学生参加考试,MathCounts只能6-8年级参加。这里参加讨论
: 的几个家长,娃最大也就刚入六年级吧,所以大家这方面的讨论少点。
: 就影响力来说,MathCounts应该算是最有影响的初中数学竞赛 - 总统会接见冠军队和
: 个人赛前三,还有些组织赞助scholarship。AMC 8算是比较官方的数学竞赛。基本上初
: 中竞赛娃,这两个赛是一定参加的。MOEMS的影响力比这两个小。
: MathCounts是以团体赛为主。一个学校出一只4个学生组成的校队参加团体赛和个人赛
: ,另外6个学生只参加个人赛。各个队先参加Chapter地区比赛,出线的校队和个人参加
: 州赛。州赛个人前四名组成州队参加全国赛。
: MathCounts题如其名,侧重速算和数感。难度和AMC 8差不多,但题目都是填空题。比

P******e
发帖数: 1325
126
赞信息量。以前哪个id说过,拿到mathcounts在他们那个城市的前几名,就能保送mit
了。

★ 发自iPhone App: ChineseWeb 1.0.2
★ 发自iPhone App: ChineseWeb 1.0.2

【在 s***n 的大作中提到】
: MOEMS的书Amazon就有。
: 不象AMC 8和MOEMS允许小学生参加考试,MathCounts只能6-8年级参加。这里参加讨论
: 的几个家长,娃最大也就刚入六年级吧,所以大家这方面的讨论少点。
: 就影响力来说,MathCounts应该算是最有影响的初中数学竞赛 - 总统会接见冠军队和
: 个人赛前三,还有些组织赞助scholarship。AMC 8算是比较官方的数学竞赛。基本上初
: 中竞赛娃,这两个赛是一定参加的。MOEMS的影响力比这两个小。
: MathCounts是以团体赛为主。一个学校出一只4个学生组成的校队参加团体赛和个人赛
: ,另外6个学生只参加个人赛。各个队先参加Chapter地区比赛,出线的校队和个人参加
: 州赛。州赛个人前四名组成州队参加全国赛。
: MathCounts题如其名,侧重速算和数感。难度和AMC 8差不多,但题目都是填空题。比

t*******r
发帖数: 22634
127
其实我不是真要搞 music,因为这玩意儿扰民不算,过程都还听得见,徒然增加
expectation 的 burden。以前试过都不行,我对此基本绝望。
但我想了个办法,我要搞的就好比是 silent music,不扰民。虽然对 BSO 都没用,对
藤校更是无用,但自个儿开心能 kill time 最实在。
其实我自己搞的也好比是 silent music,基本不扰自家人和自家附近的民。

【在 d**********h 的大作中提到】
: 可以不战而屈人之兵的战略家,古今中外有几个,大部分是外强中干的天机老人罢了
: 潮水兄,真要有这个层次的战略家,前知五百年,后知五百载,娃就真不愁了
: 不过,您的计划是符合您自己战略目标的,good!

t*******r
发帖数: 22634
128
反正俺已经铁了心不搞数学竞赛啦。
我看你们跳坑,为你们鼓掌喝彩!

mit

【在 P******e 的大作中提到】
: 赞信息量。以前哪个id说过,拿到mathcounts在他们那个城市的前几名,就能保送mit
: 了。
:
: ★ 发自iPhone App: ChineseWeb 1.0.2
: ★ 发自iPhone App: ChineseWeb 1.0.2

s***n
发帖数: 1280
129
不太可能吧。我看过大学申请信息交流的网站。一些人讨论初中获的奖怎么写到大学申
请表中。讨论结果就是就算你拿了MathCounts全国冠军,你在大学申请表上也找不到地
方填这个信息 - 人家只问高中拿的奖。你只有想办法把这个奖写到Essay里去,录取官
才能看到。
另一方面如果申请者只在初中数学竞赛拿了大奖,高中数学竞赛却没什么收获的话,会
让录取官觉得娃退步了,反而降低录取几率。
但另另一方面,很多mathcounts取得好成绩的孩子以后高中数学竞赛成绩也不错。那些
当年参加Mathcounts全国赛的孩子很多都出现在USAMO。
所以某个意义上,那个id说的也可能是实话:确实有某个孩子mathcounts得了Chapter
前几名,他也确实以后被保送到了MIT,但缺省掉的信息可能是那个孩子在高中阶段拿
了更多更大的奖。他是凭着那些奖被报送的,而不是初中时的地区小奖。

mit

【在 P******e 的大作中提到】
: 赞信息量。以前哪个id说过,拿到mathcounts在他们那个城市的前几名,就能保送mit
: 了。
:
: ★ 发自iPhone App: ChineseWeb 1.0.2
: ★ 发自iPhone App: ChineseWeb 1.0.2

t*******r
发帖数: 22634
130
分析得很有道理!

Chapter

【在 s***n 的大作中提到】
: 不太可能吧。我看过大学申请信息交流的网站。一些人讨论初中获的奖怎么写到大学申
: 请表中。讨论结果就是就算你拿了MathCounts全国冠军,你在大学申请表上也找不到地
: 方填这个信息 - 人家只问高中拿的奖。你只有想办法把这个奖写到Essay里去,录取官
: 才能看到。
: 另一方面如果申请者只在初中数学竞赛拿了大奖,高中数学竞赛却没什么收获的话,会
: 让录取官觉得娃退步了,反而降低录取几率。
: 但另另一方面,很多mathcounts取得好成绩的孩子以后高中数学竞赛成绩也不错。那些
: 当年参加Mathcounts全国赛的孩子很多都出现在USAMO。
: 所以某个意义上,那个id说的也可能是实话:确实有某个孩子mathcounts得了Chapter
: 前几名,他也确实以后被保送到了MIT,但缺省掉的信息可能是那个孩子在高中阶段拿

相关主题
小学升初中,儿子数学跳级的一点经验9岁, 如何报名考amc10
有明天考AMC 8的吗?算不算有数学天分
求科普: 奥数 vs. Math Olympiad vs. Math Count vs. Math circle vs. 超前学数学[葩论] 论“有限小灶策略”
进入Parenting版参与讨论
Y********d
发帖数: 1478
131
根据事实1,假设2,条件3,最后推导出结论4。
赞deduction的模版示范!

Chapter

【在 s***n 的大作中提到】
: 不太可能吧。我看过大学申请信息交流的网站。一些人讨论初中获的奖怎么写到大学申
: 请表中。讨论结果就是就算你拿了MathCounts全国冠军,你在大学申请表上也找不到地
: 方填这个信息 - 人家只问高中拿的奖。你只有想办法把这个奖写到Essay里去,录取官
: 才能看到。
: 另一方面如果申请者只在初中数学竞赛拿了大奖,高中数学竞赛却没什么收获的话,会
: 让录取官觉得娃退步了,反而降低录取几率。
: 但另另一方面,很多mathcounts取得好成绩的孩子以后高中数学竞赛成绩也不错。那些
: 当年参加Mathcounts全国赛的孩子很多都出现在USAMO。
: 所以某个意义上,那个id说的也可能是实话:确实有某个孩子mathcounts得了Chapter
: 前几名,他也确实以后被保送到了MIT,但缺省掉的信息可能是那个孩子在高中阶段拿

s***n
发帖数: 1280
132
事情的经过是这样的:
潮水扛着把锹号召大家去移山,然后在山前挖了个坑,看大家一个个跳进去...

【在 t*******r 的大作中提到】
: 反正俺已经铁了心不搞数学竞赛啦。
: 我看你们跳坑,为你们鼓掌喝彩!
:
: mit

d*******e
发帖数: 863
133
多谢!

【在 s***n 的大作中提到】
: MOEMS的书Amazon就有。
: 不象AMC 8和MOEMS允许小学生参加考试,MathCounts只能6-8年级参加。这里参加讨论
: 的几个家长,娃最大也就刚入六年级吧,所以大家这方面的讨论少点。
: 就影响力来说,MathCounts应该算是最有影响的初中数学竞赛 - 总统会接见冠军队和
: 个人赛前三,还有些组织赞助scholarship。AMC 8算是比较官方的数学竞赛。基本上初
: 中竞赛娃,这两个赛是一定参加的。MOEMS的影响力比这两个小。
: MathCounts是以团体赛为主。一个学校出一只4个学生组成的校队参加团体赛和个人赛
: ,另外6个学生只参加个人赛。各个队先参加Chapter地区比赛,出线的校队和个人参加
: 州赛。州赛个人前四名组成州队参加全国赛。
: MathCounts题如其名,侧重速算和数感。难度和AMC 8差不多,但题目都是填空题。比

J***A
发帖数: 1511
134
赞科普!

MOEMS的书Amazon就有。不象AMC 8和MOEMS允许小学生参加考试,MathCounts只能6-8年
级参加。这里参加讨论的几个家长,娃最大也就刚入六年级吧,所以大家这........

【在 s***n 的大作中提到】
: MOEMS的书Amazon就有。
: 不象AMC 8和MOEMS允许小学生参加考试,MathCounts只能6-8年级参加。这里参加讨论
: 的几个家长,娃最大也就刚入六年级吧,所以大家这方面的讨论少点。
: 就影响力来说,MathCounts应该算是最有影响的初中数学竞赛 - 总统会接见冠军队和
: 个人赛前三,还有些组织赞助scholarship。AMC 8算是比较官方的数学竞赛。基本上初
: 中竞赛娃,这两个赛是一定参加的。MOEMS的影响力比这两个小。
: MathCounts是以团体赛为主。一个学校出一只4个学生组成的校队参加团体赛和个人赛
: ,另外6个学生只参加个人赛。各个队先参加Chapter地区比赛,出线的校队和个人参加
: 州赛。州赛个人前四名组成州队参加全国赛。
: MathCounts题如其名,侧重速算和数感。难度和AMC 8差不多,但题目都是填空题。比

s***n
发帖数: 1280
135
是的。Repetitive routine work要做的什么程度,每个孩子会不太一样,聪明的两三
个例子就知道了,没开窍的可能要十几上百遍。Repetitive routine work做得不够,
孩子会概念不牢;做得太多,孩子会无聊。这个程度需要老师和家长来掌握。问题是很
多公立学校老师喜欢一刀切: 来来来,大家每人做十遍,结果就是聪明的孩子觉得上课
无聊,没开窍的还是没开窍。对于后者,家长课后开小灶可以解决。对于前者,家长也
可以开小灶上难题,让孩子保持学习兴趣,并争取把孩子推到更适合他的快班。但也有
些情况下,没有快班或没有合适的快班,而长期让聪明的孩子在课堂上感到学习无聊,
会打击他们的学习积极性,这种情况下家长就应该考虑另起炉灶。



【在 t*******r 的大作中提到】
: 我觉得,实践上 repetitive routine work 方面做何种程度,目前没有真正意义上的
: scientific study。实践操作而言,我觉得 (1) 能执行下来,(2) 能满足需要,我觉
: 得就差不多了。每个娃不一样,每家的需求也不一样,这很难给个一刀切的标准。
: 另外我前面说的 acquire 的意思,是说 neuroplasticity 也可能有 acquired 因子。

t*******r
发帖数: 22634
136
属实。
当然公立学校老师也不是喜欢一刀切,而是规模产品不可能为每家定制。

【在 s***n 的大作中提到】
: 是的。Repetitive routine work要做的什么程度,每个孩子会不太一样,聪明的两三
: 个例子就知道了,没开窍的可能要十几上百遍。Repetitive routine work做得不够,
: 孩子会概念不牢;做得太多,孩子会无聊。这个程度需要老师和家长来掌握。问题是很
: 多公立学校老师喜欢一刀切: 来来来,大家每人做十遍,结果就是聪明的孩子觉得上课
: 无聊,没开窍的还是没开窍。对于后者,家长课后开小灶可以解决。对于前者,家长也
: 可以开小灶上难题,让孩子保持学习兴趣,并争取把孩子推到更适合他的快班。但也有
: 些情况下,没有快班或没有合适的快班,而长期让聪明的孩子在课堂上感到学习无聊,
: 会打击他们的学习积极性,这种情况下家长就应该考虑另起炉灶。
:
: 的

s***n
发帖数: 1280
137
大致同意你说的。不过因为你此前很多讨论用AMC竞赛为例,所以我说的考试也是指的
竞赛。对于一般考试,你说的我基本赞同。对竞赛来说,实际上有很多基本知识点延伸
出来的知识点和技巧。所以竞赛,特别是难题,很多时候是看你对这些延伸出来的知识
点和技巧的掌握情况。掌握了,这道题就可以多依赖deduction,孩子也会觉得这道题
容易。没掌握,孩子就得多依赖induction了。
我以前看我儿子做难题,并没有想到induction层面上。我的感觉是他的观察力比较弱
,而观察恰恰是induction的第一步,所以我儿子实际上是induction的能力弱。
我也觉得观察力/induction是人智力的一部分,而且有天生的因素。我看身边的孩子,
能看出他们观察力上的差距。小学阶段,观察力/induction能力很少被训练或被测试。
所以我把这种差距归结为天生。

close
(3

【在 t*******r 的大作中提到】
: 对于一场考试而言,我个人认为永远是 deduction 更有效,因为是 (1) 考试的 close
: -ended problem 的 nature,(2) 考试是事前给定 limited scope 并且 testable,(3
: ) 考试是测试 acquired knowledge 而不是 acquiring knowledge。
: 但另一方面,我觉得 induction 的能力是个 life skill。虽然不能被考试所测试,但
: 对大部分人 life quality 而言,induction 最少最少也能让 life 更轻松些。
: 或者另一个角度,induction 是人类几万年来跟 mother nature 混的时候进化出来的
: 。光 deduction 对 mother nature 不太有效。另外电脑目前还不能 induction。
: Arguably,我个人 unscientific 地猜测,induction 也可能是 consciousness 的
: basis 之一。
:

t*******r
发帖数: 22634
138
我觉得 induction 比 deduction 需要更多的时间来 develop / mature,早年不好判
断。
或者 quantify 一下的话,如果 deduction 在初中阶段显露的话,那 induction 可能
要到高中。
另外我觉得 induction 更大的用处是在 science 上,而不是 math。当然小学初中的
science 就好比是 phonics 阶段。
个人看法。

【在 s***n 的大作中提到】
: 大致同意你说的。不过因为你此前很多讨论用AMC竞赛为例,所以我说的考试也是指的
: 竞赛。对于一般考试,你说的我基本赞同。对竞赛来说,实际上有很多基本知识点延伸
: 出来的知识点和技巧。所以竞赛,特别是难题,很多时候是看你对这些延伸出来的知识
: 点和技巧的掌握情况。掌握了,这道题就可以多依赖deduction,孩子也会觉得这道题
: 容易。没掌握,孩子就得多依赖induction了。
: 我以前看我儿子做难题,并没有想到induction层面上。我的感觉是他的观察力比较弱
: ,而观察恰恰是induction的第一步,所以我儿子实际上是induction的能力弱。
: 我也觉得观察力/induction是人智力的一部分,而且有天生的因素。我看身边的孩子,
: 能看出他们观察力上的差距。小学阶段,观察力/induction能力很少被训练或被测试。
: 所以我把这种差距归结为天生。

P******e
发帖数: 1325
139
你娃才小学生吧?已经研究过大学申请信息交流的网站啦?
mit和caltech是对数学竞赛选手最有热情的学校,usamo那点人不够分的,预订了
mathcounts的种子也是有可能的。
有没有mathcounts种子选手的家长来说说看啊?

Chapter

【在 s***n 的大作中提到】
: 不太可能吧。我看过大学申请信息交流的网站。一些人讨论初中获的奖怎么写到大学申
: 请表中。讨论结果就是就算你拿了MathCounts全国冠军,你在大学申请表上也找不到地
: 方填这个信息 - 人家只问高中拿的奖。你只有想办法把这个奖写到Essay里去,录取官
: 才能看到。
: 另一方面如果申请者只在初中数学竞赛拿了大奖,高中数学竞赛却没什么收获的话,会
: 让录取官觉得娃退步了,反而降低录取几率。
: 但另另一方面,很多mathcounts取得好成绩的孩子以后高中数学竞赛成绩也不错。那些
: 当年参加Mathcounts全国赛的孩子很多都出现在USAMO。
: 所以某个意义上,那个id说的也可能是实话:确实有某个孩子mathcounts得了Chapter
: 前几名,他也确实以后被保送到了MIT,但缺省掉的信息可能是那个孩子在高中阶段拿

s***n
发帖数: 1280
140
我儿子刚上初中。不过他数学比较强,去年一直跟着高中竞赛队训。每次他上课,我就
跟其他初中高中竞赛娃家长闲聊,包括我们州MathCounts州队的孩子家长。所以大学申
请,mathcounts。初高中竞赛啥的我都算有点可靠的一二手资料。
MIT和CalTech是对理科竞赛选手感兴趣。据说USAMO的选手90%都能入MIT。不过分完
USAMO后,MIT看的应该是AIME成绩。MIT的申请表上会问申请者的AIME和AMC 12成绩,
但不会问MathCounts成绩。
现在的行情是亚裔理工女娃容易入MIT,能进AIME就有很大希望了(潮水有没有考虑回来
?)。但亚裔理工男娃竞争太激烈,进了USAMO都不能保证。

【在 P******e 的大作中提到】
: 你娃才小学生吧?已经研究过大学申请信息交流的网站啦?
: mit和caltech是对数学竞赛选手最有热情的学校,usamo那点人不够分的,预订了
: mathcounts的种子也是有可能的。
: 有没有mathcounts种子选手的家长来说说看啊?
:
: Chapter

相关主题
[葩论] 论“有限小灶策略”小学数学有比赛项目吗
给初中学生家长的一些建议 (转载)请教:怎样说服孩儿他爸多关心孩子的学习?
小学数学上超前班有什么好处吗?学校早晚要教的东西,早早学会了又如何?
进入Parenting版参与讨论
t*******r
发帖数: 22634
141
MIT 太远,数学竞赛太累。我就看着加州农校,外加教娃农校毕业以后抢个滩。
另外女娃就算容易进 MIT,考试难度还是一样的,外加女娃不可能像男娃那样
花时间(当然,可能比我这种浪荡公子还是多花点时间的)。我娃水平不够的
话,还不如去加州农校。这 STEM 又不是文科,MIT 就算想多些 STEM 女生,
鼓励教授们在大学考试尽可能放水,但那白纸黑字也没法放水不是?万一最后
被迫去读个坑爹型文科,我不是全打水漂?
我看还是加州农校对我等普通女娃更靠谱,上有一批藤校和 UCB 刷下来的
false negatives,下也有混进农校的娃娃们垫底。

【在 s***n 的大作中提到】
: 我儿子刚上初中。不过他数学比较强,去年一直跟着高中竞赛队训。每次他上课,我就
: 跟其他初中高中竞赛娃家长闲聊,包括我们州MathCounts州队的孩子家长。所以大学申
: 请,mathcounts。初高中竞赛啥的我都算有点可靠的一二手资料。
: MIT和CalTech是对理科竞赛选手感兴趣。据说USAMO的选手90%都能入MIT。不过分完
: USAMO后,MIT看的应该是AIME成绩。MIT的申请表上会问申请者的AIME和AMC 12成绩,
: 但不会问MathCounts成绩。
: 现在的行情是亚裔理工女娃容易入MIT,能进AIME就有很大希望了(潮水有没有考虑回来
: ?)。但亚裔理工男娃竞争太激烈,进了USAMO都不能保证。

d**********h
发帖数: 2795
142
唉,我看潮水兄应该从战略角度培养加州农校
待到我娃进农校,农校堪比大藤俏

【在 t*******r 的大作中提到】
: MIT 太远,数学竞赛太累。我就看着加州农校,外加教娃农校毕业以后抢个滩。
: 另外女娃就算容易进 MIT,考试难度还是一样的,外加女娃不可能像男娃那样
: 花时间(当然,可能比我这种浪荡公子还是多花点时间的)。我娃水平不够的
: 话,还不如去加州农校。这 STEM 又不是文科,MIT 就算想多些 STEM 女生,
: 鼓励教授们在大学考试尽可能放水,但那白纸黑字也没法放水不是?万一最后
: 被迫去读个坑爹型文科,我不是全打水漂?
: 我看还是加州农校对我等普通女娃更靠谱,上有一批藤校和 UCB 刷下来的
: false negatives,下也有混进农校的娃娃们垫底。

t*******r
发帖数: 22634
143
其实换个角度,说个娃的故事。这有点像我娃这里的初中分班第一次发回家作业,
有班发 6 年级题、有班发 7 年级题,还有班发 8 年级题。结果三个娃一起做
作业的时候,发到 8 年级题的先做完,闲着没事给发到 6 年级题的答个疑。
旁边发到 7 年级的做了N久终于做完。
还有个例子,我娃小学数学俱乐部的时候,有一个当年州考数学满分(那是不是
computer adaptive,每个班不少满分),看到俱乐部的 invitation 后犹豫
半天。那娃一开始就很老实告诉人她很多题是猜选择枝的(但也会有 invitation
的)。后来知道俱乐部就是玩玩的也就一起去玩。据俺娃谣传,那娃俱乐部的第
一次练习,拿个零蛋看着。不过人娃也有心理准备,哈哈笑笑就完事了。
所以我想想,在未来不可预知的情况下,在美帝这种小康社会,对于我等普通娃,
加州农校虽然不够高大上,但从低风险确保按时到货的角度看,更旱涝保收些。

【在 s***n 的大作中提到】
: 我儿子刚上初中。不过他数学比较强,去年一直跟着高中竞赛队训。每次他上课,我就
: 跟其他初中高中竞赛娃家长闲聊,包括我们州MathCounts州队的孩子家长。所以大学申
: 请,mathcounts。初高中竞赛啥的我都算有点可靠的一二手资料。
: MIT和CalTech是对理科竞赛选手感兴趣。据说USAMO的选手90%都能入MIT。不过分完
: USAMO后,MIT看的应该是AIME成绩。MIT的申请表上会问申请者的AIME和AMC 12成绩,
: 但不会问MathCounts成绩。
: 现在的行情是亚裔理工女娃容易入MIT,能进AIME就有很大希望了(潮水有没有考虑回来
: ?)。但亚裔理工男娃竞争太激烈,进了USAMO都不能保证。

y*******g
发帖数: 1395
144
UC Davis和Stanford的新生高中成绩状况(如图)
垫底的有没有我不知道,反正即便有,也没有你以为的那么多

【在 t*******r 的大作中提到】
: 其实换个角度,说个娃的故事。这有点像我娃这里的初中分班第一次发回家作业,
: 有班发 6 年级题、有班发 7 年级题,还有班发 8 年级题。结果三个娃一起做
: 作业的时候,发到 8 年级题的先做完,闲着没事给发到 6 年级题的答个疑。
: 旁边发到 7 年级的做了N久终于做完。
: 还有个例子,我娃小学数学俱乐部的时候,有一个当年州考数学满分(那是不是
: computer adaptive,每个班不少满分),看到俱乐部的 invitation 后犹豫
: 半天。那娃一开始就很老实告诉人她很多题是猜选择枝的(但也会有 invitation
: 的)。后来知道俱乐部就是玩玩的也就一起去玩。据俺娃谣传,那娃俱乐部的第
: 一次练习,拿个零蛋看着。不过人娃也有心理准备,哈哈笑笑就完事了。
: 所以我想想,在未来不可预知的情况下,在美帝这种小康社会,对于我等普通娃,

t*******r
发帖数: 22634
145
高中 GPA 4.0 只能说明细心不容易出错。
话说我们当年大一不是有那么一波细心娃,但是不适应本科大一突然加快的速度、
深度、以及不再有那么多 repetitive training(相对高中而言),结果第一年
高数和普物期中考试就直接先 decay 一轮当垫底。
当然期末考试老师还是有良心放水让大伙儿过了。
所以对加州农校有垫底娃这点,我很放心。

【在 y*******g 的大作中提到】
: UC Davis和Stanford的新生高中成绩状况(如图)
: 垫底的有没有我不知道,反正即便有,也没有你以为的那么多

y*******g
发帖数: 1395
146
我就是想说,UC Davis内部的都是Stanford的种子,只不过S家的名额不够了,所以被
刷下来而已,其他UC也是如此。所以千万别以为牌子是Davis就以为里面没那么狠的竞
争力了。。。所以以为小孩进去了也没问题了,真相远没那么粗糙。
说完了。

【在 t*******r 的大作中提到】
: 高中 GPA 4.0 只能说明细心不容易出错。
: 话说我们当年大一不是有那么一波细心娃,但是不适应本科大一突然加快的速度、
: 深度、以及不再有那么多 repetitive training(相对高中而言),结果第一年
: 高数和普物期中考试就直接先 decay 一轮当垫底。
: 当然期末考试老师还是有良心放水让大伙儿过了。
: 所以对加州农校有垫底娃这点,我很放心。

t*******r
发帖数: 22634
147
其实藤校也不是没有垫底的,那入学标准也是 false positive 一大堆。
不过藤校有 “进去不分专业,大三才分专业” 的大杀器,外加学生服务处百般说
服(藤校要保证毕业率不是?),导致垫底的都自动去了坑爹专业。所以藤校要留
在非坑爹专业里垫底,那心理素质要不是一般的强大。
加州农校进去先分个学院,然后垫底的也一般就在学院里垫底。不会外逃至坑爹
专业。
另外这里大家有娃的 track of record / National Percentiles,外加学校
里谣言满天飞,大伙儿对相对位置这事儿,还是比较清楚的。

【在 y*******g 的大作中提到】
: 我就是想说,UC Davis内部的都是Stanford的种子,只不过S家的名额不够了,所以被
: 刷下来而已,其他UC也是如此。所以千万别以为牌子是Davis就以为里面没那么狠的竞
: 争力了。。。所以以为小孩进去了也没问题了,真相远没那么粗糙。
: 说完了。

P******e
发帖数: 1325
148
你娃是真厉害的,要记得给大家通报成绩。
潮水一定要认真考虑啊,AIME不难,平均每个州90多个,其中女生也有十几个。
MIT也有相对好混的商科类专业,不做马工,上花街也是不错的。

★ 发自iPhone App: ChineseWeb 1.0.2

【在 s***n 的大作中提到】
: 我儿子刚上初中。不过他数学比较强,去年一直跟着高中竞赛队训。每次他上课,我就
: 跟其他初中高中竞赛娃家长闲聊,包括我们州MathCounts州队的孩子家长。所以大学申
: 请,mathcounts。初高中竞赛啥的我都算有点可靠的一二手资料。
: MIT和CalTech是对理科竞赛选手感兴趣。据说USAMO的选手90%都能入MIT。不过分完
: USAMO后,MIT看的应该是AIME成绩。MIT的申请表上会问申请者的AIME和AMC 12成绩,
: 但不会问MathCounts成绩。
: 现在的行情是亚裔理工女娃容易入MIT,能进AIME就有很大希望了(潮水有没有考虑回来
: ?)。但亚裔理工男娃竞争太激烈,进了USAMO都不能保证。

j*p
发帖数: 780
149
你们参加什么数学竞赛?
现在学校和竞赛队学啥数学?

【在 s***n 的大作中提到】
: 我儿子刚上初中。不过他数学比较强,去年一直跟着高中竞赛队训。每次他上课,我就
: 跟其他初中高中竞赛娃家长闲聊,包括我们州MathCounts州队的孩子家长。所以大学申
: 请,mathcounts。初高中竞赛啥的我都算有点可靠的一二手资料。
: MIT和CalTech是对理科竞赛选手感兴趣。据说USAMO的选手90%都能入MIT。不过分完
: USAMO后,MIT看的应该是AIME成绩。MIT的申请表上会问申请者的AIME和AMC 12成绩,
: 但不会问MathCounts成绩。
: 现在的行情是亚裔理工女娃容易入MIT,能进AIME就有很大希望了(潮水有没有考虑回来
: ?)。但亚裔理工男娃竞争太激烈,进了USAMO都不能保证。

s***n
发帖数: 1280
150
上个学年个人竞赛参加了 MOEMS, AMC 8/10/12,AIME和州大的一个高中竞赛。团体赛
参加了ARML,附近大学的一个高中锦标赛和附近高中的一个初中锦标赛。今年个人赛应
该没什么变化,但团体赛会多三四个锦标赛 + Mathcounts 吧。
我们在烂区GT里面,学校只给上Math 7 honor。好在老师好像比较开通,准备说服老师
让他在课上搞MathCounts。
他去了两个竞赛队。一个是俱乐部性质的,主要针对ARML等各种团体赛搞点专题讲座啥
的;另一个是竞赛班性质,针对AMC/AIME,比较系统的专题讲座和真题讲解。

【在 j*p 的大作中提到】
: 你们参加什么数学竞赛?
: 现在学校和竞赛队学啥数学?

相关主题
AMC 8 成绩 2016我来说一下天才的鉴定方法
请教小孩数学教育问题[华盛顿邮报] 9 - 14 岁是数学兴趣的关键时期?
小学数学书推荐?怎么推小学数学好?小学一年级的孩子。
进入Parenting版参与讨论
t*******r
发帖数: 22634
151
你娃的确牛,你娃这种情况的确要 “另起炉灶”。
你把你娃背景说清楚,就没有前面 “有限小灶” vs “另起炉灶” 的争议了。我觉得
大部分普通娃,实际上更适合搞 “有限小灶” 而不是 “另起炉灶”。
另外学校学区也在与时俱进。所以对于大部分普通娃而言,我个人觉得 “有限小灶”
更适合些吧。因为毕竟普通娃总体而言,还是在学校体制内教育的大框架里的。

【在 s***n 的大作中提到】
: 上个学年个人竞赛参加了 MOEMS, AMC 8/10/12,AIME和州大的一个高中竞赛。团体赛
: 参加了ARML,附近大学的一个高中锦标赛和附近高中的一个初中锦标赛。今年个人赛应
: 该没什么变化,但团体赛会多三四个锦标赛 + Mathcounts 吧。
: 我们在烂区GT里面,学校只给上Math 7 honor。好在老师好像比较开通,准备说服老师
: 让他在课上搞MathCounts。
: 他去了两个竞赛队。一个是俱乐部性质的,主要针对ARML等各种团体赛搞点专题讲座啥
: 的;另一个是竞赛班性质,针对AMC/AIME,比较系统的专题讲座和真题讲解。

t*******r
发帖数: 22634
152
其实我个人希望学区能在整个初中阶段一直保持数学批量成建制的大路货快车道的 3x
speed,并且适当加深一点难度。
因为对于大部分普通娃而言,在初中阶段,是 learning speed and depth make
difference,而不是当前的 knowledge status。
而且我觉得初中也最适合搞这个 “动车提速” 实战试验。因为初中不像高中那样出现
真正记录在案的 GPA,可以试验找到对自家最合适的速度,犯错也不是啥大不了的事儿
。这样到高中时就知根知底,手中有粮、心中不慌。

【在 t*******r 的大作中提到】
: 你娃的确牛,你娃这种情况的确要 “另起炉灶”。
: 你把你娃背景说清楚,就没有前面 “有限小灶” vs “另起炉灶” 的争议了。我觉得
: 大部分普通娃,实际上更适合搞 “有限小灶” 而不是 “另起炉灶”。
: 另外学校学区也在与时俱进。所以对于大部分普通娃而言,我个人觉得 “有限小灶”
: 更适合些吧。因为毕竟普通娃总体而言,还是在学校体制内教育的大框架里的。

P******e
发帖数: 1325
153
这是真牛蛙,上学年才六年级就AIME了,狠赞。
你们用的什么教材来'另起炉灶'的?

★ 发自iPhone App: ChineseWeb 1.0.2

【在 s***n 的大作中提到】
: 上个学年个人竞赛参加了 MOEMS, AMC 8/10/12,AIME和州大的一个高中竞赛。团体赛
: 参加了ARML,附近大学的一个高中锦标赛和附近高中的一个初中锦标赛。今年个人赛应
: 该没什么变化,但团体赛会多三四个锦标赛 + Mathcounts 吧。
: 我们在烂区GT里面,学校只给上Math 7 honor。好在老师好像比较开通,准备说服老师
: 让他在课上搞MathCounts。
: 他去了两个竞赛队。一个是俱乐部性质的,主要针对ARML等各种团体赛搞点专题讲座啥
: 的;另一个是竞赛班性质,针对AMC/AIME,比较系统的专题讲座和真题讲解。

t*******r
发帖数: 22634
154
她家娃是上一年实战参加 AIME,也就是说,她家娃五年级就 AMC12 top 5% 获得参加
AIME 资格,并且实战参加 AIME 了。

【在 P******e 的大作中提到】
: 这是真牛蛙,上学年才六年级就AIME了,狠赞。
: 你们用的什么教材来'另起炉灶'的?
:
: ★ 发自iPhone App: ChineseWeb 1.0.2

y*******g
发帖数: 1395
155
如果你家的没有身负任何训练任务,也没有任何需要完成的其他项目,这个成绩应该是
A+啊,
我们家的是A-,有时候会取得B+, 但那是确实没有时间的前提下,而且小朋友说初中数
学很简单,不需要辅导,他们还被学校分配到低年级给做TA了,负责判卷和纠错,所以
我没任何怨言

我娃也反应说目前初中数学课有趣也有难度,不像小学时期那么枯燥了。

【在 t*******r 的大作中提到】
: 她家娃是上一年实战参加 AIME,也就是说,她家娃五年级就 AMC12 top 5% 获得参加
: AIME 资格,并且实战参加 AIME 了。

t*******r
发帖数: 22634
156
我们小学没有 GPA,所以没有 A+ 的说法。
另外初中不仅仅是数学,还有很多其它的课,娃说 social study 这种课也不是太容易
的。
其实从初中数学大路货快道的目标和课时作业时间上看,如果不是要走竞赛 track,那
课后有限小灶加料半小时到一小时,从小时数上看,我觉得其实是合理的。
其实另一个角度说,娃初中了也有一定的主见了。既然板门店只能谈判下半个小时多一
点,这本身可能也有其一定的科学必然性。古人云:强扭的瓜不甜。剩下的多余时间,
可以考虑用酱油文艺娱乐来填。另外娃的成长可能也需要相当的 brain idle 的
unstructured play time,或者说,瞎折腾也是生命的一种表现形式。

【在 y*******g 的大作中提到】
: 如果你家的没有身负任何训练任务,也没有任何需要完成的其他项目,这个成绩应该是
: A+啊,
: 我们家的是A-,有时候会取得B+, 但那是确实没有时间的前提下,而且小朋友说初中数
: 学很简单,不需要辅导,他们还被学校分配到低年级给做TA了,负责判卷和纠错,所以
: 我没任何怨言
:
: 我娃也反应说目前初中数学课有趣也有难度,不像小学时期那么枯燥了。

y*******g
发帖数: 1395
157
你的不是在初中吗?
哪天我把我家的成绩单给你看看吧,实际上也是全A的,就一门B+的,

【在 t*******r 的大作中提到】
: 我们小学没有 GPA,所以没有 A+ 的说法。
: 另外初中不仅仅是数学,还有很多其它的课,娃说 social study 这种课也不是太容易
: 的。
: 其实从初中数学大路货快道的目标和课时作业时间上看,如果不是要走竞赛 track,那
: 课后有限小灶加料半小时到一小时,从小时数上看,我觉得其实是合理的。
: 其实另一个角度说,娃初中了也有一定的主见了。既然板门店只能谈判下半个小时多一
: 点,这本身可能也有其一定的科学必然性。古人云:强扭的瓜不甜。剩下的多余时间,
: 可以考虑用酱油文艺娱乐来填。另外娃的成长可能也需要相当的 brain idle 的
: unstructured play time,或者说,瞎折腾也是生命的一种表现形式。

y*******g
发帖数: 1395
158
说一句打击你的话,ss这门课是最好拿分的,据说小朋友最爱学这个了
我抽空去找一下成绩单吧,我印象中,基本都是A以上的

【在 t*******r 的大作中提到】
: 我们小学没有 GPA,所以没有 A+ 的说法。
: 另外初中不仅仅是数学,还有很多其它的课,娃说 social study 这种课也不是太容易
: 的。
: 其实从初中数学大路货快道的目标和课时作业时间上看,如果不是要走竞赛 track,那
: 课后有限小灶加料半小时到一小时,从小时数上看,我觉得其实是合理的。
: 其实另一个角度说,娃初中了也有一定的主见了。既然板门店只能谈判下半个小时多一
: 点,这本身可能也有其一定的科学必然性。古人云:强扭的瓜不甜。剩下的多余时间,
: 可以考虑用酱油文艺娱乐来填。另外娃的成长可能也需要相当的 brain idle 的
: unstructured play time,或者说,瞎折腾也是生命的一种表现形式。

t*******r
发帖数: 22634
159
六年级现在才开学,没成绩。
不过我对看别家娃成绩单没兴趣,不是我家娃不关我事。
在美帝,学校会给各家娃的 National Percentiles or equivalent 的,犯不着在纠结
其他娃的成绩单上浪费小时数。

【在 y*******g 的大作中提到】
: 你的不是在初中吗?
: 哪天我把我家的成绩单给你看看吧,实际上也是全A的,就一门B+的,

t*******r
发帖数: 22634
160
你再进入突突 mode 我就不回你的帖了。我都跟你说了多少次了你还不长记性,美帝学
校给 National Percentiles,因此不用在意其他娃的成绩单啥的。
当然,学校不会给藤校和竞赛的 National Percentiles,大伙儿会争论这个。你这
GPA 成绩单这种学校给冰箱贴的,别争了。
当然,你要争那也是你的自由。不过你再争我就不回贴了。

【在 y*******g 的大作中提到】
: 说一句打击你的话,ss这门课是最好拿分的,据说小朋友最爱学这个了
: 我抽空去找一下成绩单吧,我印象中,基本都是A以上的

相关主题
请问,AMC 8应该从几年级开始练习呀?有明天考AMC 8的吗?
数学教育 一家之言 番外篇求科普: 奥数 vs. Math Olympiad vs. Math Count vs. Math circle vs. 超前学数学
小学升初中,儿子数学跳级的一点经验9岁, 如何报名考amc10
进入Parenting版参与讨论
y*******g
发帖数: 1395
161
也对,看了心堵。
以前每次放假前,我都煽动小朋友去偷窥别人成绩,小朋友眨巴眨巴两只闪亮的大眼睛
摆摆手,告诉我这是隐私,规定不能看人家的, 我说你丫就不能把耳朵竖起来?
后来就领略了我的旨意,基本上,能做到班里谁拿什么成绩都能耳聪目染了,直接告诉
我大概其全班的表现情况了

【在 t*******r 的大作中提到】
: 你再进入突突 mode 我就不回你的帖了。我都跟你说了多少次了你还不长记性,美帝学
: 校给 National Percentiles,因此不用在意其他娃的成绩单啥的。
: 当然,学校不会给藤校和竞赛的 National Percentiles,大伙儿会争论这个。你这
: GPA 成绩单这种学校给冰箱贴的,别争了。
: 当然,你要争那也是你的自由。不过你再争我就不回贴了。

y*******g
发帖数: 1395
162
你如果连自己班里(同等教育条件和生活水准)的同学都比不过的话,跑那么老远,去
比大千世界里,各方面素质的前提条件均为未知数的学生,
我感觉,你是在刻意回避着什么, 也就是说,在班里倒数第一,大概你不觉得该着急
,因为你看了national perc的还会庆幸没那么差---但你忘了那个遥远的圈子的其他条
件了,人家也许是黑人,但能跑能唱啊。。。分数比你低点,根本不耽误事啊
btw,i don't care who replies or not.

【在 t*******r 的大作中提到】
: 你再进入突突 mode 我就不回你的帖了。我都跟你说了多少次了你还不长记性,美帝学
: 校给 National Percentiles,因此不用在意其他娃的成绩单啥的。
: 当然,学校不会给藤校和竞赛的 National Percentiles,大伙儿会争论这个。你这
: GPA 成绩单这种学校给冰箱贴的,别争了。
: 当然,你要争那也是你的自由。不过你再争我就不回贴了。

y*******g
发帖数: 1395
163
对我肯定没啥损失了。。。你还是憋那些文学女青年去吧

【在 t*******r 的大作中提到】
: 你再进入突突 mode 我就不回你的帖了。我都跟你说了多少次了你还不长记性,美帝学
: 校给 National Percentiles,因此不用在意其他娃的成绩单啥的。
: 当然,学校不会给藤校和竞赛的 National Percentiles,大伙儿会争论这个。你这
: GPA 成绩单这种学校给冰箱贴的,别争了。
: 当然,你要争那也是你的自由。不过你再争我就不回贴了。

t*******r
发帖数: 22634
164
不过前面 update 的新信息,对看者的参考确实是有用的。
“另起炉灶” 的适用条件,是大幅度超前。有背景信息的确更能加强 quantify 的
feel,也少些不必要的争论。
J***A
发帖数: 1511
165
都规定看同班同学成绩单是隐私了。 为什么还非要自己孩子看呢, 觉得你这个做法不
可取, 你在美帝就得守美帝的规矩。 尊重自己和别人的孩子。
而且有一个误区就是为什么非要和同班的比呢? 申请大学也好, 什么什么也好, 和
你孩子竞争的都是全国, 至少也是全state的人。
曾经听人说过, 在国内有些学生读高中的时候, 声称自己根本不学习, 为此还特别
在班里装成不学习的样子, 拉着班里自己的竞争对手陪着他们玩, 然后晚上回家偷偷
学到半夜! 不是一个人, 整个班级风气如此。
觉得这样不可笑和可悲吗? 你把自己班的同学坑了骗了有什么意义, 和你竞争的不是
一个省的人吗?
何况who cares 你一天学一个小时还是25个小时, 大学考试不就是一卷定输赢吗?
这个national percentage 其实也同理, 眼光放远点, 别总盯着周围的同学和朋友,
大家共同进步, 互相提携比明争暗斗, 互相往下拽的好。

【在 y*******g 的大作中提到】
: 你如果连自己班里(同等教育条件和生活水准)的同学都比不过的话,跑那么老远,去
: 比大千世界里,各方面素质的前提条件均为未知数的学生,
: 我感觉,你是在刻意回避着什么, 也就是说,在班里倒数第一,大概你不觉得该着急
: ,因为你看了national perc的还会庆幸没那么差---但你忘了那个遥远的圈子的其他条
: 件了,人家也许是黑人,但能跑能唱啊。。。分数比你低点,根本不耽误事啊
: btw,i don't care who replies or not.

y*******g
发帖数: 1395
166
我家的就不劳您费心了。。。人家身价已经进入top 1%了, 还是管好你自家的吧,
真是狗拿耗子,多管闲事

【在 J***A 的大作中提到】
: 都规定看同班同学成绩单是隐私了。 为什么还非要自己孩子看呢, 觉得你这个做法不
: 可取, 你在美帝就得守美帝的规矩。 尊重自己和别人的孩子。
: 而且有一个误区就是为什么非要和同班的比呢? 申请大学也好, 什么什么也好, 和
: 你孩子竞争的都是全国, 至少也是全state的人。
: 曾经听人说过, 在国内有些学生读高中的时候, 声称自己根本不学习, 为此还特别
: 在班里装成不学习的样子, 拉着班里自己的竞争对手陪着他们玩, 然后晚上回家偷偷
: 学到半夜! 不是一个人, 整个班级风气如此。
: 觉得这样不可笑和可悲吗? 你把自己班的同学坑了骗了有什么意义, 和你竞争的不是
: 一个省的人吗?
: 何况who cares 你一天学一个小时还是25个小时, 大学考试不就是一卷定输赢吗?

J***A
发帖数: 1511
167
纯属抬杠:
初中说明不了什么, decay 很严重的,没听某个版友的例子吗?已经 mit 还不能保证
有工作呢,富豪也有破产的, 身价和身家1% 不算啥:)
身高1%还有点用,比如男孩已经180了 至少不太可能缩回去了!

我家的就不劳您费心了。。。人家身价已经进入top 1%了, 还是管好你自家的吧,真
是狗拿耗子,多管闲事

【在 y*******g 的大作中提到】
: 我家的就不劳您费心了。。。人家身价已经进入top 1%了, 还是管好你自家的吧,
: 真是狗拿耗子,多管闲事

t*******r
发帖数: 22634
168
180 也有麻杆化的可能,也不好说。

【在 J***A 的大作中提到】
: 纯属抬杠:
: 初中说明不了什么, decay 很严重的,没听某个版友的例子吗?已经 mit 还不能保证
: 有工作呢,富豪也有破产的, 身价和身家1% 不算啥:)
: 身高1%还有点用,比如男孩已经180了 至少不太可能缩回去了!
:
: 我家的就不劳您费心了。。。人家身价已经进入top 1%了, 还是管好你自家的吧,真
: 是狗拿耗子,多管闲事

t*******r
发帖数: 22634
169
比版上牛蛙差海了,1% 自己看看决定上州立,旱涝保收,那就可以了。来这版
showoff 就 1% 还是比较丢人的。

【在 y*******g 的大作中提到】
: 我家的就不劳您费心了。。。人家身价已经进入top 1%了, 还是管好你自家的吧,
: 真是狗拿耗子,多管闲事

y*******g
发帖数: 1395
170
我可没有你那么弱不禁风,扛不起打击。。。
只要比你家的两个都强出了百倍,我就很开心了, LOL
是你在跟我比,不是其他牛蛙的爹在跟我比。。。

【在 t*******r 的大作中提到】
: 比版上牛蛙差海了,1% 自己看看决定上州立,旱涝保收,那就可以了。来这版
: showoff 就 1% 还是比较丢人的。

相关主题
9岁, 如何报名考amc10给初中学生家长的一些建议 (转载)
算不算有数学天分小学数学上超前班有什么好处吗?
[葩论] 论“有限小灶策略”小学数学有比赛项目吗
进入Parenting版参与讨论
y*******g
发帖数: 1395
171
建议你从我老公的种族开刀,这样对小朋友身高的估算,更方便些

【在 J***A 的大作中提到】
: 纯属抬杠:
: 初中说明不了什么, decay 很严重的,没听某个版友的例子吗?已经 mit 还不能保证
: 有工作呢,富豪也有破产的, 身价和身家1% 不算啥:)
: 身高1%还有点用,比如男孩已经180了 至少不太可能缩回去了!
:
: 我家的就不劳您费心了。。。人家身价已经进入top 1%了, 还是管好你自家的吧,真
: 是狗拿耗子,多管闲事

t*******r
发帖数: 22634
172
强一万倍也只能保证是进州立,你不如说比 Home Depot 门口老墨娃强一千万倍,但还
是只能确保州立不是?

【在 y*******g 的大作中提到】
: 我可没有你那么弱不禁风,扛不起打击。。。
: 只要比你家的两个都强出了百倍,我就很开心了, LOL
: 是你在跟我比,不是其他牛蛙的爹在跟我比。。。

t*******r
发帖数: 22634
173
加州红木?

【在 y*******g 的大作中提到】
: 建议你从我老公的种族开刀,这样对小朋友身高的估算,更方便些
y*******g
发帖数: 1395
174
你怎么爱操那么多没用的心, 连我都懒得操,好几年以后的事,还cares得了那么远的
。享受当下,不好吗?
你要愿意替我们家的小朋友操,我也不拦着,反正没小费就是了, 哪天领过去到你家
听几场忆苦思甜的报告。。。不用费我的事儿,教育了,你看成吗?
每周听一次,反正你闲着也是闲着。 我还乐得享受有人替我呢

【在 t*******r 的大作中提到】
: 强一万倍也只能保证是进州立,你不如说比 Home Depot 门口老墨娃强一千万倍,但还
: 是只能确保州立不是?

y*******g
发帖数: 1395
175
世界上最高的人种是哪儿的?

【在 t*******r 的大作中提到】
: 加州红木?
t*******r
发帖数: 22634
176
物种

【在 y*******g 的大作中提到】
: 世界上最高的人种是哪儿的?
J***A
发帖数: 1511
177
我家ld 中国人妥妥的了, 什么种族不种族的!
我家是女孩, 长高了什么用? 自动屏蔽掉大批不太高的优秀男士? 还是浪费布料!
说真心话, 不是哪个女孩高了都好看, 以为个个张梓琳?
话糙点 驴大了马大了值钱, 人大了值钱吗?
很遗憾我和ld都高, 说实话真愁姑娘长郑海霞那么高又没人家那个打球的本事!

【在 y*******g 的大作中提到】
: 建议你从我老公的种族开刀,这样对小朋友身高的估算,更方便些
y*******g
发帖数: 1395
178
你这精神有点分裂了,是你在问我家的身高,结果你突突突的自问自答了自己和ld的身
高和隐性地暴露了你娃不太高的尴尬局面(其实你和你ld真实多高,我也不敢确信),
我可没有问你和你ld的身高,还有你孩子的身高,和你认识的那头驴的高度,更没有问。
你这个1000度的眼镜,得重新配一个了。。。



【在 J***A 的大作中提到】
: 我家ld 中国人妥妥的了, 什么种族不种族的!
: 我家是女孩, 长高了什么用? 自动屏蔽掉大批不太高的优秀男士? 还是浪费布料!
: 说真心话, 不是哪个女孩高了都好看, 以为个个张梓琳?
: 话糙点 驴大了马大了值钱, 人大了值钱吗?
: 很遗憾我和ld都高, 说实话真愁姑娘长郑海霞那么高又没人家那个打球的本事!

t*******r
发帖数: 22634
179
美国梦很重要,祝福您先。

【在 y*******g 的大作中提到】
: 你怎么爱操那么多没用的心, 连我都懒得操,好几年以后的事,还cares得了那么远的
: 。享受当下,不好吗?
: 你要愿意替我们家的小朋友操,我也不拦着,反正没小费就是了, 哪天领过去到你家
: 听几场忆苦思甜的报告。。。不用费我的事儿,教育了,你看成吗?
: 每周听一次,反正你闲着也是闲着。 我还乐得享受有人替我呢

y*******g
发帖数: 1395
180
祝您睡个好梦,晚安,Mr.潮水

【在 t*******r 的大作中提到】
: 美国梦很重要,祝福您先。
相关主题
请教:怎样说服孩儿他爸多关心孩子的学习?请教小孩数学教育问题
学校早晚要教的东西,早早学会了又如何?小学数学书推荐?
AMC 8 成绩 2016我来说一下天才的鉴定方法
进入Parenting版参与讨论
J***A
发帖数: 1511
181
汗颜啊, 看错了, 以为你上次关心了我家ld的收入,又来关心他身高了:)
你理解错了, 我不关心你家孩子的身高, 只是告诉你学习1% 没什么含金量, 连身高
1%都不如呢!
你信不信我和我ld 的身高也无所谓了, 反正都各自骗到lg/lp,单身的更该纠结吧!

你这精神有点分裂了,是你在问我家的身高,结果你突突突的自问自答了自己和ld的身
高和隐性地暴露了你娃不太高的尴尬局面(其实你和你ld真实多高,我也不敢确信),
我可没有问你和你ld........

【在 y*******g 的大作中提到】
: 你这精神有点分裂了,是你在问我家的身高,结果你突突突的自问自答了自己和ld的身
: 高和隐性地暴露了你娃不太高的尴尬局面(其实你和你ld真实多高,我也不敢确信),
: 我可没有问你和你ld的身高,还有你孩子的身高,和你认识的那头驴的高度,更没有问。
: 你这个1000度的眼镜,得重新配一个了。。。
:
: !

J***A
发帖数: 1511
182
我只是把我字读成你而已!
你进入突突模式就没有回你的必要了。
突突贴删得挺快也算善莫大焉, 所以给你解个惑。

建议你从我老公的种族开刀,这样对小朋友身高的估算,更方便些

【在 y*******g 的大作中提到】
: 建议你从我老公的种族开刀,这样对小朋友身高的估算,更方便些
y*******g
发帖数: 1395
183
所以你眼瞎,并不是我的错。period

【在 J***A 的大作中提到】
: 我只是把我字读成你而已!
: 你进入突突模式就没有回你的必要了。
: 突突贴删得挺快也算善莫大焉, 所以给你解个惑。
:
: 建议你从我老公的种族开刀,这样对小朋友身高的估算,更方便些

j*p
发帖数: 780
184
Wow, 这太厉害了!分享一下你们的学习经历,math 7 honor学啥内容,mathcounts怎
么学?有这么多竞赛机会的学校,应该不是差区了。
我们学校重视文科,数学课时少内容简单,娃都学到algreba 2了,只参加过AMC10。

【在 s***n 的大作中提到】
: 上个学年个人竞赛参加了 MOEMS, AMC 8/10/12,AIME和州大的一个高中竞赛。团体赛
: 参加了ARML,附近大学的一个高中锦标赛和附近高中的一个初中锦标赛。今年个人赛应
: 该没什么变化,但团体赛会多三四个锦标赛 + Mathcounts 吧。
: 我们在烂区GT里面,学校只给上Math 7 honor。好在老师好像比较开通,准备说服老师
: 让他在课上搞MathCounts。
: 他去了两个竞赛队。一个是俱乐部性质的,主要针对ARML等各种团体赛搞点专题讲座啥
: 的;另一个是竞赛班性质,针对AMC/AIME,比较系统的专题讲座和真题讲解。

t*******r
发帖数: 22634
185
其实我建议 syuan 和你考虑另开一楼,写写和讨论讨论有关 “另起炉灶” 的冰箱贴
。这样类似的情况也便于参考讨论。
即使对版上大部分人可能也不太会 “另起炉灶”,但有 “另起炉灶” 的冰箱贴,那
也是个不错的 reference 我觉得。至少也能便于普通群众们 quantify 一下何种背景
需要 “另起炉灶”,何种情况就不需要。这样大伙儿手中有冰箱贴,心中就不慌。
版务可以考虑上包子诱惑。

【在 j*p 的大作中提到】
: Wow, 这太厉害了!分享一下你们的学习经历,math 7 honor学啥内容,mathcounts怎
: 么学?有这么多竞赛机会的学校,应该不是差区了。
: 我们学校重视文科,数学课时少内容简单,娃都学到algreba 2了,只参加过AMC10。

j*p
发帖数: 780
186
一直没有follow你们的另起炉灶冰箱贴,能给总结一下吗?
真希望syuan能分享更多经验供大家学习参考。

【在 t*******r 的大作中提到】
: 其实我建议 syuan 和你考虑另开一楼,写写和讨论讨论有关 “另起炉灶” 的冰箱贴
: 。这样类似的情况也便于参考讨论。
: 即使对版上大部分人可能也不太会 “另起炉灶”,但有 “另起炉灶” 的冰箱贴,那
: 也是个不错的 reference 我觉得。至少也能便于普通群众们 quantify 一下何种背景
: 需要 “另起炉灶”,何种情况就不需要。这样大伙儿手中有冰箱贴,心中就不慌。
: 版务可以考虑上包子诱惑。

t*******r
发帖数: 22634
187
我是 “有限小灶” 而不是 “另起炉灶”。我已经发过贴了。
关键是,我和 syuan 讨论半天,才知道伊的 “另起炉灶” 的背景是五年级参加 AIME
竞赛。这个还是大概说明一下策略所对应的背景才好,否则我个人觉得,确实比较容
易误导普通娃父母。
另外这个背景,跟 repetitive training 也很相关。数学竞赛要求的速度熟练和理解
度,确实对 repetitive training 的要求高很多。没有背景,普通娃家长还以为一般
的小学初中数学课也要那么多 repetitive training。这就有点像目标是 SEAL 狙击手
vs 去小孟买警局混个底层位子,对射击训练要求的差别。

【在 j*p 的大作中提到】
: 一直没有follow你们的另起炉灶冰箱贴,能给总结一下吗?
: 真希望syuan能分享更多经验供大家学习参考。

t*******r
发帖数: 22634
188
另外我刚才根据最新信息思考了一下,在课后加料里:
我觉得最需要 repetitive training 的,反而是数学竞赛的尖子,就好比 SEAL 的狙
击手。
其次需要 repetitive training 的,是分数上加一点就能确保进州大的这种,毕竟是
实在可控的利益。
而最不需要 repetitive training 的,反而是大路货型的快车道上的那种。因为实际
的分数上不上也下不下,没有太多实际分数的利益。而确保跟得上快车道的进度,以及
确保将来轻松跟上进度,更实在。
虽然上我面这个看法看起来非常 counter-intuitive,但逻辑上好像 plausible。

【在 j*p 的大作中提到】
: 一直没有follow你们的另起炉灶冰箱贴,能给总结一下吗?
: 真希望syuan能分享更多经验供大家学习参考。

s**********y
发帖数: 509
189
这个相当于小学五年级参加全国高中数学联赛, 拿了地区(地级市)前十名。 要在中
国是要上报纸的。 syuan: 敬仰如江水。 与淘天才就算有差别, 应该也不大了。
syuan不应该在这混了。

AIME

【在 t*******r 的大作中提到】
: 我是 “有限小灶” 而不是 “另起炉灶”。我已经发过贴了。
: 关键是,我和 syuan 讨论半天,才知道伊的 “另起炉灶” 的背景是五年级参加 AIME
: 竞赛。这个还是大概说明一下策略所对应的背景才好,否则我个人觉得,确实比较容
: 易误导普通娃父母。
: 另外这个背景,跟 repetitive training 也很相关。数学竞赛要求的速度熟练和理解
: 度,确实对 repetitive training 的要求高很多。没有背景,普通娃家长还以为一般
: 的小学初中数学课也要那么多 repetitive training。这就有点像目标是 SEAL 狙击手
: vs 去小孟买警局混个底层位子,对射击训练要求的差别。

Y********d
发帖数: 1478
190
同意,syuan不妨写个“另起炉灶”冰箱贴,正好对应潮水的“有限小灶”冰箱贴。
不同意,syuan不应该在这混了,有个牛蛙牛爸让我们敬仰一下,有何不可?

AIME

【在 s**********y 的大作中提到】
: 这个相当于小学五年级参加全国高中数学联赛, 拿了地区(地级市)前十名。 要在中
: 国是要上报纸的。 syuan: 敬仰如江水。 与淘天才就算有差别, 应该也不大了。
: syuan不应该在这混了。
:
: AIME

相关主题
[华盛顿邮报] 9 - 14 岁是数学兴趣的关键时期?数学教育 一家之言 番外篇
怎么推小学数学好?小学一年级的孩子。小学升初中,儿子数学跳级的一点经验
请问,AMC 8应该从几年级开始练习呀?有明天考AMC 8的吗?
进入Parenting版参与讨论
s**********y
发帖数: 509
191
强烈同意/呼唤 syuan 写帖传经送宝。
不过凭我肉眼估计, 这个在7 sigma 之外, 成功做不可复制状。

【在 Y********d 的大作中提到】
: 同意,syuan不妨写个“另起炉灶”冰箱贴,正好对应潮水的“有限小灶”冰箱贴。
: 不同意,syuan不应该在这混了,有个牛蛙牛爸让我们敬仰一下,有何不可?
:
: AIME

Y********d
发帖数: 1478
192

但是,我再一想,其实比得到冰箱贴更重要的第一步是识别娃的背景条件。
然后,我又一想,到了小学高年级尤其是初中以后,识别背景条件应该不是难事。
所以,还是要做backward induction,回归到小时候的事情。
那么,我这个充满热情的小娃妈,恳请syuan自己谈谈,娃在数学方面的表现,主要是
天赋过人,还是你循循善诱?
我相信两者可能都很重要,而天赋更是必要条件?
娃的天赋,你觉得大概是在他多大的时候以什么形式表现出来的?
给定娃的天赋,在娃很小的时候,如何引近数的概念,如何引导逻辑思考,如何培养观
察总结,等等,这些重不重要?
还是象潮水建议的,三年级以前学校教育足以?
disclaimer: 我对数学竞赛没有心结,老公有很大心结,准备磨刀霍霍向女儿。

【在 Y********d 的大作中提到】
: 同意,syuan不妨写个“另起炉灶”冰箱贴,正好对应潮水的“有限小灶”冰箱贴。
: 不同意,syuan不应该在这混了,有个牛蛙牛爸让我们敬仰一下,有何不可?
:
: AIME

Y********d
发帖数: 1478
193

同意成功不可复制。
但是根据潮水的观察,本版的娃娃在右边3 sigma 之外的不少,至少有启发意义吧。
syuan写吧,谢过先!

【在 s**********y 的大作中提到】
: 强烈同意/呼唤 syuan 写帖传经送宝。
: 不过凭我肉眼估计, 这个在7 sigma 之外, 成功做不可复制状。

s**********y
发帖数: 509
194
另外恳请介绍初中锦标赛, syuan, you have made my day!

【在 Y********d 的大作中提到】
:
: 同意成功不可复制。
: 但是根据潮水的观察,本版的娃娃在右边3 sigma 之外的不少,至少有启发意义吧。
: syuan写吧,谢过先!

j*p
发帖数: 780
195
再次拜读此贴看见了数学锦标赛和数学俱乐部,看来你们在华人多的地方?有资源优势
啊!
我们年初AMC10前,最后一次练习是请的一学生家长州大统计系的老师来讲,学校老师
已讲不了。

【在 s***n 的大作中提到】
: 上个学年个人竞赛参加了 MOEMS, AMC 8/10/12,AIME和州大的一个高中竞赛。团体赛
: 参加了ARML,附近大学的一个高中锦标赛和附近高中的一个初中锦标赛。今年个人赛应
: 该没什么变化,但团体赛会多三四个锦标赛 + Mathcounts 吧。
: 我们在烂区GT里面,学校只给上Math 7 honor。好在老师好像比较开通,准备说服老师
: 让他在课上搞MathCounts。
: 他去了两个竞赛队。一个是俱乐部性质的,主要针对ARML等各种团体赛搞点专题讲座啥
: 的;另一个是竞赛班性质,针对AMC/AIME,比较系统的专题讲座和真题讲解。

l*****8
发帖数: 16949
196
完全同意。syuan的经我也想读读,不过读的目的是因为纯属好奇和敬仰。这个经我家
孩子是肯定念不来的。

【在 s**********y 的大作中提到】
: 这个相当于小学五年级参加全国高中数学联赛, 拿了地区(地级市)前十名。 要在中
: 国是要上报纸的。 syuan: 敬仰如江水。 与淘天才就算有差别, 应该也不大了。
: syuan不应该在这混了。
:
: AIME

t*******r
发帖数: 22634
197
属实

【在 Y********d 的大作中提到】
:
: 同意成功不可复制。
: 但是根据潮水的观察,本版的娃娃在右边3 sigma 之外的不少,至少有启发意义吧。
: syuan写吧,谢过先!

t*******r
发帖数: 22634
198
赞同你的看法。

【在 Y********d 的大作中提到】
:
: 同意成功不可复制。
: 但是根据潮水的观察,本版的娃娃在右边3 sigma 之外的不少,至少有启发意义吧。
: syuan写吧,谢过先!

Y********d
发帖数: 1478
199
我那个贴子里没有看法都是问题啊。还是那句“回归到小时候”?
那你说说,两个女娃,你怎么看出来小娃比大娃数学潜力好的?
是因为你们对小娃更早投入,从而小娃更早接触数学?还是你能直观感觉出来?

【在 t*******r 的大作中提到】
: 赞同你的看法。
t*******r
发帖数: 22634
200
这就是我家上峰的 wishful thinking 而已。
我觉得差不多。

【在 Y********d 的大作中提到】
: 我那个贴子里没有看法都是问题啊。还是那句“回归到小时候”?
: 那你说说,两个女娃,你怎么看出来小娃比大娃数学潜力好的?
: 是因为你们对小娃更早投入,从而小娃更早接触数学?还是你能直观感觉出来?

相关主题
求科普: 奥数 vs. Math Olympiad vs. Math Count vs. Math circle vs. 超前学数学[葩论] 论“有限小灶策略”
9岁, 如何报名考amc10给初中学生家长的一些建议 (转载)
算不算有数学天分小学数学上超前班有什么好处吗?
进入Parenting版参与讨论
s***n
发帖数: 1280
201
我儿子的数学学习算是两条腿走路:基础知识 + 竞赛培训。
一条腿是学习基础知识,这个我们就是用的普通数学教材。高中用的是Prentice Hall
那个系列。选教材前,我去一个数理磁校网站看了下他们用什么教材,发现他们用的就
是一般高中教材。我觉得美国这边教材大多不错,基础概念解释得生动而清楚。造成好
校坏校差距的主要是老师和学生。小学高年级和初中我们没用教材,用的是ipad上的
iTooch5-8年级分年级数学App。之所以选择iTooch是因为它(1) 分知识点做练习评估 -
适合我来检查孩子哪个概念不清,(2) 做得全对有小星星小动画 - 儿子小吃这套有兴
趣做,(3) 做错有提示 - 省得我出马教,(4) 还便宜 - 这个其实是关键原因 ...同类
型的老美圈子的教育网站,比如IXL和Mathletics啥的,看上去都还可以。
竞赛培训我们买的教材很多,但很多是为以后的学习买的,不少教材我们做都没开始做
。我们买的教材主要是AOPS系列教材/竞赛教材 (推荐),和永成的MathCount和AMC 50
讲系列, First Steps for Math Olympians。做完的是First Steps for Math
Olympians(比较好的竞赛入门书,题量不大,但内容涵盖广),永成的MathCount50讲系
列(喜欢它少讲多练的讲座形式,但typo太多,书有点枯燥乏味加题海)。后者题目比较
多,我一般让我儿子选题做。AOPS系列教材/竞赛教材其实不错,但系列教材是几年的
学习量,竞赛教材我们也只有时间做一小部分。
竞赛培训其实我们主要靠的是做2000年后的真题,加上AOPS和中文学校的竞赛培训课。

【在 P******e 的大作中提到】
: 这是真牛蛙,上学年才六年级就AIME了,狠赞。
: 你们用的什么教材来'另起炉灶'的?
:
: ★ 发自iPhone App: ChineseWeb 1.0.2

s***n
发帖数: 1280
202
我们区的数理公校只参加了州大举办的高中数学竞赛,连AMC都不玩。个人竞赛我们基
本是去附近的大学考点考的,团体竞赛是跟着附近好区的数学俱乐部考的。
math 7 honor应该是学提高点的七年级初中数学。我儿子在家已经学完Pre-calc,上个
学年的几个高中竞赛也都拿了学区第一(差区的好处)。但学校说七年级数学很重要,所
以我们在学校只能学七年级初中数学(差区的坏处)。

【在 j*p 的大作中提到】
: Wow, 这太厉害了!分享一下你们的学习经历,math 7 honor学啥内容,mathcounts怎
: 么学?有这么多竞赛机会的学校,应该不是差区了。
: 我们学校重视文科,数学课时少内容简单,娃都学到algreba 2了,只参加过AMC10。

J***A
发帖数: 1511
203
赞宝贵经验!
虽然暂时用不上, 以后也不一定能用上, 还是收藏了:)

我儿子的数学学习算是两条腿走路:基础知识 竞赛培训。一条腿是学习基础知识,
这个我们就是用的普通数学教材。高中用的是Prentice Hall那个系列。选教材前,我
去一个数理........

【在 s***n 的大作中提到】
: 我儿子的数学学习算是两条腿走路:基础知识 + 竞赛培训。
: 一条腿是学习基础知识,这个我们就是用的普通数学教材。高中用的是Prentice Hall
: 那个系列。选教材前,我去一个数理磁校网站看了下他们用什么教材,发现他们用的就
: 是一般高中教材。我觉得美国这边教材大多不错,基础概念解释得生动而清楚。造成好
: 校坏校差距的主要是老师和学生。小学高年级和初中我们没用教材,用的是ipad上的
: iTooch5-8年级分年级数学App。之所以选择iTooch是因为它(1) 分知识点做练习评估 -
: 适合我来检查孩子哪个概念不清,(2) 做得全对有小星星小动画 - 儿子小吃这套有兴
: 趣做,(3) 做错有提示 - 省得我出马教,(4) 还便宜 - 这个其实是关键原因 ...同类
: 型的老美圈子的教育网站,比如IXL和Mathletics啥的,看上去都还可以。
: 竞赛培训我们买的教材很多,但很多是为以后的学习买的,不少教材我们做都没开始做

s***n
发帖数: 1280
204
你的"有限小灶"贴,我可是一言没发的。教育本来就是个开放性题目。因材施教是教育
的原则之一。不同孩子不同对策。我从来没反对过什么"有限小灶"。我原贴中把什么情
况下可以考虑"另起炉灶",为什么时候要repetitive training都解释得比较清楚了。
不费话了,上原贴,大家自己看。
“是的。Repetitive routine work要做的什么程度,每个孩子会不太一样,聪明的两三
个例子就知道了,没开窍的可能要十几上百遍。Repetitive routine work做得不够,
孩子会概念不牢;做得太多,孩子会无聊。这个程度需要老师和家长来掌握。问题是很
多公立学校老师喜欢一刀切: 来来来,大家每人做十遍,结果就是聪明的孩子觉得上课
无聊,没开窍的还是没开窍。对于后者,家长课后开小灶可以解决。对于前者,家长也
可以开小灶上难题,让孩子保持学习兴趣,并争取把孩子推到更适合他的快班。但也有
些情况下,没有快班或没有合适的快班,而长期让聪明的孩子在课堂上感到学习无聊,
会打击他们的学习积极性,这种情况下家长就应该考虑另起炉灶。”
“美国学校的重复训练比较差,离“题做百遍,理论自现”的传统方式远着呢。我觉得这
是美国数学教育差的一个原因,所以课外homeschooling的时候要弥补这方面的不足。
重复训练对少儿数学思维的形成和发展以及数学概念的建立有很大帮助,特别是小学低
年级。自然这有个量的问题,物极必反;还有个形式的问题,重复训练容易让孩子失去
兴趣,所以要尽量选择活泼点的形式再加些奖励啥的。
在小初中阶段,如果前期重复训练底子比较好的话,后期跳过重复训练,直接做GT题来
学习新内容是可行的。但这背后的主要原因是美国数学教育是螺旋式上升,车轱辘话年
年讲,每年进一点。高年级的新内容很多在低年级已经打好底子了,所以你可以跳过重
复训练的阶段。
在高中阶段,基础知识的重复训练 - 先学再做- 和解题技巧方面的练习中学习 - 先做
再学 是相辅相成的。跳过前者,想直接从后者中理解理论和概念,对大多数普通娃,
甚至数学竞赛娃,都是不太好的。”
“如果是中国的教育系统,家里重复建设就不必要了,因为学校系统的重复训练就很多了
。但美国的教育系统,特别是数学这一块,对重复性训练的重要性认识不够,直接导致
整体数学教育水平低下,所以家长在家里做些重复建设或者量孩子力做些超前建设是有
必要的。另外美国小中学数学老师水平参差不平。碰到不好的老师,而家长又有能力有
时间的话,在数学方面以homeschool为主,学校教育为辅是可行的”
“某种意义上,我们确实讨论的是homeschool - 我们讨论的不是课外加小灶,而是课外
另起炉灶,然后把学校里的数学课变为课外活动 - 写作绘画类活动。
同意兴趣很重要,但也说来话长,今天先跳过。”
“至于数学教育,我们家是完全另起炉灶的,所以这方面我有些实际经验的。”

AIME

【在 t*******r 的大作中提到】
: 我是 “有限小灶” 而不是 “另起炉灶”。我已经发过贴了。
: 关键是,我和 syuan 讨论半天,才知道伊的 “另起炉灶” 的背景是五年级参加 AIME
: 竞赛。这个还是大概说明一下策略所对应的背景才好,否则我个人觉得,确实比较容
: 易误导普通娃父母。
: 另外这个背景,跟 repetitive training 也很相关。数学竞赛要求的速度熟练和理解
: 度,确实对 repetitive training 的要求高很多。没有背景,普通娃家长还以为一般
: 的小学初中数学课也要那么多 repetitive training。这就有点像目标是 SEAL 狙击手
: vs 去小孟买警局混个底层位子,对射击训练要求的差别。

P******e
发帖数: 1325
205
娃的天赋在两三岁就可以看出一些。
我娃两岁半可以用中英文数一百块积木,三岁十个月就摆脱了掰手指掰脚趾,
直接心算二十以内加法,十以内减法。最难可以心算5个8连加。
不过她现在还不到四岁,不知道以后会不会decay了。

★ 发自iPhone App: ChineseWeb 1.0.2

【在 Y********d 的大作中提到】
: 我那个贴子里没有看法都是问题啊。还是那句“回归到小时候”?
: 那你说说,两个女娃,你怎么看出来小娃比大娃数学潜力好的?
: 是因为你们对小娃更早投入,从而小娃更早接触数学?还是你能直观感觉出来?

l*****8
发帖数: 16949
206
你孩子很牛。

【在 P******e 的大作中提到】
: 娃的天赋在两三岁就可以看出一些。
: 我娃两岁半可以用中英文数一百块积木,三岁十个月就摆脱了掰手指掰脚趾,
: 直接心算二十以内加法,十以内减法。最难可以心算5个8连加。
: 不过她现在还不到四岁,不知道以后会不会decay了。
:
: ★ 发自iPhone App: ChineseWeb 1.0.2

l*****8
发帖数: 16949
207
信息很多。赞!你这个应该单独开一贴,不然埋在这里可惜了。
顺便问两个问题:1.永成的这些东西是啥?2.你家孩子课外花在数学上的时间一周大概
有多少?谢谢。

Hall
-

【在 s***n 的大作中提到】
: 我儿子的数学学习算是两条腿走路:基础知识 + 竞赛培训。
: 一条腿是学习基础知识,这个我们就是用的普通数学教材。高中用的是Prentice Hall
: 那个系列。选教材前,我去一个数理磁校网站看了下他们用什么教材,发现他们用的就
: 是一般高中教材。我觉得美国这边教材大多不错,基础概念解释得生动而清楚。造成好
: 校坏校差距的主要是老师和学生。小学高年级和初中我们没用教材,用的是ipad上的
: iTooch5-8年级分年级数学App。之所以选择iTooch是因为它(1) 分知识点做练习评估 -
: 适合我来检查孩子哪个概念不清,(2) 做得全对有小星星小动画 - 儿子小吃这套有兴
: 趣做,(3) 做错有提示 - 省得我出马教,(4) 还便宜 - 这个其实是关键原因 ...同类
: 型的老美圈子的教育网站,比如IXL和Mathletics啥的,看上去都还可以。
: 竞赛培训我们买的教材很多,但很多是为以后的学习买的,不少教材我们做都没开始做

t*******r
发帖数: 22634
208
谢谢总结。我基本同意你的观点。
不过我从大众娃家长的角度看,我觉得有两点可能值得进一步讨论:
(1) repetitive training vs problem solving。其实我对美国的学校数学教育,也不
是没有看法。但我觉得是 problem solving 不够,而不是 repetitive training 不够
。(当然是站在大众娃家长的角度)。
(2) 目标定义:我觉得任何 training 的程度,也取决于娃的 end of day 的目标是啥
。其实美国学校数学教育不给力的一个原因,也是因为其并不是为 STEM 娃打造。
我觉得不同的娃家庭,其背景情况不一样。加上上面两点,我觉得给不同背景娃家庭之
间,有个沟通的桥梁。

两三

【在 s***n 的大作中提到】
: 你的"有限小灶"贴,我可是一言没发的。教育本来就是个开放性题目。因材施教是教育
: 的原则之一。不同孩子不同对策。我从来没反对过什么"有限小灶"。我原贴中把什么情
: 况下可以考虑"另起炉灶",为什么时候要repetitive training都解释得比较清楚了。
: 不费话了,上原贴,大家自己看。
: “是的。Repetitive routine work要做的什么程度,每个孩子会不太一样,聪明的两三
: 个例子就知道了,没开窍的可能要十几上百遍。Repetitive routine work做得不够,
: 孩子会概念不牢;做得太多,孩子会无聊。这个程度需要老师和家长来掌握。问题是很
: 多公立学校老师喜欢一刀切: 来来来,大家每人做十遍,结果就是聪明的孩子觉得上课
: 无聊,没开窍的还是没开窍。对于后者,家长课后开小灶可以解决。对于前者,家长也
: 可以开小灶上难题,让孩子保持学习兴趣,并争取把孩子推到更适合他的快班。但也有

s**********y
发帖数: 509
209
赞双基。 你这不是另起炉灶,不是建了一个锅炉, 不是建了一个发电站, 简直是建
了一个核反应堆哈。
阅读导读:
从用iPad 的App 学数学直接过渡阅读高中教材。 非超天才娃不能。 美食高中教材 (
algebra I, II, geometry), 秉承美食教材风格, 事无巨细, 详尽细致, 重点不突
出,啰嗦有余,唐僧中的金蝉子, 战斗机中的F35, 重点挑战人的耐心, 一般不适合
做天才娃的教材 (天才娃的心里特性, 包括不耐烦等, 参阅考古贴)。 你娃应该做
到了否定之否定, 确实值得大家敬仰。
从高中教材直接过渡到AMC10/12。数学禁赛不同于数学,有交集也有独特的地方。 越
往上越不可重复练习。 袁颖很简单,出题难, 出好题更难。知道一个trick和自己
workout一个trick对于ACM12以上有天壤之别。不信, 在桌的家长都学完了高中数学,
回想当年,再自己做做AMC12/AIME 就知道了。 你娃直接过渡, 颇有古仁人之风,
且ACM8/10/12一起上, 断非一般牛蛙所能及也。
您的成功应该说一般不能复制, 但不妨碍我等高山仰止。
如此速度, 明年国家奥林匹克队该有syuan娃了。

Hall
-

【在 s***n 的大作中提到】
: 我儿子的数学学习算是两条腿走路:基础知识 + 竞赛培训。
: 一条腿是学习基础知识,这个我们就是用的普通数学教材。高中用的是Prentice Hall
: 那个系列。选教材前,我去一个数理磁校网站看了下他们用什么教材,发现他们用的就
: 是一般高中教材。我觉得美国这边教材大多不错,基础概念解释得生动而清楚。造成好
: 校坏校差距的主要是老师和学生。小学高年级和初中我们没用教材,用的是ipad上的
: iTooch5-8年级分年级数学App。之所以选择iTooch是因为它(1) 分知识点做练习评估 -
: 适合我来检查孩子哪个概念不清,(2) 做得全对有小星星小动画 - 儿子小吃这套有兴
: 趣做,(3) 做错有提示 - 省得我出马教,(4) 还便宜 - 这个其实是关键原因 ...同类
: 型的老美圈子的教育网站,比如IXL和Mathletics啥的,看上去都还可以。
: 竞赛培训我们买的教材很多,但很多是为以后的学习买的,不少教材我们做都没开始做

t*******r
发帖数: 22634
210
赞字字珠玑。

【在 s**********y 的大作中提到】
: 赞双基。 你这不是另起炉灶,不是建了一个锅炉, 不是建了一个发电站, 简直是建
: 了一个核反应堆哈。
: 阅读导读:
: 从用iPad 的App 学数学直接过渡阅读高中教材。 非超天才娃不能。 美食高中教材 (
: algebra I, II, geometry), 秉承美食教材风格, 事无巨细, 详尽细致, 重点不突
: 出,啰嗦有余,唐僧中的金蝉子, 战斗机中的F35, 重点挑战人的耐心, 一般不适合
: 做天才娃的教材 (天才娃的心里特性, 包括不耐烦等, 参阅考古贴)。 你娃应该做
: 到了否定之否定, 确实值得大家敬仰。
: 从高中教材直接过渡到AMC10/12。数学禁赛不同于数学,有交集也有独特的地方。 越
: 往上越不可重复练习。 袁颖很简单,出题难, 出好题更难。知道一个trick和自己

相关主题
小学数学上超前班有什么好处吗?学校早晚要教的东西,早早学会了又如何?
小学数学有比赛项目吗AMC 8 成绩 2016
请教:怎样说服孩儿他爸多关心孩子的学习?请教小孩数学教育问题
进入Parenting版参与讨论
s***n
发帖数: 1280
211
我们这华人是比较多,但不算是竞赛强州,连个好的竞赛夏令营都没有。我们去的数学
俱乐部也是美国人主持的 - 学生倒是以亚裔为主。
其实现在网上好资源挺多,比如AOPS。好好利用可以弥补竞赛弱州的一些资源劣势。

【在 j*p 的大作中提到】
: 再次拜读此贴看见了数学锦标赛和数学俱乐部,看来你们在华人多的地方?有资源优势
: 啊!
: 我们年初AMC10前,最后一次练习是请的一学生家长州大统计系的老师来讲,学校老师
: 已讲不了。

x***1
发帖数: 999
212
真金白银啊。

Hall
-

【在 s***n 的大作中提到】
: 我儿子的数学学习算是两条腿走路:基础知识 + 竞赛培训。
: 一条腿是学习基础知识,这个我们就是用的普通数学教材。高中用的是Prentice Hall
: 那个系列。选教材前,我去一个数理磁校网站看了下他们用什么教材,发现他们用的就
: 是一般高中教材。我觉得美国这边教材大多不错,基础概念解释得生动而清楚。造成好
: 校坏校差距的主要是老师和学生。小学高年级和初中我们没用教材,用的是ipad上的
: iTooch5-8年级分年级数学App。之所以选择iTooch是因为它(1) 分知识点做练习评估 -
: 适合我来检查孩子哪个概念不清,(2) 做得全对有小星星小动画 - 儿子小吃这套有兴
: 趣做,(3) 做错有提示 - 省得我出马教,(4) 还便宜 - 这个其实是关键原因 ...同类
: 型的老美圈子的教育网站,比如IXL和Mathletics啥的,看上去都还可以。
: 竞赛培训我们买的教材很多,但很多是为以后的学习买的,不少教材我们做都没开始做

s***n
发帖数: 1280
213
1,你在Amazon上搜索 Yongcheng Chen 或 50 Lectures MathCounts,应该可以找到一
系列他的竞赛书。我听说他女儿六年级的时候拿过MathCounts全国第一。以前他在文学
城里也比较活跃。
他最早出的是一套MathCounts 50讲,后来又出了AMC 8 50讲,AMC 50讲,以及
MathCounts几个分赛的专题辅导书。现在好像开始出AIME了。
2,学校开课的日子他大概每天一小时吧。暑假和周末会有两三个小时。

【在 l*****8 的大作中提到】
: 信息很多。赞!你这个应该单独开一贴,不然埋在这里可惜了。
: 顺便问两个问题:1.永成的这些东西是啥?2.你家孩子课外花在数学上的时间一周大概
: 有多少?谢谢。
:
: Hall
: -

x***1
发帖数: 999
214
真心感谢分享。

【在 s***n 的大作中提到】
: 1,你在Amazon上搜索 Yongcheng Chen 或 50 Lectures MathCounts,应该可以找到一
: 系列他的竞赛书。我听说他女儿六年级的时候拿过MathCounts全国第一。以前他在文学
: 城里也比较活跃。
: 他最早出的是一套MathCounts 50讲,后来又出了AMC 8 50讲,AMC 50讲,以及
: MathCounts几个分赛的专题辅导书。现在好像开始出AIME了。
: 2,学校开课的日子他大概每天一小时吧。暑假和周末会有两三个小时。

t*******r
发帖数: 22634
215
这个非常牛。一般娃每天一小时基本只能州考成绩稍微好点。实话实说。
或者说,路上随便一个 Kumon 班,或者课后学校,花的时间都得有差不多一小时。
另一方面,这每天一小时咋都不能归成 repetitive training。归成 problem solving
更合理。个人看法。
n********h
发帖数: 13135
216
赞无私分享。等着你儿子拿金牌的好消息

【在 s***n 的大作中提到】
: 1,你在Amazon上搜索 Yongcheng Chen 或 50 Lectures MathCounts,应该可以找到一
: 系列他的竞赛书。我听说他女儿六年级的时候拿过MathCounts全国第一。以前他在文学
: 城里也比较活跃。
: 他最早出的是一套MathCounts 50讲,后来又出了AMC 8 50讲,AMC 50讲,以及
: MathCounts几个分赛的专题辅导书。现在好像开始出AIME了。
: 2,学校开课的日子他大概每天一小时吧。暑假和周末会有两三个小时。

t*******r
发帖数: 22634
217
同时感谢分享有价值的信息。
我觉得从小时数和内容上看,“有限小灶” 还是 “另起炉灶”,以及 repetitive
training 还是 problem solving,本质上是自然语言的模糊性造成的。
本质的差别,我认为是推进速度的差别,也就是 advancing speed 的差别,而不是其
它的那些。

【在 s***n 的大作中提到】
: 1,你在Amazon上搜索 Yongcheng Chen 或 50 Lectures MathCounts,应该可以找到一
: 系列他的竞赛书。我听说他女儿六年级的时候拿过MathCounts全国第一。以前他在文学
: 城里也比较活跃。
: 他最早出的是一套MathCounts 50讲,后来又出了AMC 8 50讲,AMC 50讲,以及
: MathCounts几个分赛的专题辅导书。现在好像开始出AIME了。
: 2,学校开课的日子他大概每天一小时吧。暑假和周末会有两三个小时。

s***n
发帖数: 1280
218
他的App是到八年级的,高中数学是九年级开始,衔接上我觉得一般数学娃都不会有问
题。他初中基础部分是自学的,高中基础教材是我教的,基本上选重点和基础的讲,唐
僧的部分大都略过了。我一般是每天和他一起等校车的时候花10-20分钟讲点内容,偶
尔周末花一两个小时用来讲新知识点。
他的高中数学学习轨迹是有点乱的。三年级的时候他自学了SAT 数学。四年级我让他开
始做AMC 10真题,做不出的题,或者我给他讲解,或者自己去AOPS或Youtube上找讲解
视频。做真题时他发挥好的话能拿130多分,可惜真考试的时候他做得不好,没能进
AIME。我觉得发挥不好的原因是他基础不牢,所以我在五年级上学期回头给他系统过高
中数学基
础教材。
我儿子的特点是概念清楚,擅长正面推导,但观察力不强,逆向思考不行。AIME的后半
部分难题现在是他的瓶颈。我对他的数学竞赛实际期望不高,再往前走一步到USAMO,
我们就收兵玩其他的去了。

【在 s**********y 的大作中提到】
: 赞双基。 你这不是另起炉灶,不是建了一个锅炉, 不是建了一个发电站, 简直是建
: 了一个核反应堆哈。
: 阅读导读:
: 从用iPad 的App 学数学直接过渡阅读高中教材。 非超天才娃不能。 美食高中教材 (
: algebra I, II, geometry), 秉承美食教材风格, 事无巨细, 详尽细致, 重点不突
: 出,啰嗦有余,唐僧中的金蝉子, 战斗机中的F35, 重点挑战人的耐心, 一般不适合
: 做天才娃的教材 (天才娃的心里特性, 包括不耐烦等, 参阅考古贴)。 你娃应该做
: 到了否定之否定, 确实值得大家敬仰。
: 从高中教材直接过渡到AMC10/12。数学禁赛不同于数学,有交集也有独特的地方。 越
: 往上越不可重复练习。 袁颖很简单,出题难, 出好题更难。知道一个trick和自己

t*******r
发帖数: 22634
219
:四年级我让他开 始做AMC 10真题,
:做不出的题,或者我给他讲解,或者
:自己去AOPS或Youtube上找讲解 视频。
我觉得这更说明是 problem solving 而不是 repetitive training。当然本质上是大
伙儿交流时自然语言的模糊性的问题。

【在 s***n 的大作中提到】
: 他的App是到八年级的,高中数学是九年级开始,衔接上我觉得一般数学娃都不会有问
: 题。他初中基础部分是自学的,高中基础教材是我教的,基本上选重点和基础的讲,唐
: 僧的部分大都略过了。我一般是每天和他一起等校车的时候花10-20分钟讲点内容,偶
: 尔周末花一两个小时用来讲新知识点。
: 他的高中数学学习轨迹是有点乱的。三年级的时候他自学了SAT 数学。四年级我让他开
: 始做AMC 10真题,做不出的题,或者我给他讲解,或者自己去AOPS或Youtube上找讲解
: 视频。做真题时他发挥好的话能拿130多分,可惜真考试的时候他做得不好,没能进
: AIME。我觉得发挥不好的原因是他基础不牢,所以我在五年级上学期回头给他系统过高
: 中数学基
: 础教材。

x***1
发帖数: 999
220
哇,怎一个赞字了得。

【在 s***n 的大作中提到】
: 他的App是到八年级的,高中数学是九年级开始,衔接上我觉得一般数学娃都不会有问
: 题。他初中基础部分是自学的,高中基础教材是我教的,基本上选重点和基础的讲,唐
: 僧的部分大都略过了。我一般是每天和他一起等校车的时候花10-20分钟讲点内容,偶
: 尔周末花一两个小时用来讲新知识点。
: 他的高中数学学习轨迹是有点乱的。三年级的时候他自学了SAT 数学。四年级我让他开
: 始做AMC 10真题,做不出的题,或者我给他讲解,或者自己去AOPS或Youtube上找讲解
: 视频。做真题时他发挥好的话能拿130多分,可惜真考试的时候他做得不好,没能进
: AIME。我觉得发挥不好的原因是他基础不牢,所以我在五年级上学期回头给他系统过高
: 中数学基
: 础教材。

相关主题
小学数学书推荐?怎么推小学数学好?小学一年级的孩子。
我来说一下天才的鉴定方法请问,AMC 8应该从几年级开始练习呀?
[华盛顿邮报] 9 - 14 岁是数学兴趣的关键时期?数学教育 一家之言 番外篇
进入Parenting版参与讨论
x***1
发帖数: 999
221
我觉得不用区别problem solving 和repetitive training的区别,互相促进吧,量达
到一定程度就会质变。没有量的积累,连门都没有。

【在 t*******r 的大作中提到】
: :四年级我让他开 始做AMC 10真题,
: :做不出的题,或者我给他讲解,或者
: :自己去AOPS或Youtube上找讲解 视频。
: 我觉得这更说明是 problem solving 而不是 repetitive training。当然本质上是大
: 伙儿交流时自然语言的模糊性的问题。

t*******r
发帖数: 22634
222
但还是要对于自家娃 quantify 多少 repeat 比较合适吧。过犹不及还是现实存在并且
也需要避免的。

【在 x***1 的大作中提到】
: 我觉得不用区别problem solving 和repetitive training的区别,互相促进吧,量达
: 到一定程度就会质变。没有量的积累,连门都没有。

x***1
发帖数: 999
223
对,如上面讨论的,多少是个够很难把握,但我总感觉孩子repeat太少,不熟练。

【在 t*******r 的大作中提到】
: 但还是要对于自家娃 quantify 多少 repeat 比较合适吧。过犹不及还是现实存在并且
: 也需要避免的。

t*******r
发帖数: 22634
224
syuan 家娃不可能 repeat 太多的。每天一小时,外加暑假两月每天三小时,这就学到
高中 AIME 竞赛了。你说每个年级内容能 repeat 多少次?我觉得她家是 skipper
training 还差不多,能 skip 就 skip 的那种,这样才可能跑步进入 AIME。我是大实
话。

【在 x***1 的大作中提到】
: 对,如上面讨论的,多少是个够很难把握,但我总感觉孩子repeat太少,不熟练。
x***1
发帖数: 999
225
看来我家孩子学习时间太少了,我觉得要坚持每天一小时,很多的。
我觉得这里不光有孩子的聪明,父母更难能可贵。
syuan孩子基础很扎实的,要达到这种程度,不可能不扎实,不可能skip就skip。

【在 t*******r 的大作中提到】
: syuan 家娃不可能 repeat 太多的。每天一小时,外加暑假两月每天三小时,这就学到
: 高中 AIME 竞赛了。你说每个年级内容能 repeat 多少次?我觉得她家是 skipper
: training 还差不多,能 skip 就 skip 的那种,这样才可能跑步进入 AIME。我是大实
: 话。

t*******r
发帖数: 22634
226
你娃的学校数学,每天一小时数学是基本的。
或者我这么说,算总账的话,syuan 家娃每年差不多 500 小时,算它 6 年,那总共
3000 小时。3000 小时从小学 K 班数学,一直学到 AIME。。。不说啥了。。。

【在 x***1 的大作中提到】
: 看来我家孩子学习时间太少了,我觉得要坚持每天一小时,很多的。
: 我觉得这里不光有孩子的聪明,父母更难能可贵。
: syuan孩子基础很扎实的,要达到这种程度,不可能不扎实,不可能skip就skip。

x***1
发帖数: 999
227
我是说家里。

【在 t*******r 的大作中提到】
: 你娃的学校数学,每天一小时数学是基本的。
: 或者我这么说,算总账的话,syuan 家娃每年差不多 500 小时,算它 6 年,那总共
: 3000 小时。3000 小时从小学 K 班数学,一直学到 AIME。。。不说啥了。。。

s***n
发帖数: 1280
228
是的。大多是problem solving。他小的时候学小学数学有一阵子repetitive training
比较多。学到初高中后,基本上只有学新知识点的时候需要些repetitive training。

solving

【在 t*******r 的大作中提到】
: 这个非常牛。一般娃每天一小时基本只能州考成绩稍微好点。实话实说。
: 或者说,路上随便一个 Kumon 班,或者课后学校,花的时间都得有差不多一小时。
: 另一方面,这每天一小时咋都不能归成 repetitive training。归成 problem solving
: 更合理。个人看法。

x***1
发帖数: 999
229
那个时间是你们辅导的时间吧。

training

【在 s***n 的大作中提到】
: 是的。大多是problem solving。他小的时候学小学数学有一阵子repetitive training
: 比较多。学到初高中后,基本上只有学新知识点的时候需要些repetitive training。
:
: solving

j*p
发帖数: 780
230
起步早,每天一小时看来很关键啊。你们有家庭作业和啥课外活动?
我们平时锻炼回家做完作业拉琴就没时间了。周末有时间才补点,把学校以前学过的加
深学一下。

【在 s***n 的大作中提到】
: 1,你在Amazon上搜索 Yongcheng Chen 或 50 Lectures MathCounts,应该可以找到一
: 系列他的竞赛书。我听说他女儿六年级的时候拿过MathCounts全国第一。以前他在文学
: 城里也比较活跃。
: 他最早出的是一套MathCounts 50讲,后来又出了AMC 8 50讲,AMC 50讲,以及
: MathCounts几个分赛的专题辅导书。现在好像开始出AIME了。
: 2,学校开课的日子他大概每天一小时吧。暑假和周末会有两三个小时。

相关主题
小学升初中,儿子数学跳级的一点经验9岁, 如何报名考amc10
有明天考AMC 8的吗?算不算有数学天分
求科普: 奥数 vs. Math Olympiad vs. Math Count vs. Math circle vs. 超前学数学[葩论] 论“有限小灶策略”
进入Parenting版参与讨论
s***n
发帖数: 1280
231
大家可能有点误解。我强调repetitive training一是说小学早期建立抽象思维的时候
;二是说娃学习新概念的时候。我儿子的概念比较强,所以他的repetitive training
相对较少。
我儿子的高中数学学习其实有点象你说的乱中取胜 - 做题中problem solving中学习。
等他学到一定程度了,我再带着他过了一遍高中内容,补补漏,顺带repetitive
training下比较弱的知识点。

【在 t*******r 的大作中提到】
: :四年级我让他开 始做AMC 10真题,
: :做不出的题,或者我给他讲解,或者
: :自己去AOPS或Youtube上找讲解 视频。
: 我觉得这更说明是 problem solving 而不是 repetitive training。当然本质上是大
: 伙儿交流时自然语言的模糊性的问题。

s**********y
发帖数: 509
232
平几也是爷两在街头聊着聊着就聊齐全了? 道真是牛蛙牛爸。

【在 s***n 的大作中提到】
: 他的App是到八年级的,高中数学是九年级开始,衔接上我觉得一般数学娃都不会有问
: 题。他初中基础部分是自学的,高中基础教材是我教的,基本上选重点和基础的讲,唐
: 僧的部分大都略过了。我一般是每天和他一起等校车的时候花10-20分钟讲点内容,偶
: 尔周末花一两个小时用来讲新知识点。
: 他的高中数学学习轨迹是有点乱的。三年级的时候他自学了SAT 数学。四年级我让他开
: 始做AMC 10真题,做不出的题,或者我给他讲解,或者自己去AOPS或Youtube上找讲解
: 视频。做真题时他发挥好的话能拿130多分,可惜真考试的时候他做得不好,没能进
: AIME。我觉得发挥不好的原因是他基础不牢,所以我在五年级上学期回头给他系统过高
: 中数学基
: 础教材。

j*p
发帖数: 780
233
他们做父母用心是肯定的,跟着学习和去外面注册竞赛等。
我们全靠学校了,俺最近才知道MOEMS不是选择题。娃爹肯定还不知道MOEMS是啥。

【在 x***1 的大作中提到】
: 看来我家孩子学习时间太少了,我觉得要坚持每天一小时,很多的。
: 我觉得这里不光有孩子的聪明,父母更难能可贵。
: syuan孩子基础很扎实的,要达到这种程度,不可能不扎实,不可能skip就skip。

x***1
发帖数: 999
234
我还上网查了一下 AIME。差距太大。

【在 j*p 的大作中提到】
: 他们做父母用心是肯定的,跟着学习和去外面注册竞赛等。
: 我们全靠学校了,俺最近才知道MOEMS不是选择题。娃爹肯定还不知道MOEMS是啥。

s***n
发帖数: 1280
235
我们在车里聊,还好。否则美国教材那么老厚一本,我们就变成数学体育一块练了。
不过平几确实是我儿子的竞赛弱项。他自己也知道,所以最近在自学AOPS的几何教材。
效果不好的话,就让他上网课了。

【在 s**********y 的大作中提到】
: 平几也是爷两在街头聊着聊着就聊齐全了? 道真是牛蛙牛爸。
s***n
发帖数: 1280
236
我儿子偏科。他学数学做数学题效率很高,但做其他科目作业就很磨洋工。基本回家后
一小半时间耗在作业上。
我们锻炼得少。计划是每天至少半小时,但执行不力。除数学外,他的主要课外活动是
弹钢琴。体育课外活动比较杂,游泳,足球,柔道,网球,此起彼伏打酱油得搞着,还
没看到哪个项目他特别喜欢。

【在 j*p 的大作中提到】
: 起步早,每天一小时看来很关键啊。你们有家庭作业和啥课外活动?
: 我们平时锻炼回家做完作业拉琴就没时间了。周末有时间才补点,把学校以前学过的加
: 深学一下。

J***A
发帖数: 1511
237
这个太牛了, 没上初中就高中竞赛第一, 以后绝对得参加国际奥赛拿金牌!!
又一个陶天才!

我们区的数理公校只参加了州大举办的高中数学竞赛,连AMC都不玩。个人竞赛我们基
本是去附近的大学考点考的,团体竞赛是跟着附近好区的数学俱乐部考的。math 7
honor应该是学........

【在 s***n 的大作中提到】
: 我们区的数理公校只参加了州大举办的高中数学竞赛,连AMC都不玩。个人竞赛我们基
: 本是去附近的大学考点考的,团体竞赛是跟着附近好区的数学俱乐部考的。
: math 7 honor应该是学提高点的七年级初中数学。我儿子在家已经学完Pre-calc,上个
: 学年的几个高中竞赛也都拿了学区第一(差区的好处)。但学校说七年级数学很重要,所
: 以我们在学校只能学七年级初中数学(差区的坏处)。

J***A
发帖数: 1511
238
有一项这么强根本就不用纠结短板了。
全面发展是对人才的要求!
你儿子这种绝对天才让他发挥所长足矣了!

我儿子偏科。他学数学做数学题效率很高,但做其他科目作业就很磨洋工。基本回家后
一小半时间耗在作业上。我们锻炼得少。计划是每天至少半小时,但执行不力。除数学
外,他的主要课外活动是弹........

【在 s***n 的大作中提到】
: 我儿子偏科。他学数学做数学题效率很高,但做其他科目作业就很磨洋工。基本回家后
: 一小半时间耗在作业上。
: 我们锻炼得少。计划是每天至少半小时,但执行不力。除数学外,他的主要课外活动是
: 弹钢琴。体育课外活动比较杂,游泳,足球,柔道,网球,此起彼伏打酱油得搞着,还
: 没看到哪个项目他特别喜欢。

x***1
发帖数: 999
239
对,不要纠结其他了。

【在 J***A 的大作中提到】
: 有一项这么强根本就不用纠结短板了。
: 全面发展是对人才的要求!
: 你儿子这种绝对天才让他发挥所长足矣了!
:
: 我儿子偏科。他学数学做数学题效率很高,但做其他科目作业就很磨洋工。基本回家后
: 一小半时间耗在作业上。我们锻炼得少。计划是每天至少半小时,但执行不力。除数学
: 外,他的主要课外活动是弹........

s**********y
发帖数: 509
240
我好像看明白了, 您这招叫以赛代练, 三年级上高考数学 (SAT), 四年级上初中
数学竞赛, 五年级上AMC12, 高中数学联赛。一年上一个台阶, 一年一变样, 三年大
变样。
这样说起来, syuan娃应该比syuan厉害多了。能力/commitment 一样不缺。 心无旁
骛, 一不问我为什么要学数学(这么抽象的数学), 二不问我为什么要参加竞赛,
三做不出来题,绝不气馁,通过禁赛找到弱点, 再补拙。 然后所有均在五年级完成。

AMC 10 虽然补考微积分, 但做题中实际用到微积分的概念。这个我想syuan娃也已经
做到了。

【在 s***n 的大作中提到】
: 我儿子偏科。他学数学做数学题效率很高,但做其他科目作业就很磨洋工。基本回家后
: 一小半时间耗在作业上。
: 我们锻炼得少。计划是每天至少半小时,但执行不力。除数学外,他的主要课外活动是
: 弹钢琴。体育课外活动比较杂,游泳,足球,柔道,网球,此起彼伏打酱油得搞着,还
: 没看到哪个项目他特别喜欢。

相关主题
[葩论] 论“有限小灶策略”小学数学有比赛项目吗
给初中学生家长的一些建议 (转载)请教:怎样说服孩儿他爸多关心孩子的学习?
小学数学上超前班有什么好处吗?学校早晚要教的东西,早早学会了又如何?
进入Parenting版参与讨论
t*******r
发帖数: 22634
241
我觉得你说的三点都不是决定因子,决定因子是:每天一小时,总共 3000 小时,从小
学 K 班学到 AIME。
或者这么说,一般的娃,每天加料半小时,不需要太多理由。而她家娃万众瞩目,那每
天加料一小时就不是问题。不需要问理由,因为一小时干完以后,有的是玩的时间。或
者说,闲着也是闲着。
每天加料一个半小时以上,或者加料一小时也是扔水里的那种情况,才会需要问你所问
的三个问题。syuan 她家不需要问。实话实说。

【在 s**********y 的大作中提到】
: 我好像看明白了, 您这招叫以赛代练, 三年级上高考数学 (SAT), 四年级上初中
: 数学竞赛, 五年级上AMC12, 高中数学联赛。一年上一个台阶, 一年一变样, 三年大
: 变样。
: 这样说起来, syuan娃应该比syuan厉害多了。能力/commitment 一样不缺。 心无旁
: 骛, 一不问我为什么要学数学(这么抽象的数学), 二不问我为什么要参加竞赛,
: 三做不出来题,绝不气馁,通过禁赛找到弱点, 再补拙。 然后所有均在五年级完成。
:
: AMC 10 虽然补考微积分, 但做题中实际用到微积分的概念。这个我想syuan娃也已经
: 做到了。

t*******r
发帖数: 22634
242
同意你的看法。

【在 J***A 的大作中提到】
: 有一项这么强根本就不用纠结短板了。
: 全面发展是对人才的要求!
: 你儿子这种绝对天才让他发挥所长足矣了!
:
: 我儿子偏科。他学数学做数学题效率很高,但做其他科目作业就很磨洋工。基本回家后
: 一小半时间耗在作业上。我们锻炼得少。计划是每天至少半小时,但执行不力。除数学
: 外,他的主要课外活动是弹........

t*******r
发帖数: 22634
243
俺终于传播了 “乱中取胜” 的概念,庆祝俺自己一下。
其实我们这里数学快班的老师,在 presentation 的时候,我个人觉得也是有相当的
“乱中取胜” 的理念的。当然,询问娃的时候,我觉得在具体的 implementation 上
,problem solving 方面可能要加强。

training

【在 s***n 的大作中提到】
: 大家可能有点误解。我强调repetitive training一是说小学早期建立抽象思维的时候
: ;二是说娃学习新概念的时候。我儿子的概念比较强,所以他的repetitive training
: 相对较少。
: 我儿子的高中数学学习其实有点象你说的乱中取胜 - 做题中problem solving中学习。
: 等他学到一定程度了,我再带着他过了一遍高中内容,补补漏,顺带repetitive
: training下比较弱的知识点。

t*******r
发帖数: 22634
244
我就是在说家里。
当然对她家娃的情况,学校数学约等于零,trivial。

【在 x***1 的大作中提到】
: 我是说家里。
x***1
发帖数: 999
245
额的神啊

【在 s**********y 的大作中提到】
: 我好像看明白了, 您这招叫以赛代练, 三年级上高考数学 (SAT), 四年级上初中
: 数学竞赛, 五年级上AMC12, 高中数学联赛。一年上一个台阶, 一年一变样, 三年大
: 变样。
: 这样说起来, syuan娃应该比syuan厉害多了。能力/commitment 一样不缺。 心无旁
: 骛, 一不问我为什么要学数学(这么抽象的数学), 二不问我为什么要参加竞赛,
: 三做不出来题,绝不气馁,通过禁赛找到弱点, 再补拙。 然后所有均在五年级完成。
:
: AMC 10 虽然补考微积分, 但做题中实际用到微积分的概念。这个我想syuan娃也已经
: 做到了。

x***1
发帖数: 999
246
对,他家的娃,学校已经不存在了。
学学高斯吧,找一个有名的数学家,悉心培育吧。

【在 t*******r 的大作中提到】
: 我就是在说家里。
: 当然对她家娃的情况,学校数学约等于零,trivial。

t*******r
发帖数: 22634
247
我觉得你错了。起步早不是关键,每天一小时不是关键。3000 小时从 K 班数学跑步进
入 AIME 的速度深度才是关键。
或者我这么说,如果拥有能 3000 小时从 K版数学跑步进入 AIME 的能力的话,那说实
话,六年级数学一屁股红叉也无所谓。那就每天两小时,八年级跑步进入 AIME/AMO 不
就完事了?

【在 j*p 的大作中提到】
: 起步早,每天一小时看来很关键啊。你们有家庭作业和啥课外活动?
: 我们平时锻炼回家做完作业拉琴就没时间了。周末有时间才补点,把学校以前学过的加
: 深学一下。

t*******r
发帖数: 22634
248
别说六年级一屁股红叉了,九年级一屁股红叉都没有关系。搞物理竞赛好了,九年级一
年,每天两小时物理,假期再加点料,1000 小时跑步通关 F=ma test。

【在 t*******r 的大作中提到】
: 我觉得你错了。起步早不是关键,每天一小时不是关键。3000 小时从 K 班数学跑步进
: 入 AIME 的速度深度才是关键。
: 或者我这么说,如果拥有能 3000 小时从 K版数学跑步进入 AIME 的能力的话,那说实
: 话,六年级数学一屁股红叉也无所谓。那就每天两小时,八年级跑步进入 AIME/AMO 不
: 就完事了?

s**********y
发帖数: 509
249
您这3000小时从何时算起? 每天1小时, 一年365小时。 syuan娃五年级, 11岁,莫
非从 。。。?

【在 t*******r 的大作中提到】
: 我觉得你错了。起步早不是关键,每天一小时不是关键。3000 小时从 K 班数学跑步进
: 入 AIME 的速度深度才是关键。
: 或者我这么说,如果拥有能 3000 小时从 K版数学跑步进入 AIME 的能力的话,那说实
: 话,六年级数学一屁股红叉也无所谓。那就每天两小时,八年级跑步进入 AIME/AMO 不
: 就完事了?

r*g
发帖数: 3159
250
必须进来膜拜。

【在 s***n 的大作中提到】
: 我儿子偏科。他学数学做数学题效率很高,但做其他科目作业就很磨洋工。基本回家后
: 一小半时间耗在作业上。
: 我们锻炼得少。计划是每天至少半小时,但执行不力。除数学外,他的主要课外活动是
: 弹钢琴。体育课外活动比较杂,游泳,足球,柔道,网球,此起彼伏打酱油得搞着,还
: 没看到哪个项目他特别喜欢。

相关主题
AMC 8 成绩 2016我来说一下天才的鉴定方法
请教小孩数学教育问题[华盛顿邮报] 9 - 14 岁是数学兴趣的关键时期?
小学数学书推荐?怎么推小学数学好?小学一年级的孩子。
进入Parenting版参与讨论
s**********y
发帖数: 509
251
过奖了, 有点微言大义而已; 比不上您挥洒自如,玉树凌风

【在 t*******r 的大作中提到】
: 赞字字珠玑。
t*******r
发帖数: 22634
252
syuan 她说了暑假里每天 3 小时,所以我估算每年 500 小时,从 K 班开始算 6 年。
总共 3000 小时。

【在 s**********y 的大作中提到】
: 您这3000小时从何时算起? 每天1小时, 一年365小时。 syuan娃五年级, 11岁,莫
: 非从 。。。?

s**********y
发帖数: 509
253
如此说来,比淘天才只差一点点了。

【在 t*******r 的大作中提到】
: syuan 她说了暑假里每天 3 小时,所以我估算每年 500 小时,从 K 班开始算 6 年。
: 总共 3000 小时。

t*******r
发帖数: 22634
254
陶天才是 5000 小时从 K 班数学跑步进入 IMO 铜牌?

【在 s**********y 的大作中提到】
: 如此说来,比淘天才只差一点点了。
s**********y
发帖数: 509
255
好像是四年级拿的铜牌。 多少小时不知。

【在 t*******r 的大作中提到】
: 陶天才是 5000 小时从 K 班数学跑步进入 IMO 铜牌?
t*******r
发帖数: 22634
256
估计可能也就 3000 小时。

【在 s**********y 的大作中提到】
: 好像是四年级拿的铜牌。 多少小时不知。
s***n
发帖数: 1280
257
我们是从四年级开始才这个运动量搞竞赛的。二三年级,平均下来也就上学的日子半小
时,放假和周末一个半小时。K-1基本就放羊的。

【在 t*******r 的大作中提到】
: syuan 她说了暑假里每天 3 小时,所以我估算每年 500 小时,从 K 班开始算 6 年。
: 总共 3000 小时。

t*******r
发帖数: 22634
258
我重新估算了一下,800 小时从九九乘法表跑步进入 AIME。
版上从此不用再争论九九乘法表了吧。

【在 s***n 的大作中提到】
: 我们是从四年级开始才这个运动量搞竞赛的。二三年级,平均下来也就上学的日子半小
: 时,放假和周末一个半小时。K-1基本就放羊的。

l*****8
发帖数: 16949
259
多谢!你孩子不是牛蛙,使天才蛙。
你孩子的数学历程很有启发。3年级就学SAT,看起来你们是先掌握知识,用潮水喜欢的
比喻就是先占领滩头阵地,再扩大战果。然后4年级以后就专攻竞赛。当然你孩子有这
个能力,你也很懂得因才施教。
你孩子看起来是真心热爱数学,估计看数学书和我们看武侠小说一样津津有味,而且理
解能力和记忆能力都超强。这种成功一般人没法复制。一个小时还是半个小时,重复1
遍还是重复10遍这些对你孩子的成功估计只有epsilon的影响。

【在 s***n 的大作中提到】
: 1,你在Amazon上搜索 Yongcheng Chen 或 50 Lectures MathCounts,应该可以找到一
: 系列他的竞赛书。我听说他女儿六年级的时候拿过MathCounts全国第一。以前他在文学
: 城里也比较活跃。
: 他最早出的是一套MathCounts 50讲,后来又出了AMC 8 50讲,AMC 50讲,以及
: MathCounts几个分赛的专题辅导书。现在好像开始出AIME了。
: 2,学校开课的日子他大概每天一小时吧。暑假和周末会有两三个小时。

Y********d
发帖数: 1478
260
请字字珠玑和挥洒自如的两位回家查水表,记得水表旁边还有个信箱:-)

【在 s**********y 的大作中提到】
: 过奖了, 有点微言大义而已; 比不上您挥洒自如,玉树凌风
相关主题
请问,AMC 8应该从几年级开始练习呀?有明天考AMC 8的吗?
数学教育 一家之言 番外篇求科普: 奥数 vs. Math Olympiad vs. Math Count vs. Math circle vs. 超前学数学
小学升初中,儿子数学跳级的一点经验9岁, 如何报名考amc10
进入Parenting版参与讨论
t*******r
发帖数: 22634
261
这个天才娃可能还是分类型的,SAT 数学还是比较枯燥的吧。大部分天才娃听说是早上
AMC 8 / 10,好像很少听说有正规先上 SAT 数学的。
当然也可能我孤陋寡闻。

1

【在 l*****8 的大作中提到】
: 多谢!你孩子不是牛蛙,使天才蛙。
: 你孩子的数学历程很有启发。3年级就学SAT,看起来你们是先掌握知识,用潮水喜欢的
: 比喻就是先占领滩头阵地,再扩大战果。然后4年级以后就专攻竞赛。当然你孩子有这
: 个能力,你也很懂得因才施教。
: 你孩子看起来是真心热爱数学,估计看数学书和我们看武侠小说一样津津有味,而且理
: 解能力和记忆能力都超强。这种成功一般人没法复制。一个小时还是半个小时,重复1
: 遍还是重复10遍这些对你孩子的成功估计只有epsilon的影响。

Y********d
发帖数: 1478
262

1
赞你的这个评论,靠谱!

【在 l*****8 的大作中提到】
: 多谢!你孩子不是牛蛙,使天才蛙。
: 你孩子的数学历程很有启发。3年级就学SAT,看起来你们是先掌握知识,用潮水喜欢的
: 比喻就是先占领滩头阵地,再扩大战果。然后4年级以后就专攻竞赛。当然你孩子有这
: 个能力,你也很懂得因才施教。
: 你孩子看起来是真心热爱数学,估计看数学书和我们看武侠小说一样津津有味,而且理
: 解能力和记忆能力都超强。这种成功一般人没法复制。一个小时还是半个小时,重复1
: 遍还是重复10遍这些对你孩子的成功估计只有epsilon的影响。

t*******r
发帖数: 22634
263
你是不是先核实一下有多少数学竞赛娃先是正规上 SAT prep 而不是 AMC8?
当然 statistically significant 与否,对天才可能也无所谓就是了。

【在 Y********d 的大作中提到】
:
: 1
: 赞你的这个评论,靠谱!

Y********d
发帖数: 1478
264
赞又一枚小牛娃!
raw talent 很重要,别听潮水在那边危言耸听,那个decay论他自己最近也不说了;
他常说的是,本版的娃娃很多在右边3个sigma以右,你又给他增加了一个观察值 :-)

【在 P******e 的大作中提到】
: 娃的天赋在两三岁就可以看出一些。
: 我娃两岁半可以用中英文数一百块积木,三岁十个月就摆脱了掰手指掰脚趾,
: 直接心算二十以内加法,十以内减法。最难可以心算5个8连加。
: 不过她现在还不到四岁,不知道以后会不会decay了。
:
: ★ 发自iPhone App: ChineseWeb 1.0.2

Y********d
发帖数: 1478
265
明明是你开导我,统计模型于自家个例没有意义的嘛?
那么对于outlier的检验更是没有power了?
:-)

【在 t*******r 的大作中提到】
: 你是不是先核实一下有多少数学竞赛娃先是正规上 SAT prep 而不是 AMC8?
: 当然 statistically significant 与否,对天才可能也无所谓就是了。

l*****8
发帖数: 16949
266
SAT数学没有任何难度,但需要掌握很多知识(对小学生而言)。我猜他家娃这个根本
就不是正规上SAT数学,而是这些内容都学过了,然后做几套真题。对他娃这样的数学
能力,只要学过了内容,就不存在不会做得问题。
像这样的孩子估计全美国也没几个,你不知道当然不算孤陋寡闻。

【在 t*******r 的大作中提到】
: 这个天才娃可能还是分类型的,SAT 数学还是比较枯燥的吧。大部分天才娃听说是早上
: AMC 8 / 10,好像很少听说有正规先上 SAT 数学的。
: 当然也可能我孤陋寡闻。
:
: 1

l*****8
发帖数: 16949
267
其实我现在正在纠结该怎么推一推小娃的数学。小娃课外活动时间花得太多,能花在数
学上的时间极其有限。我不太确定该往深度(做竞赛题)还是往广度(学高年级的内容
)加料。前一阵主要是往深度推。看来如果孩子有一定接受能力的话,先广后深也许更
合适。当然我家娃和syuan娃是不能比的,不过这个方法还是很有启发。
s***n
发帖数: 1280
268
你对我儿子倒是过誉了。他从小对数学有兴趣,抽象思维形成的早,逻辑思维发展的早
,所以他跟很多高年级数学娃一样,对抽象的数学感兴趣。你如果去过数学俱乐部,跟
数学娃打过交道的话,就不会觉得不问为什么要学数学有什么大不了。
一个人容易对自己优势的项目产生和保持兴趣,这是很正常的心理。足球好的参加足球
比赛,数学好的参加数学比赛,顺理成章。倒是如果我逼着我儿子整天训练去参加美国
高中足球联赛,他肯定会我为什么要参加这个比赛...
第三点我儿子确实做得比我好。他喜欢独立思考,很少轻易放弃一道题。这个对竞赛培
训是不利的,但对长远发展应该是有利的。竞赛培训希望娃短时间内学会做某种类型的
题,孩子一时做不出来就会直接教了。今年准备AIME的时候我们就碰到这问题。我给他
报了AOPS的AIME网课,但课上和作业里他自己做不出来的题他不愿意直接看答案学解法
,想留着以后慢慢做。我想想看,觉得从长远的角度看,他这样也不错,就没逼着他看
答案学解题技巧了。
AMC 10/12/AIME 都不会考微积分。有个别题目可以用微积分解出来,但也肯定有初等
代数的解法。我儿子的微积分还是初步。他做题,我只看到他用了极限的思想。

【在 s**********y 的大作中提到】
: 我好像看明白了, 您这招叫以赛代练, 三年级上高考数学 (SAT), 四年级上初中
: 数学竞赛, 五年级上AMC12, 高中数学联赛。一年上一个台阶, 一年一变样, 三年大
: 变样。
: 这样说起来, syuan娃应该比syuan厉害多了。能力/commitment 一样不缺。 心无旁
: 骛, 一不问我为什么要学数学(这么抽象的数学), 二不问我为什么要参加竞赛,
: 三做不出来题,绝不气馁,通过禁赛找到弱点, 再补拙。 然后所有均在五年级完成。
:
: AMC 10 虽然补考微积分, 但做题中实际用到微积分的概念。这个我想syuan娃也已经
: 做到了。

t*******r
发帖数: 22634
269
玩玩 SAT 我觉得可以理解,毕竟 SAT 有代数,光看懂题目对小学娃就不容易。
而且她家娃也不一定需要学过内容才能做,induction 一下加选择题还可以猜,应该不
会枯燥。

【在 l*****8 的大作中提到】
: SAT数学没有任何难度,但需要掌握很多知识(对小学生而言)。我猜他家娃这个根本
: 就不是正规上SAT数学,而是这些内容都学过了,然后做几套真题。对他娃这样的数学
: 能力,只要学过了内容,就不存在不会做得问题。
: 像这样的孩子估计全美国也没几个,你不知道当然不算孤陋寡闻。

t*******r
发帖数: 22634
270
你家的情况跟我家比较像。如果你不是要去参加数学竞赛,每天大约也就半小时,我个
人觉得是,algebra 1 过一下,主力是拿 AMC8 => AMC10 的题目当 problem
solving 做,按需学高年级,而不是先推高年级知识。理由如下:
(1) math 的核心是 problem solving,高年级的知识迟早会教。
(2) 看高年级数学冰箱书太枯燥,娃阻力不是一般的大,有想法优美但不
implementable 的风险。
当然如果你要实际参加数学竞赛,那另说。

【在 l*****8 的大作中提到】
: 其实我现在正在纠结该怎么推一推小娃的数学。小娃课外活动时间花得太多,能花在数
: 学上的时间极其有限。我不太确定该往深度(做竞赛题)还是往广度(学高年级的内容
: )加料。前一阵主要是往深度推。看来如果孩子有一定接受能力的话,先广后深也许更
: 合适。当然我家娃和syuan娃是不能比的,不过这个方法还是很有启发。

相关主题
9岁, 如何报名考amc10给初中学生家长的一些建议 (转载)
算不算有数学天分小学数学上超前班有什么好处吗?
[葩论] 论“有限小灶策略”小学数学有比赛项目吗
进入Parenting版参与讨论
s**********y
发帖数: 509
271
看起来你娃绝对热爱数学,自推有术,家长倒不要拘束了他。 AIME以后还有好多层次
, 大可放手一搏, 就成专业亦无不可。

【在 s***n 的大作中提到】
: 你对我儿子倒是过誉了。他从小对数学有兴趣,抽象思维形成的早,逻辑思维发展的早
: ,所以他跟很多高年级数学娃一样,对抽象的数学感兴趣。你如果去过数学俱乐部,跟
: 数学娃打过交道的话,就不会觉得不问为什么要学数学有什么大不了。
: 一个人容易对自己优势的项目产生和保持兴趣,这是很正常的心理。足球好的参加足球
: 比赛,数学好的参加数学比赛,顺理成章。倒是如果我逼着我儿子整天训练去参加美国
: 高中足球联赛,他肯定会我为什么要参加这个比赛...
: 第三点我儿子确实做得比我好。他喜欢独立思考,很少轻易放弃一道题。这个对竞赛培
: 训是不利的,但对长远发展应该是有利的。竞赛培训希望娃短时间内学会做某种类型的
: 题,孩子一时做不出来就会直接教了。今年准备AIME的时候我们就碰到这问题。我给他
: 报了AOPS的AIME网课,但课上和作业里他自己做不出来的题他不愿意直接看答案学解法

t*******r
发帖数: 22634
272
属实

【在 s**********y 的大作中提到】
: 看起来你娃绝对热爱数学,自推有术,家长倒不要拘束了他。 AIME以后还有好多层次
: , 大可放手一搏, 就成专业亦无不可。

Y********d
发帖数: 1478
273
赞“他喜欢独立思考,很少轻易放弃一道题。”
我原来以为,你因材施教,他有兴趣有能力快狠准的作题目,所以容易竞赛出成绩。
很高兴看到他喜欢独立思考,遇到难题不要看答案,而要好好想。
这个已经不仅是能力的问题,而是品位,或者说是天性的事情了。
竞赛到头了,也就是最快最好的作出来别人出好的题目,而这世上最重要的是找到有意
义的题目。

【在 s***n 的大作中提到】
: 你对我儿子倒是过誉了。他从小对数学有兴趣,抽象思维形成的早,逻辑思维发展的早
: ,所以他跟很多高年级数学娃一样,对抽象的数学感兴趣。你如果去过数学俱乐部,跟
: 数学娃打过交道的话,就不会觉得不问为什么要学数学有什么大不了。
: 一个人容易对自己优势的项目产生和保持兴趣,这是很正常的心理。足球好的参加足球
: 比赛,数学好的参加数学比赛,顺理成章。倒是如果我逼着我儿子整天训练去参加美国
: 高中足球联赛,他肯定会我为什么要参加这个比赛...
: 第三点我儿子确实做得比我好。他喜欢独立思考,很少轻易放弃一道题。这个对竞赛培
: 训是不利的,但对长远发展应该是有利的。竞赛培训希望娃短时间内学会做某种类型的
: 题,孩子一时做不出来就会直接教了。今年准备AIME的时候我们就碰到这问题。我给他
: 报了AOPS的AIME网课,但课上和作业里他自己做不出来的题他不愿意直接看答案学解法

s***n
发帖数: 1280
274
我们当时才是孤陋寡闻。他三年级的时候我们才刚听说有AMC 8这个考试。拿了一套卷
子给他做,25题对了19题。我一看,这才C啊,就没管了。后来才知道这个分数已经可
以拿奖了。
他看的那几盘SAT数学,不是系统讲SAT数学的内容,而是做题讲解的形式。这个形式似
乎比较对我儿子胃口。他很多初中数学知识就是这么学的 - 他二年级开始在中文学校
里学Gr 5/6的GT Math。三年级开始学Gr 7/8的GT Math。

【在 t*******r 的大作中提到】
: 这个天才娃可能还是分类型的,SAT 数学还是比较枯燥的吧。大部分天才娃听说是早上
: AMC 8 / 10,好像很少听说有正规先上 SAT 数学的。
: 当然也可能我孤陋寡闻。
:
: 1

t*******r
发帖数: 22634
275
三年级第一次做 AMC8 就能做对 19 题,说实话我觉得全美都不会有几个人。
大部分人五年级就算能第一次做,一般在 16 到 20 题之间多半直接卡住,得一路
skip 下去。

【在 s***n 的大作中提到】
: 我们当时才是孤陋寡闻。他三年级的时候我们才刚听说有AMC 8这个考试。拿了一套卷
: 子给他做,25题对了19题。我一看,这才C啊,就没管了。后来才知道这个分数已经可
: 以拿奖了。
: 他看的那几盘SAT数学,不是系统讲SAT数学的内容,而是做题讲解的形式。这个形式似
: 乎比较对我儿子胃口。他很多初中数学知识就是这么学的 - 他二年级开始在中文学校
: 里学Gr 5/6的GT Math。三年级开始学Gr 7/8的GT Math。

s**********y
发帖数: 509
276
赞品味。娃的心性, 眼界, 品味最为难得; 仿佛天籁, 非人力可退。

【在 Y********d 的大作中提到】
: 赞“他喜欢独立思考,很少轻易放弃一道题。”
: 我原来以为,你因材施教,他有兴趣有能力快狠准的作题目,所以容易竞赛出成绩。
: 很高兴看到他喜欢独立思考,遇到难题不要看答案,而要好好想。
: 这个已经不仅是能力的问题,而是品位,或者说是天性的事情了。
: 竞赛到头了,也就是最快最好的作出来别人出好的题目,而这世上最重要的是找到有意
: 义的题目。

l*****8
发帖数: 16949
277
哈,真是这样。我其实每天半个小时都不可能保证的,每星期至少有3天她活动要到8点
才能结束。加上其它可能的比如sleep over,学校作业太多,有时候我有事,每星期也
就最多3天能学点东西。
我从来没给孩子直接看高年级数学书,就直接给她讲个概念,然后做几道练习。有时她
接受的比较好,有些(比如排列组合)就有点吃力。
我的目的其实和你差不多,但竞赛也要参加一点(其实是我们当地的低级竞赛),主要
是让孩子对数学有兴趣。娃费了半天劲,总要看到点实际成果才能有动力。

【在 t*******r 的大作中提到】
: 你家的情况跟我家比较像。如果你不是要去参加数学竞赛,每天大约也就半小时,我个
: 人觉得是,algebra 1 过一下,主力是拿 AMC8 => AMC10 的题目当 problem
: solving 做,按需学高年级,而不是先推高年级知识。理由如下:
: (1) math 的核心是 problem solving,高年级的知识迟早会教。
: (2) 看高年级数学冰箱书太枯燥,娃阻力不是一般的大,有想法优美但不
: implementable 的风险。
: 当然如果你要实际参加数学竞赛,那另说。

s***n
发帖数: 1280
278
我们在家里是没教过的。他当时在中文学校上7/8年级GT,也是做题讲解的形式,不是
系统学习。我一直没搞清他是怎么从题目讲解的那点内容学新知识的。我想到的一个可
能解释是美国这边的数学教学内容很多是循环式上升的。一些高中的知识点在低年级就
开始介绍引入,以后逐年深入,所以SAT Math很多内容他其实都有点底子,知道大致讲
什么东西。
我一直觉得美国这样的数学教育布局适合数学娃发展和超前学。

【在 l*****8 的大作中提到】
: SAT数学没有任何难度,但需要掌握很多知识(对小学生而言)。我猜他家娃这个根本
: 就不是正规上SAT数学,而是这些内容都学过了,然后做几套真题。对他娃这样的数学
: 能力,只要学过了内容,就不存在不会做得问题。
: 像这样的孩子估计全美国也没几个,你不知道当然不算孤陋寡闻。

t*******r
发帖数: 22634
279
我觉得她家娃的过人之处是能力超群。
至于想解出合适难度的问题,那个我觉倒得是大部分娃的天性,curiosity driven。
我这么说,是因为我记得我去接我家小娃,有过好几次小娃拼没拼过的 jigsaw puzzle
,小娃都不会让我帮忙。我要么等,要么劝小娃明天再来拼。曾经有一次,发生两个
puzzle 有混起来的未见过情况,我为了小娃能继续拼完,只能打电话让 LP 去接大娃。
当然,大部分娃不会有 syuan 家那么先进,除了父母以外,一般人也不会注意就是了。

【在 Y********d 的大作中提到】
: 赞“他喜欢独立思考,很少轻易放弃一道题。”
: 我原来以为,你因材施教,他有兴趣有能力快狠准的作题目,所以容易竞赛出成绩。
: 很高兴看到他喜欢独立思考,遇到难题不要看答案,而要好好想。
: 这个已经不仅是能力的问题,而是品位,或者说是天性的事情了。
: 竞赛到头了,也就是最快最好的作出来别人出好的题目,而这世上最重要的是找到有意
: 义的题目。

t*******r
发帖数: 22634
280
algebra 1 书里面没有排列组合,我是自己教,外加查 mathsisfun.com (我家娃
prefer 这个网站)。
你如果要参加竞赛,我就不知道咋办。对俺家而言,娃可用小时数有限的情况下,总是
要取舍的。我们板门店协议可以不参加竞赛,如果因为 friends 原因要参加俱乐部的
话,娃不用把竞赛成绩啥的带回来,路上娃可以找个阴沟把竞赛成绩单给扔了,或者逃
赛也可以 (只要别惹恼老师)。
这样就可以两耳不闻竞赛事,一心只做圣贤题。适合上不了小时数的普通娃,节省小时
数。

【在 l*****8 的大作中提到】
: 哈,真是这样。我其实每天半个小时都不可能保证的,每星期至少有3天她活动要到8点
: 才能结束。加上其它可能的比如sleep over,学校作业太多,有时候我有事,每星期也
: 就最多3天能学点东西。
: 我从来没给孩子直接看高年级数学书,就直接给她讲个概念,然后做几道练习。有时她
: 接受的比较好,有些(比如排列组合)就有点吃力。
: 我的目的其实和你差不多,但竞赛也要参加一点(其实是我们当地的低级竞赛),主要
: 是让孩子对数学有兴趣。娃费了半天劲,总要看到点实际成果才能有动力。

相关主题
请教:怎样说服孩儿他爸多关心孩子的学习?请教小孩数学教育问题
学校早晚要教的东西,早早学会了又如何?小学数学书推荐?
AMC 8 成绩 2016我来说一下天才的鉴定方法
进入Parenting版参与讨论
Y********d
发帖数: 1478
281

puzzle
娃。
了。
这个贴子信息量太大了:
小娃的curiosity,BTW,不是每个娃都有这个天性,或者在正确方面的这个天性;
老爸的对这种curiosity的保护和超级耐心;
看来你家上峰的wishful thinking 没准会变成rational expectation。
还有,你和你老婆都随身带手机。我就属于出门很少带手机的人,自由惯了。

【在 t*******r 的大作中提到】
: 我觉得她家娃的过人之处是能力超群。
: 至于想解出合适难度的问题,那个我觉倒得是大部分娃的天性,curiosity driven。
: 我这么说,是因为我记得我去接我家小娃,有过好几次小娃拼没拼过的 jigsaw puzzle
: ,小娃都不会让我帮忙。我要么等,要么劝小娃明天再来拼。曾经有一次,发生两个
: puzzle 有混起来的未见过情况,我为了小娃能继续拼完,只能打电话让 LP 去接大娃。
: 当然,大部分娃不会有 syuan 家那么先进,除了父母以外,一般人也不会注意就是了。

t*******r
发帖数: 22634
282
wishful thinking 不会变成现实,因为我老大小时候也一视同仁。只是老大的幼儿园
不标配 jigsaw puzzle 而已。但老大的 private school 时标配 chess。

【在 Y********d 的大作中提到】
:
: puzzle
: 娃。
: 了。
: 这个贴子信息量太大了:
: 小娃的curiosity,BTW,不是每个娃都有这个天性,或者在正确方面的这个天性;
: 老爸的对这种curiosity的保护和超级耐心;
: 看来你家上峰的wishful thinking 没准会变成rational expectation。
: 还有,你和你老婆都随身带手机。我就属于出门很少带手机的人,自由惯了。

l*****8
发帖数: 16949
283
嗯,这个是我猜到的。我认识的有点数学才能的数学先进娃(不是父母拿题推出来的)
,都是这个路数。他们看中小学数学书一半是看一两个例子就直奔练习了。按部就班读
教科书的都是普通娃。

【在 s***n 的大作中提到】
: 我们在家里是没教过的。他当时在中文学校上7/8年级GT,也是做题讲解的形式,不是
: 系统学习。我一直没搞清他是怎么从题目讲解的那点内容学新知识的。我想到的一个可
: 能解释是美国这边的数学教学内容很多是循环式上升的。一些高中的知识点在低年级就
: 开始介绍引入,以后逐年深入,所以SAT Math很多内容他其实都有点底子,知道大致讲
: 什么东西。
: 我一直觉得美国这样的数学教育布局适合数学娃发展和超前学。

Y********d
发帖数: 1478
284
我觉得人的一辈子都应该多少有点 wishful thinking,尤其在养娃这个事情上。
说的好听点叫做愿景,说的难听点叫做白日梦。
白日梦做多了的人,我常劝他喝杯冰水。
愿景太少了的人,我常劝他回家,打开电视机,看会儿韩剧。

【在 t*******r 的大作中提到】
: wishful thinking 不会变成现实,因为我老大小时候也一视同仁。只是老大的幼儿园
: 不标配 jigsaw puzzle 而已。但老大的 private school 时标配 chess。

P******e
发帖数: 1325
285
这第三点像是奔着数学家的路子去了。就像学音乐舞蹈一不小心学成了专业的一样。
你会推你娃走这条路么?很穷的。

★ 发自iPhone App: ChineseWeb 1.0.2

【在 s***n 的大作中提到】
: 你对我儿子倒是过誉了。他从小对数学有兴趣,抽象思维形成的早,逻辑思维发展的早
: ,所以他跟很多高年级数学娃一样,对抽象的数学感兴趣。你如果去过数学俱乐部,跟
: 数学娃打过交道的话,就不会觉得不问为什么要学数学有什么大不了。
: 一个人容易对自己优势的项目产生和保持兴趣,这是很正常的心理。足球好的参加足球
: 比赛,数学好的参加数学比赛,顺理成章。倒是如果我逼着我儿子整天训练去参加美国
: 高中足球联赛,他肯定会我为什么要参加这个比赛...
: 第三点我儿子确实做得比我好。他喜欢独立思考,很少轻易放弃一道题。这个对竞赛培
: 训是不利的,但对长远发展应该是有利的。竞赛培训希望娃短时间内学会做某种类型的
: 题,孩子一时做不出来就会直接教了。今年准备AIME的时候我们就碰到这问题。我给他
: 报了AOPS的AIME网课,但课上和作业里他自己做不出来的题他不愿意直接看答案学解法

t*******r
发帖数: 22634
286
数学教授不见得穷吧,中产阶级还是钢钢的是不是?
当然天才家的期望值,跟混州大的可能也不一样就是了。

【在 P******e 的大作中提到】
: 这第三点像是奔着数学家的路子去了。就像学音乐舞蹈一不小心学成了专业的一样。
: 你会推你娃走这条路么?很穷的。
:
: ★ 发自iPhone App: ChineseWeb 1.0.2

l*****8
发帖数: 16949
287
是这个理。有这个能力,做自己喜欢的事,未必见得比做自己不喜欢的事多挣几个钱差
。而且在名校学数学的真想追钱的话,去华尔街易如反掌。哈佛的一个经济学教授年轻
时就是个数学天才,现在管理哈佛基金,每年收入好几百万,教课只是他的爱好。当然
也有选择一直搞纯数学的,因为他们更爱好做这样的工作。

【在 t*******r 的大作中提到】
: 数学教授不见得穷吧,中产阶级还是钢钢的是不是?
: 当然天才家的期望值,跟混州大的可能也不一样就是了。

Y********d
发帖数: 1478
288
是的。
其实能力强了,选择还是多的,这个时候就看自己追求什么了。

【在 l*****8 的大作中提到】
: 是这个理。有这个能力,做自己喜欢的事,未必见得比做自己不喜欢的事多挣几个钱差
: 。而且在名校学数学的真想追钱的话,去华尔街易如反掌。哈佛的一个经济学教授年轻
: 时就是个数学天才,现在管理哈佛基金,每年收入好几百万,教课只是他的爱好。当然
: 也有选择一直搞纯数学的,因为他们更爱好做这样的工作。

n********h
发帖数: 13135
289
开个奥赛培训班都挣不少,关键是擅长而且喜欢。

【在 l*****8 的大作中提到】
: 是这个理。有这个能力,做自己喜欢的事,未必见得比做自己不喜欢的事多挣几个钱差
: 。而且在名校学数学的真想追钱的话,去华尔街易如反掌。哈佛的一个经济学教授年轻
: 时就是个数学天才,现在管理哈佛基金,每年收入好几百万,教课只是他的爱好。当然
: 也有选择一直搞纯数学的,因为他们更爱好做这样的工作。

t*******r
发帖数: 22634
290
我不知道 syuan 家娃如何从做题中学习。但我基本知道我家娃。当然俺家普通娃轨迹
显然不会一样,也不能跟天才娃比。
我觉得我家普通娃学 AMC8,我觉得首先靠 reading 和 analytical thinking,可以在
大部分情况下读懂 AMC8 答案,或者 mathsisfun.com 网页,因此不需要我太多干预。
当然学校 math 的 spiral 也有用,但我觉得学校 math 的 spiral 主要还是给 math
reading 提供 schema (prior knowledge)。
这个主要还是我家普通娃轨迹不太一样。我家娃一直是 reading 优先于 math。我在娃
三年级的时候,主力方向是搞 library non-fiction reading,(三年级数学加料的小
时数是零)。而且 non-fiction library reading 跟 math 不一样,reading 天然是非
常有趣的,easily 可以做到一天三小时,一年半就 2000 小时上去了。另外娃四年级
MEOMS 阶段加料了 math analytical thinking,我觉得对读懂 AMC8 的答案,以及读
懂 mathsisfun.com,应该帮助不小。
当然俺家普通娃智商不够,没法跟天才娃比。但从父母的角度而言,毕竟智商是 “成
事在天” 的事,父母把 “谋事在人” 做好就可以了。

【在 l*****8 的大作中提到】
: 嗯,这个是我猜到的。我认识的有点数学才能的数学先进娃(不是父母拿题推出来的)
: ,都是这个路数。他们看中小学数学书一半是看一两个例子就直奔练习了。按部就班读
: 教科书的都是普通娃。

相关主题
[华盛顿邮报] 9 - 14 岁是数学兴趣的关键时期?数学教育 一家之言 番外篇
怎么推小学数学好?小学一年级的孩子。小学升初中,儿子数学跳级的一点经验
请问,AMC 8应该从几年级开始练习呀?有明天考AMC 8的吗?
进入Parenting版参与讨论
x***1
发帖数: 999
291
谢谢分享。

math


【在 t*******r 的大作中提到】
: 我不知道 syuan 家娃如何从做题中学习。但我基本知道我家娃。当然俺家普通娃轨迹
: 显然不会一样,也不能跟天才娃比。
: 我觉得我家普通娃学 AMC8,我觉得首先靠 reading 和 analytical thinking,可以在
: 大部分情况下读懂 AMC8 答案,或者 mathsisfun.com 网页,因此不需要我太多干预。
: 当然学校 math 的 spiral 也有用,但我觉得学校 math 的 spiral 主要还是给 math
: reading 提供 schema (prior knowledge)。
: 这个主要还是我家普通娃轨迹不太一样。我家娃一直是 reading 优先于 math。我在娃
: 三年级的时候,主力方向是搞 library non-fiction reading,(三年级数学加料的小
: 时数是零)。而且 non-fiction library reading 跟 math 不一样,reading 天然是非
: 常有趣的,easily 可以做到一天三小时,一年半就 2000 小时上去了。另外娃四年级

t*******r
发帖数: 22634
292
我觉得我家普通娃,数学方面的主要长期动力是来自于 science。娃知道 math 是
science 的 foundation toolkit,并且娃也知道到高中才有真正的 science 的出现。
而对 science 兴趣很容易建立,人类对 mother nature 的 curiosity。
虽然那个来自 science 动力并不强,但比较稳定不受风雨影响。所以我觉得我家娃不
那么需要把数学竞赛作为学数学的推动力之一。另外个性而言,大方向上我家女娃也不
那么争强好胜。

【在 l*****8 的大作中提到】
: 哈,真是这样。我其实每天半个小时都不可能保证的,每星期至少有3天她活动要到8点
: 才能结束。加上其它可能的比如sleep over,学校作业太多,有时候我有事,每星期也
: 就最多3天能学点东西。
: 我从来没给孩子直接看高年级数学书,就直接给她讲个概念,然后做几道练习。有时她
: 接受的比较好,有些(比如排列组合)就有点吃力。
: 我的目的其实和你差不多,但竞赛也要参加一点(其实是我们当地的低级竞赛),主要
: 是让孩子对数学有兴趣。娃费了半天劲,总要看到点实际成果才能有动力。

J***A
发帖数: 1511
293
数学好好像可以做商学院的教授,比如accounting的教授, 应该是教授里收入最高的
,据说是ap起薪都15-20万加的。经济系的教授薪水也高,也是很多需要很高的数学功
底, 纯数学系的教授,做得好的, 一样高薪。 何况教授还可以做咨询搞副业开公司
卖专利什么的。 当然我这只是见过猪跑的传言, 具体还以logic说的为准:)

这第三点像是奔着数学家的路子去了。就像学音乐舞蹈一不小心学成了专业的一样。你
会推你娃走这条路么?很穷的。

【在 P******e 的大作中提到】
: 这第三点像是奔着数学家的路子去了。就像学音乐舞蹈一不小心学成了专业的一样。
: 你会推你娃走这条路么?很穷的。
:
: ★ 发自iPhone App: ChineseWeb 1.0.2

d**********h
发帖数: 2795
294
嗯,这个我估计事先私下里推过数学,美帝公立学校三年级数学对上amc8就是渣
实力有了,只是第一次拿amc8练手
还有是选择题,总能凭直觉做对一个半个的

【在 t*******r 的大作中提到】
: 三年级第一次做 AMC8 就能做对 19 题,说实话我觉得全美都不会有几个人。
: 大部分人五年级就算能第一次做,一般在 16 到 20 题之间多半直接卡住,得一路
: skip 下去。

j*p
发帖数: 780
295
对我们来说这个小时数是关键,课外活动数学竞赛的priority排在游泳篮球和
Orchestra后面。
每家娃情况不同,我们活动多时间分散,还没打算减短运动时间增加每天的数学时间。
谢谢syuan分享经验,很受启发。娃也受到鼓励,今年学习mathcounts应该有劲头。希
望能state胜出,cheers!。
p.s.下午娃try out travel篮球队,祝他好运。:-)

【在 t*******r 的大作中提到】
: 我觉得你错了。起步早不是关键,每天一小时不是关键。3000 小时从 K 班数学跑步进
: 入 AIME 的速度深度才是关键。
: 或者我这么说,如果拥有能 3000 小时从 K版数学跑步进入 AIME 的能力的话,那说实
: 话,六年级数学一屁股红叉也无所谓。那就每天两小时,八年级跑步进入 AIME/AMO 不
: 就完事了?

l*****8
发帖数: 16949
296
人家2年级就学了5/6年级的GT math, 3年级学了7/8年级,学完了广度已经在那儿了,
AMC8该知道的内容就已经知道了。就是还没做竞赛题而已。

【在 d**********h 的大作中提到】
: 嗯,这个我估计事先私下里推过数学,美帝公立学校三年级数学对上amc8就是渣
: 实力有了,只是第一次拿amc8练手
: 还有是选择题,总能凭直觉做对一个半个的

l*****8
发帖数: 16949
297
可能你是谦虚,不过你儿子不愿意放弃一道题正是搞数学竞赛需要的。当然这个对AMC
这个级别的题可能不适合(不过你儿显然已经过了这个阶段了),国际奥林匹克,平均
一道题一个半小时,就是需要一直啃下去的。而且一直不放弃实际上对搞数学是非常非
常重要的。他在思考过程中肯定尝试了很多走不通的路,这些路他仔仔细细想过了,下
次遇到一条这样走走得通的题,可能一下就通过了。而且如果最后解决了,那是一辈子
都忘不了的,这个和看来的答案完全不可同日而语。你孩子绝对是搞数学的天才。

【在 s***n 的大作中提到】
: 你对我儿子倒是过誉了。他从小对数学有兴趣,抽象思维形成的早,逻辑思维发展的早
: ,所以他跟很多高年级数学娃一样,对抽象的数学感兴趣。你如果去过数学俱乐部,跟
: 数学娃打过交道的话,就不会觉得不问为什么要学数学有什么大不了。
: 一个人容易对自己优势的项目产生和保持兴趣,这是很正常的心理。足球好的参加足球
: 比赛,数学好的参加数学比赛,顺理成章。倒是如果我逼着我儿子整天训练去参加美国
: 高中足球联赛,他肯定会我为什么要参加这个比赛...
: 第三点我儿子确实做得比我好。他喜欢独立思考,很少轻易放弃一道题。这个对竞赛培
: 训是不利的,但对长远发展应该是有利的。竞赛培训希望娃短时间内学会做某种类型的
: 题,孩子一时做不出来就会直接教了。今年准备AIME的时候我们就碰到这问题。我给他
: 报了AOPS的AIME网课,但课上和作业里他自己做不出来的题他不愿意直接看答案学解法

l*****8
发帖数: 16949
298
这个其实对数学天赋好的娃都不是问题。基本先来个概念,然后做一两道题验证一下概
念就行了。触类旁通对他们都是很简单的事。所以学起来其实不需要花多少时间。当然
普通娃这样搞会基础太差。
不过对我们大家有用的还是如何推扑通蛙。你的这些思考都很有启发。

math


【在 t*******r 的大作中提到】
: 我不知道 syuan 家娃如何从做题中学习。但我基本知道我家娃。当然俺家普通娃轨迹
: 显然不会一样,也不能跟天才娃比。
: 我觉得我家普通娃学 AMC8,我觉得首先靠 reading 和 analytical thinking,可以在
: 大部分情况下读懂 AMC8 答案,或者 mathsisfun.com 网页,因此不需要我太多干预。
: 当然学校 math 的 spiral 也有用,但我觉得学校 math 的 spiral 主要还是给 math
: reading 提供 schema (prior knowledge)。
: 这个主要还是我家普通娃轨迹不太一样。我家娃一直是 reading 优先于 math。我在娃
: 三年级的时候,主力方向是搞 library non-fiction reading,(三年级数学加料的小
: 时数是零)。而且 non-fiction library reading 跟 math 不一样,reading 天然是非
: 常有趣的,easily 可以做到一天三小时,一年半就 2000 小时上去了。另外娃四年级

l*****8
发帖数: 16949
299
大学教授的工资基本和社会上这个行业普通人的薪水成比例的。因为你给低了就没有
qualified人来当教授了。比如医法商学院教授的工资最高,文科的教授工资最低。计
算机的工资比其它stem的就高。数学教授确实属于STEM里工资比较低的,但比文科教授
高。另外也和学校,地区有关。做到好学校的教授工资就不低了,另外如果搞应用数学
,在经济系任职,或者给金融公司做顾问,收入就不低了。

【在 J***A 的大作中提到】
: 数学好好像可以做商学院的教授,比如accounting的教授, 应该是教授里收入最高的
: ,据说是ap起薪都15-20万加的。经济系的教授薪水也高,也是很多需要很高的数学功
: 底, 纯数学系的教授,做得好的, 一样高薪。 何况教授还可以做咨询搞副业开公司
: 卖专利什么的。 当然我这只是见过猪跑的传言, 具体还以logic说的为准:)
:
: 这第三点像是奔着数学家的路子去了。就像学音乐舞蹈一不小心学成了专业的一样。你
: 会推你娃走这条路么?很穷的。

t*******r
发帖数: 22634
300
我觉得这个讨论区分了两类在小学初中阶段很容易混淆的娃,math 娃 vs science 娃。
之所以不容易区分,是因为小学初中没有真正的 science,真正的 science 要到高中
物理/化学开始。当然现在还有信息学了。
其实我猜在小学数学俱乐部里,真正的 math 娃可能很少,大部分其实是 science 娃
在那里装蒜。
这个区分的指标不是水平高 vs 低,而是 trait 和偏好。我觉得,math 娃才是真正喜
欢 math,science 娃本质上把 math 是当成 tool。当然 tool 本身也可以是玩具。
我觉得我前面的那些策略,更适合 science 娃。而相对而言,science 娃在 math 方
面也稍稍落后一些。这些娃用 non-fiction reading 帮个忙,也是一种不错的战略。

【在 l*****8 的大作中提到】
: 这个其实对数学天赋好的娃都不是问题。基本先来个概念,然后做一两道题验证一下概
: 念就行了。触类旁通对他们都是很简单的事。所以学起来其实不需要花多少时间。当然
: 普通娃这样搞会基础太差。
: 不过对我们大家有用的还是如何推扑通蛙。你的这些思考都很有启发。
:
: math
: 级

相关主题
求科普: 奥数 vs. Math Olympiad vs. Math Count vs. Math circle vs. 超前学数学[葩论] 论“有限小灶策略”
9岁, 如何报名考amc10给初中学生家长的一些建议 (转载)
算不算有数学天分小学数学上超前班有什么好处吗?
进入Parenting版参与讨论
t*******r
发帖数: 22634
301
另外我声明一下这个不是开玩笑。对智商非常高的娃,800 小时从四年级数学跑步进入
AIME 是完全可能的,具体小时数估算如下,比如:
150 小时 AMC8
250 小时 AMC10
250 小时 AMC12
基本全真题就可以刷一遍
当然前提是娃的智商非常高,普通娃就算了。

【在 t*******r 的大作中提到】
: 我重新估算了一下,800 小时从九九乘法表跑步进入 AIME。
: 版上从此不用再争论九九乘法表了吧。

t*******r
发帖数: 22634
302
从这个角度看,初中数学班课内要求 explain experimental probability vs
theoretical probability,完全是合理应该并且很重要的。因为班里大部分 kids 最
终不是走理论数学的道路。而初中阶段的 science 课,没有用 math 建模的概念。
从这个角度看,娃课上的 explain experimental probability vs theoretical
probability,应该更多的从 physics / computer-science 建模的角度去看,而不是
钻 theoretical mathematics 的牛角尖。
看来再给娃解释一下 sense of purpose,然后加强一下为现实建模的思想,这个概念
窟窿就可以补掉它了。今年下午干掉 it。

娃。

【在 t*******r 的大作中提到】
: 我觉得这个讨论区分了两类在小学初中阶段很容易混淆的娃,math 娃 vs science 娃。
: 之所以不容易区分,是因为小学初中没有真正的 science,真正的 science 要到高中
: 物理/化学开始。当然现在还有信息学了。
: 其实我猜在小学数学俱乐部里,真正的 math 娃可能很少,大部分其实是 science 娃
: 在那里装蒜。
: 这个区分的指标不是水平高 vs 低,而是 trait 和偏好。我觉得,math 娃才是真正喜
: 欢 math,science 娃本质上把 math 是当成 tool。当然 tool 本身也可以是玩具。
: 我觉得我前面的那些策略,更适合 science 娃。而相对而言,science 娃在 math 方
: 面也稍稍落后一些。这些娃用 non-fiction reading 帮个忙,也是一种不错的战略。

P******e
发帖数: 1325
303
这个说的靠谱。金牌之后还是法商医吧。
花街大概是金牌密度最大的地方了。用智商换钱,这是最快的途径。
当然了,也可以谈谈诗歌和远方。有一些死心眼的数学娃,认定只有纯数高大上,
数学的其他分支还看不上,然后top school phd出来,找postdoc都吃力,
更不用说好学校教授了。

★ 发自iPhone App: ChineseWeb 1.0.2
★ 发自iPhone App: ChineseWeb 1.0.2

【在 l*****8 的大作中提到】
: 大学教授的工资基本和社会上这个行业普通人的薪水成比例的。因为你给低了就没有
: qualified人来当教授了。比如医法商学院教授的工资最高,文科的教授工资最低。计
: 算机的工资比其它stem的就高。数学教授确实属于STEM里工资比较低的,但比文科教授
: 高。另外也和学校,地区有关。做到好学校的教授工资就不低了,另外如果搞应用数学
: ,在经济系任职,或者给金融公司做顾问,收入就不低了。

s***n
发帖数: 1280
304
搞数学或物理的去学法律其实有逻辑思维优势。过去一二十年法学院入学考试LSAT平均分
最高的本科专业就是物理和数学。

【在 P******e 的大作中提到】
: 这个说的靠谱。金牌之后还是法商医吧。
: 花街大概是金牌密度最大的地方了。用智商换钱,这是最快的途径。
: 当然了,也可以谈谈诗歌和远方。有一些死心眼的数学娃,认定只有纯数高大上,
: 数学的其他分支还看不上,然后top school phd出来,找postdoc都吃力,
: 更不用说好学校教授了。
:
: ★ 发自iPhone App: ChineseWeb 1.0.2
: ★ 发自iPhone App: ChineseWeb 1.0.2

s***n
发帖数: 1280
305
我身边的理工聪明娃中,我大致能看得出偏理论偏数学的娃和偏science偏具体应用的
娃。不过我觉得没必要这么早区分数学娃和科学娃,把方向定死。不要说这两者都基于
逻辑思维,联系紧密,而且孩子智力还在发展中,兴趣也可能转移。
我儿子应该算是数学娃。我现在在慢慢地把他往Science/物理方向引,买了些物理方面
的科普读物,也鼓励他去Youtube上看Crash Course (他喜欢天文类的)。结果几天前,
他拿了我给他买了半年的那本卡通物理书主动找我让我给他讲物理 - 终于上钩了...
我其实不准备初中三年高中四年一门心思数学竞赛,我的计划是初中的时候推点数学,
推点物理,推点计算机,还有时间推点机器人,然后高中就由他自己决定走哪条路在哪
方面加小时数。

娃。

【在 t*******r 的大作中提到】
: 我觉得这个讨论区分了两类在小学初中阶段很容易混淆的娃,math 娃 vs science 娃。
: 之所以不容易区分,是因为小学初中没有真正的 science,真正的 science 要到高中
: 物理/化学开始。当然现在还有信息学了。
: 其实我猜在小学数学俱乐部里,真正的 math 娃可能很少,大部分其实是 science 娃
: 在那里装蒜。
: 这个区分的指标不是水平高 vs 低,而是 trait 和偏好。我觉得,math 娃才是真正喜
: 欢 math,science 娃本质上把 math 是当成 tool。当然 tool 本身也可以是玩具。
: 我觉得我前面的那些策略,更适合 science 娃。而相对而言,science 娃在 math 方
: 面也稍稍落后一些。这些娃用 non-fiction reading 帮个忙,也是一种不错的战略。

t*******r
发帖数: 22634
306
其实我觉得我没见过真正的 math 娃。
但对 science 娃,对父母而言,这个都不用去刻意区分。我这么说,我家大娃三年级
刚刚会 read-to-learn,刚刚开始 library non-fiction reading 的时候,我目测她
在娃版 medical book 上,三年级一年就扔了差不多有 1000 小时,而且对每个不同领
域都有她喜欢/不喜欢的 preference。
而对于 math,天地良心啊,我在娃三年级时用 cosmology 打埋伏,然后四年级 MOEMS
期间把所有基本概念都投射到 space-time 来 interpret / understand,再到五年级
上 AMC8,总共三年,连 cosmology 加料时间一起算进去,我觉得特么都超不过 500
小时,真正做题时间可能也就 100 小时多点。
这差距不是一般的大。
当然我痛定思痛,觉得这强扭的瓜不会甜。数学还是保证够用就行了,每天半小时多点
就差不多了。我将来还是顺着娃的兴趣,继续琢磨如何 science 上如何加点料,熬过
初中阶段就好。

【在 s***n 的大作中提到】
: 我身边的理工聪明娃中,我大致能看得出偏理论偏数学的娃和偏science偏具体应用的
: 娃。不过我觉得没必要这么早区分数学娃和科学娃,把方向定死。不要说这两者都基于
: 逻辑思维,联系紧密,而且孩子智力还在发展中,兴趣也可能转移。
: 我儿子应该算是数学娃。我现在在慢慢地把他往Science/物理方向引,买了些物理方面
: 的科普读物,也鼓励他去Youtube上看Crash Course (他喜欢天文类的)。结果几天前,
: 他拿了我给他买了半年的那本卡通物理书主动找我让我给他讲物理 - 终于上钩了...
: 我其实不准备初中三年高中四年一门心思数学竞赛,我的计划是初中的时候推点数学,
: 推点物理,推点计算机,还有时间推点机器人,然后高中就由他自己决定走哪条路在哪
: 方面加小时数。
:

t*******r
发帖数: 22634
307
其实根本不用等到小学三年级,我家 K 班的小娃,大字一个不认,但号称她最喜欢的
book 是 Discovery Kids 的 Encyclopedia of everything。
当然好处是,这也增加她学 K 班认字儿歌课的动力。

MOEMS

【在 t*******r 的大作中提到】
: 其实我觉得我没见过真正的 math 娃。
: 但对 science 娃,对父母而言,这个都不用去刻意区分。我这么说,我家大娃三年级
: 刚刚会 read-to-learn,刚刚开始 library non-fiction reading 的时候,我目测她
: 在娃版 medical book 上,三年级一年就扔了差不多有 1000 小时,而且对每个不同领
: 域都有她喜欢/不喜欢的 preference。
: 而对于 math,天地良心啊,我在娃三年级时用 cosmology 打埋伏,然后四年级 MOEMS
: 期间把所有基本概念都投射到 space-time 来 interpret / understand,再到五年级
: 上 AMC8,总共三年,连 cosmology 加料时间一起算进去,我觉得特么都超不过 500
: 小时,真正做题时间可能也就 100 小时多点。
: 这差距不是一般的大。

Y********d
发帖数: 1478
308

MOEMS
你家大娃的例子生动的说明,interest is the best inner drive.

【在 t*******r 的大作中提到】
: 其实我觉得我没见过真正的 math 娃。
: 但对 science 娃,对父母而言,这个都不用去刻意区分。我这么说,我家大娃三年级
: 刚刚会 read-to-learn,刚刚开始 library non-fiction reading 的时候,我目测她
: 在娃版 medical book 上,三年级一年就扔了差不多有 1000 小时,而且对每个不同领
: 域都有她喜欢/不喜欢的 preference。
: 而对于 math,天地良心啊,我在娃三年级时用 cosmology 打埋伏,然后四年级 MOEMS
: 期间把所有基本概念都投射到 space-time 来 interpret / understand,再到五年级
: 上 AMC8,总共三年,连 cosmology 加料时间一起算进去,我觉得特么都超不过 500
: 小时,真正做题时间可能也就 100 小时多点。
: 这差距不是一般的大。

y*******g
发帖数: 1395
309
怎么没有,我那个年代的都有math娃和物理娃的
高一以后,就非常明显了,到高二就定位了。。。

【在 t*******r 的大作中提到】
: 其实我觉得我没见过真正的 math 娃。
: 但对 science 娃,对父母而言,这个都不用去刻意区分。我这么说,我家大娃三年级
: 刚刚会 read-to-learn,刚刚开始 library non-fiction reading 的时候,我目测她
: 在娃版 medical book 上,三年级一年就扔了差不多有 1000 小时,而且对每个不同领
: 域都有她喜欢/不喜欢的 preference。
: 而对于 math,天地良心啊,我在娃三年级时用 cosmology 打埋伏,然后四年级 MOEMS
: 期间把所有基本概念都投射到 space-time 来 interpret / understand,再到五年级
: 上 AMC8,总共三年,连 cosmology 加料时间一起算进去,我觉得特么都超不过 500
: 小时,真正做题时间可能也就 100 小时多点。
: 这差距不是一般的大。

Y********d
发帖数: 1478
310


你家小娃的例子让我想到:Where does interest come from?
我觉得至少有三种可能让她觉得“最喜欢的book是Encyclopedia of everything”。
1。你们提供了从芭比娃娃到Discovery Kids的选择全集,她自己选了后者;
2。你们只提供了Discovery Kids,她从中选择了Encyclopedia of everything;
3。你们什么都没管,她接触家里,幼儿园里,社会里各种选择后慢慢喜欢上
Encyclopedia of everything,你们再进一步提供更多的Encyclopedia of everything。

【在 t*******r 的大作中提到】
: 其实根本不用等到小学三年级,我家 K 班的小娃,大字一个不认,但号称她最喜欢的
: book 是 Discovery Kids 的 Encyclopedia of everything。
: 当然好处是,这也增加她学 K 班认字儿歌课的动力。
:
: MOEMS

相关主题
小学数学上超前班有什么好处吗?学校早晚要教的东西,早早学会了又如何?
小学数学有比赛项目吗AMC 8 成绩 2016
请教:怎样说服孩儿他爸多关心孩子的学习?请教小孩数学教育问题
进入Parenting版参与讨论
t*******r
发帖数: 22634
311
好吧,我承认我在小娃小时候,主力是提供 PBS Kids Education Video,当然不同节
目的覆盖面很全,我觉得 comprehensive。
玩具的话,小娃还是有很多 Pony 的。

everything。

【在 Y********d 的大作中提到】
:
: 的
: 你家小娃的例子让我想到:Where does interest come from?
: 我觉得至少有三种可能让她觉得“最喜欢的book是Encyclopedia of everything”。
: 1。你们提供了从芭比娃娃到Discovery Kids的选择全集,她自己选了后者;
: 2。你们只提供了Discovery Kids,她从中选择了Encyclopedia of everything;
: 3。你们什么都没管,她接触家里,幼儿园里,社会里各种选择后慢慢喜欢上
: Encyclopedia of everything,你们再进一步提供更多的Encyclopedia of everything。

d**********h
发帖数: 2795
312
这也不错,推数学,得科学家
其实,数学,科学早年不分家,后来细化,分了家,而且越分越细
对孩子讲授这些,当然应该搅和在一起讲。第一个学一加一的孩子又不用手指头之类实
物的吗?第一个学乘法概念的时候有不联系加法的吗?(画一堆行列)
抽象和实体本来就不是绝对分开的,绝对分开,则没有一个人可以理解概念。

MOEMS

【在 t*******r 的大作中提到】
: 其实我觉得我没见过真正的 math 娃。
: 但对 science 娃,对父母而言,这个都不用去刻意区分。我这么说,我家大娃三年级
: 刚刚会 read-to-learn,刚刚开始 library non-fiction reading 的时候,我目测她
: 在娃版 medical book 上,三年级一年就扔了差不多有 1000 小时,而且对每个不同领
: 域都有她喜欢/不喜欢的 preference。
: 而对于 math,天地良心啊,我在娃三年级时用 cosmology 打埋伏,然后四年级 MOEMS
: 期间把所有基本概念都投射到 space-time 来 interpret / understand,再到五年级
: 上 AMC8,总共三年,连 cosmology 加料时间一起算进去,我觉得特么都超不过 500
: 小时,真正做题时间可能也就 100 小时多点。
: 这差距不是一般的大。

Y********d
发帖数: 1478
313

看来是全面提供,重点突出。所以父母的偏好对娃的偏好形成还是有不可磨灭的影响的
。想想其实也是fair的。因为养娃就是在parentalism 和altruism之间的平衡。
顺便mark一下PBS Kids Education Video。最近每天半小时巧虎的故事,行为礼貌学得
不错,什么时候也给她看点科普video开开兴趣。

【在 t*******r 的大作中提到】
: 好吧,我承认我在小娃小时候,主力是提供 PBS Kids Education Video,当然不同节
: 目的覆盖面很全,我觉得 comprehensive。
: 玩具的话,小娃还是有很多 Pony 的。
:
: everything。

Y********d
发帖数: 1478
314

我印象中,有“数学是自然科学的哲学”一说?
这个大概也是版上的父母都热衷于数学的原因吧。

【在 d**********h 的大作中提到】
: 这也不错,推数学,得科学家
: 其实,数学,科学早年不分家,后来细化,分了家,而且越分越细
: 对孩子讲授这些,当然应该搅和在一起讲。第一个学一加一的孩子又不用手指头之类实
: 物的吗?第一个学乘法概念的时候有不联系加法的吗?(画一堆行列)
: 抽象和实体本来就不是绝对分开的,绝对分开,则没有一个人可以理解概念。
:
: MOEMS

d**********h
发帖数: 2795
315
不知道 :)
据我所知,哲学是研究形而上的东东
确实,数学和自然科学完全不是一回事儿,只是可以互相利用而有交集。尤其有了纯数
学和应用数学之分之后 。
咱不是数学专业,道听途说居多

【在 Y********d 的大作中提到】
:
: 我印象中,有“数学是自然科学的哲学”一说?
: 这个大概也是版上的父母都热衷于数学的原因吧。

t*******r
发帖数: 22634
316
热衷数学是因为数学是最先分快/慢车道的,也是 STEM 的基础。跟是不是哲学关系不
大。
另外小学娃的哲学思想,在 academic 方面,主要是来自娃版图书馆泛读,read-to-
learn,跟小学数学关系不大。

【在 Y********d 的大作中提到】
:
: 我印象中,有“数学是自然科学的哲学”一说?
: 这个大概也是版上的父母都热衷于数学的原因吧。

J***A
发帖数: 1511
317
对你说这个有同感。
理化都要以数学为基础,化学再需要点背反应方程式的苦功,物理到很多时候觉得在用
物理定律解数学应用题。所以数学好的人早年表现多半是数理化都好。

这也不错,推数学,得科学家其实,数学,科学早年不分家,后来细化,分了家,而且
越分越细对孩子讲授这些,当然应该搅和在一起讲。第一个学一加一的孩子又不用手指
头之类实物的吗?第一个学........

【在 d**********h 的大作中提到】
: 这也不错,推数学,得科学家
: 其实,数学,科学早年不分家,后来细化,分了家,而且越分越细
: 对孩子讲授这些,当然应该搅和在一起讲。第一个学一加一的孩子又不用手指头之类实
: 物的吗?第一个学乘法概念的时候有不联系加法的吗?(画一堆行列)
: 抽象和实体本来就不是绝对分开的,绝对分开,则没有一个人可以理解概念。
:
: MOEMS

t*******r
发帖数: 22634
318
为啥不是反过来?数理化计算机好的,表现为数学好,因为理化计算机还没开课。

【在 J***A 的大作中提到】
: 对你说这个有同感。
: 理化都要以数学为基础,化学再需要点背反应方程式的苦功,物理到很多时候觉得在用
: 物理定律解数学应用题。所以数学好的人早年表现多半是数理化都好。
:
: 这也不错,推数学,得科学家其实,数学,科学早年不分家,后来细化,分了家,而且
: 越分越细对孩子讲授这些,当然应该搅和在一起讲。第一个学一加一的孩子又不用手指
: 头之类实物的吗?第一个学........

J***A
发帖数: 1511
319
化学高中以后没接触了, 觉得到高中为止就是背几个反映方程加点数学n元一次方程?
大学时学过二类的数学物理方法, 深感各种物理问题都得靠数学啊!量子力学薛定谔
偏微分方程就不说了, 电子学里信号处理 傅立叶变换, 电磁场的麦克斯韦方程组..
.... 好像力热光电一切物理的定理最后的归宿都是数学模型? 物理牛人光数学好肯定
还不够, 但数学不好恐怕万万不行。 我记得有谣传说牛顿的万有引力基础概念是胡克
先提的, 可惜胡克数学不够好, 所以没推到牛顿那一步?
计算机大学就修过一门课, 现在就是处理数据时编编简单的不能称之为程序的程序...
... 就不讨论了:(
难道因为数学教得早, 之后理化计算机用到的, 都被我自动定义成数学知识?
不过我还是觉得数学是理化的基础,理化和计算机都是纯数学加上各学科自己的东西..
....

为啥不是反过来?数理化计算机好的,表现为数学好,因为理化计算机还没开课。

【在 t*******r 的大作中提到】
: 为啥不是反过来?数理化计算机好的,表现为数学好,因为理化计算机还没开课。
k****r
发帖数: 851
320
需要这么费心思啊,我要绝望了.
为啥我们小时候没这么多事?是不是我们那时候老师本来作业就布置的多?教的深? 美帝
坑我坑一辈子了...

MOEMS

【在 t*******r 的大作中提到】
: 其实我觉得我没见过真正的 math 娃。
: 但对 science 娃,对父母而言,这个都不用去刻意区分。我这么说,我家大娃三年级
: 刚刚会 read-to-learn,刚刚开始 library non-fiction reading 的时候,我目测她
: 在娃版 medical book 上,三年级一年就扔了差不多有 1000 小时,而且对每个不同领
: 域都有她喜欢/不喜欢的 preference。
: 而对于 math,天地良心啊,我在娃三年级时用 cosmology 打埋伏,然后四年级 MOEMS
: 期间把所有基本概念都投射到 space-time 来 interpret / understand,再到五年级
: 上 AMC8,总共三年,连 cosmology 加料时间一起算进去,我觉得特么都超不过 500
: 小时,真正做题时间可能也就 100 小时多点。
: 这差距不是一般的大。

相关主题
小学数学书推荐?怎么推小学数学好?小学一年级的孩子。
我来说一下天才的鉴定方法请问,AMC 8应该从几年级开始练习呀?
[华盛顿邮报] 9 - 14 岁是数学兴趣的关键时期?数学教育 一家之言 番外篇
进入Parenting版参与讨论
J***A
发帖数: 1511
321
还得补充一点, 现在搞点什么都非得跟大数据扯上关系, 连纯工科都不能幸免.....
.现在是学好统计和计算机, 走遍天下都不怕:)

为啥不是反过来?数理化计算机好的,表现为数学好,因为理化计算机还没开课。

【在 t*******r 的大作中提到】
: 为啥不是反过来?数理化计算机好的,表现为数学好,因为理化计算机还没开课。
t*******r
发帖数: 22634
322
实际上我觉得是美帝系统的相对效率高。
我觉得我小时候做题量多半有我娃的三倍还多,但以成果论英雄的角度,以目前 6 年
级看,我觉得我当年初一最多也就跟我娃现在差不多,除了计算能力强一点。
我知道大伙儿一般都推崇我们当年的国内数学,很鄙视美国数学。但那是跟学校数学慢
道比。如果去跟 AMC 8 / 10 的大众型快道比,我觉得我们当年也未必强多少,最大的
强项可能就是个熟练程度我觉得。
将来不知道,但我目测将来如果有差别,更多的可能是 nature 的差别。

【在 k****r 的大作中提到】
: 需要这么费心思啊,我要绝望了.
: 为啥我们小时候没这么多事?是不是我们那时候老师本来作业就布置的多?教的深? 美帝
: 坑我坑一辈子了...
:
: MOEMS

t*******r
发帖数: 22634
323
电算了,你说的这些都不用肉算了。除非你读理论物理,不过现在理论物理也是电算为
主。
就有点像蒙古海军骑兵琢磨为啥 USMC 连骑马都不会。话糙理不糙!


..
..
..

【在 J***A 的大作中提到】
: 化学高中以后没接触了, 觉得到高中为止就是背几个反映方程加点数学n元一次方程?
: 大学时学过二类的数学物理方法, 深感各种物理问题都得靠数学啊!量子力学薛定谔
: 偏微分方程就不说了, 电子学里信号处理 傅立叶变换, 电磁场的麦克斯韦方程组..
: .... 好像力热光电一切物理的定理最后的归宿都是数学模型? 物理牛人光数学好肯定
: 还不够, 但数学不好恐怕万万不行。 我记得有谣传说牛顿的万有引力基础概念是胡克
: 先提的, 可惜胡克数学不够好, 所以没推到牛顿那一步?
: 计算机大学就修过一门课, 现在就是处理数据时编编简单的不能称之为程序的程序...
: ... 就不讨论了:(
: 难道因为数学教得早, 之后理化计算机用到的, 都被我自动定义成数学知识?
: 不过我还是觉得数学是理化的基础,理化和计算机都是纯数学加上各学科自己的东西..

y*******g
发帖数: 1395
324
这个电算是20年前就如此了,俺真的不知道某大妈整天在这里故意掰扯自己有点理工背
景,到底为哪般, 是显摆自己年龄超额支出了,还是显摆自己学过一点中学物理,然
后再也没碰过
marvell,在90年代就用电算了,已经20年了

【在 t*******r 的大作中提到】
: 电算了,你说的这些都不用肉算了。除非你读理论物理,不过现在理论物理也是电算为
: 主。
: 就有点像蒙古海军骑兵琢磨为啥 USMC 连骑马都不会。话糙理不糙!
:
: ?
: ..
: ..
: ..

k****r
发帖数: 851
325
问题是amc8/10不是老师给你推的,是你们这样负责任的家长自己在推. 给家长找多少事
啊.
我妈老说,孩子大点你就轻松了,看来这是没完了.

【在 t*******r 的大作中提到】
: 实际上我觉得是美帝系统的相对效率高。
: 我觉得我小时候做题量多半有我娃的三倍还多,但以成果论英雄的角度,以目前 6 年
: 级看,我觉得我当年初一最多也就跟我娃现在差不多,除了计算能力强一点。
: 我知道大伙儿一般都推崇我们当年的国内数学,很鄙视美国数学。但那是跟学校数学慢
: 道比。如果去跟 AMC 8 / 10 的大众型快道比,我觉得我们当年也未必强多少,最大的
: 强项可能就是个熟练程度我觉得。
: 将来不知道,但我目测将来如果有差别,更多的可能是 nature 的差别。

J***A
发帖数: 1511
326
纯抬杠的说, 电算肉算还不都是数学吗......
电脑不就是个二进制的人肉计算器吗?
现在研究什么量子计算, 还不都是在纠结怎么产生几个确定的态......提速也不过是
提高进制数......
完了, 我一定是数学学呆了, 或者太自恨没学数学,满脑子只看到数学:)))

电算了,你说的这些都不用肉算了。除非你读理论物理,不过现在理论物理也是电算为
主。就有点像蒙古海军骑兵琢磨为啥 USMC 连骑马都不会。话糙理不糙!

【在 t*******r 的大作中提到】
: 电算了,你说的这些都不用肉算了。除非你读理论物理,不过现在理论物理也是电算为
: 主。
: 就有点像蒙古海军骑兵琢磨为啥 USMC 连骑马都不会。话糙理不糙!
:
: ?
: ..
: ..
: ..

J***A
发帖数: 1511
327
Maxwell 本身不是数学模型? 用什么算关系大吗? 用电算不是算? 算不是数学? 潮
水的数学功底我是见过的, 他藐视数学还是有资本的,你还是去幼儿园重修再来评论
吧!

这个电算是20年前就如此了,俺真的不知道某大妈整天在这里故意掰扯自己有点理工背
景,到底为哪般, 是显摆自己年龄超额支出了,还是显摆自己学过一点中学物理,然
后再也没碰过marve........

【在 y*******g 的大作中提到】
: 这个电算是20年前就如此了,俺真的不知道某大妈整天在这里故意掰扯自己有点理工背
: 景,到底为哪般, 是显摆自己年龄超额支出了,还是显摆自己学过一点中学物理,然
: 后再也没碰过
: marvell,在90年代就用电算了,已经20年了

y*******g
发帖数: 1395
328
你啥都不懂,他故意没用专有名词
我告诉你吧, 90年代的 VLSI这都是电算的,我一个师兄学物理方向的,告诉我的
而且专业名词不叫电算,叫啥我不告诉你,以后你继续丢丑去, 潮水以前在有点像
marvell这种公司里做过,他说的是对的

【在 J***A 的大作中提到】
: 纯抬杠的说, 电算肉算还不都是数学吗......
: 电脑不就是个二进制的人肉计算器吗?
: 现在研究什么量子计算, 还不都是在纠结怎么产生几个确定的态......提速也不过是
: 提高进制数......
: 完了, 我一定是数学学呆了, 或者太自恨没学数学,满脑子只看到数学:)))
:
: 电算了,你说的这些都不用肉算了。除非你读理论物理,不过现在理论物理也是电算为
: 主。就有点像蒙古海军骑兵琢磨为啥 USMC 连骑马都不会。话糙理不糙!

y*******g
发帖数: 1395
329
你搞笑去吧,还”数学模型“呢, 你连啥是模型都没闹清楚
连朱棣文都不敢说自己是做模型的

【在 J***A 的大作中提到】
: Maxwell 本身不是数学模型? 用什么算关系大吗? 用电算不是算? 算不是数学? 潮
: 水的数学功底我是见过的, 他藐视数学还是有资本的,你还是去幼儿园重修再来评论
: 吧!
:
: 这个电算是20年前就如此了,俺真的不知道某大妈整天在这里故意掰扯自己有点理工背
: 景,到底为哪般, 是显摆自己年龄超额支出了,还是显摆自己学过一点中学物理,然
: 后再也没碰过marve........

y*******g
发帖数: 1395
330
理论物理在日本早就电算了,起码有30年了(之前我没生出来,我也不知道), 美国
科学家在80年代都去日本做这个的,现在美国也有电算能力了
我说,这个词用着真别扭啊

现在理论物理也是电算为主。

【在 t*******r 的大作中提到】
: 电算了,你说的这些都不用肉算了。除非你读理论物理,不过现在理论物理也是电算为
: 主。
: 就有点像蒙古海军骑兵琢磨为啥 USMC 连骑马都不会。话糙理不糙!
:
: ?
: ..
: ..
: ..

相关主题
小学升初中,儿子数学跳级的一点经验9岁, 如何报名考amc10
有明天考AMC 8的吗?算不算有数学天分
求科普: 奥数 vs. Math Olympiad vs. Math Count vs. Math circle vs. 超前学数学[葩论] 论“有限小灶策略”
进入Parenting版参与讨论
y*******g
发帖数: 1395
331
VLSI是狗屁数学啊,有数学公式吗?
照你这么说,intel/AMD/也是搞数学搞模型的了

【在 J***A 的大作中提到】
: 纯抬杠的说, 电算肉算还不都是数学吗......
: 电脑不就是个二进制的人肉计算器吗?
: 现在研究什么量子计算, 还不都是在纠结怎么产生几个确定的态......提速也不过是
: 提高进制数......
: 完了, 我一定是数学学呆了, 或者太自恨没学数学,满脑子只看到数学:)))
:
: 电算了,你说的这些都不用肉算了。除非你读理论物理,不过现在理论物理也是电算为
: 主。就有点像蒙古海军骑兵琢磨为啥 USMC 连骑马都不会。话糙理不糙!

J***A
发帖数: 1511
332
你知道麦克斯韦方程是什么吗? 是不是数学模型? 你知道薛定鄂定理吗? 不是数学
模型? 你的意思是你不用理解这些模型本身知道怎么算就行了?
我不是学物理的, 电算的意思我真的可能理解错了, 希望潮水给科普下。
物理学的博士在哈佛耶鲁当教授的我倒认识不少, 真有物理问题我会跟他们请教的,
跟月光您讨论, 呵呵 还是算了吧!

你搞笑去吧,还”数学模型“呢, 你连啥是模型都没闹清楚连朱棣文都不敢说自己是
做模型的

【在 y*******g 的大作中提到】
: 你搞笑去吧,还”数学模型“呢, 你连啥是模型都没闹清楚
: 连朱棣文都不敢说自己是做模型的

t*******r
发帖数: 22634
333
话虽然这么说,但如果那天你有 impact driver 去啪啪啪螺丝们,你就不会再纠结拧
螺丝们是不是适合女生的问题吧。
当然缺点是用 impact driver 啪啪啪螺丝容易上瘾,每次恨不得给多啪啪几个螺丝。
不过那屠龙刀本来是用来装 deck 这种大型重磅体力活的。
哥们您先买把 impact driver 爽一回,估计就不会纠结这个问题了。记得戴上 impact
goggle,因为新手用 impact driver,保不准漫天都是谁的螺丝在飞。。。

【在 J***A 的大作中提到】
: 纯抬杠的说, 电算肉算还不都是数学吗......
: 电脑不就是个二进制的人肉计算器吗?
: 现在研究什么量子计算, 还不都是在纠结怎么产生几个确定的态......提速也不过是
: 提高进制数......
: 完了, 我一定是数学学呆了, 或者太自恨没学数学,满脑子只看到数学:)))
:
: 电算了,你说的这些都不用肉算了。除非你读理论物理,不过现在理论物理也是电算为
: 主。就有点像蒙古海军骑兵琢磨为啥 USMC 连骑马都不会。话糙理不糙!

J***A
发帖数: 1511
334
你认为intel amd的人不搞统计模型?
知道现在semiconductor的器件做到20nm以下大数据和统计在对其性能测试, failure
rate 评估中的重要性吗?
自己搜搜它们的paper再来吧, 没文化不可怕, Stanford的没文化就很可怕了!

VLSI是狗屁数学啊,有数学公式吗?照你这么说,intel/AMD/也是搞数学搞模型的了

【在 y*******g 的大作中提到】
: VLSI是狗屁数学啊,有数学公式吗?
: 照你这么说,intel/AMD/也是搞数学搞模型的了

y*******g
发帖数: 1395
335
赞”谁的螺丝在空中飞舞“!!!!

impact

【在 t*******r 的大作中提到】
: 话虽然这么说,但如果那天你有 impact driver 去啪啪啪螺丝们,你就不会再纠结拧
: 螺丝们是不是适合女生的问题吧。
: 当然缺点是用 impact driver 啪啪啪螺丝容易上瘾,每次恨不得给多啪啪几个螺丝。
: 不过那屠龙刀本来是用来装 deck 这种大型重磅体力活的。
: 哥们您先买把 impact driver 爽一回,估计就不会纠结这个问题了。记得戴上 impact
: goggle,因为新手用 impact driver,保不准漫天都是谁的螺丝在飞。。。

J***A
发帖数: 1511
336
vlsi是物理! 厉害 受教了, 哈哈哈哈!

VLSI是狗屁数学啊,有数学公式吗?照你这么说,intel/AMD/也是搞数学搞模型的了

【在 y*******g 的大作中提到】
: VLSI是狗屁数学啊,有数学公式吗?
: 照你这么说,intel/AMD/也是搞数学搞模型的了

t*******r
发帖数: 22634
337
我上面用 impact driver 只是个比方。
也就是说,现代 li-ion battery driven impact driver,并没有降低对活塞和气缸之
间...敲错了...螺丝和木头之间...螺丝和木头之间的关系的理解。否则要么是漫天都
是谁的螺丝在飞,要么满地都是谁的木头在裂。但对手工解方程算微积分这种体力活的
要求大大降低了。

【在 J***A 的大作中提到】
: 纯抬杠的说, 电算肉算还不都是数学吗......
: 电脑不就是个二进制的人肉计算器吗?
: 现在研究什么量子计算, 还不都是在纠结怎么产生几个确定的态......提速也不过是
: 提高进制数......
: 完了, 我一定是数学学呆了, 或者太自恨没学数学,满脑子只看到数学:)))
:
: 电算了,你说的这些都不用肉算了。除非你读理论物理,不过现在理论物理也是电算为
: 主。就有点像蒙古海军骑兵琢磨为啥 USMC 连骑马都不会。话糙理不糙!

t*******r
发帖数: 22634
338
不仅是 fail rate,每个管子门,其速度特性有散布,这些散步最终对 CPU 速度主频
影响多大,也得靠数学模型。

failure

【在 J***A 的大作中提到】
: 你认为intel amd的人不搞统计模型?
: 知道现在semiconductor的器件做到20nm以下大数据和统计在对其性能测试, failure
: rate 评估中的重要性吗?
: 自己搜搜它们的paper再来吧, 没文化不可怕, Stanford的没文化就很可怕了!
:
: VLSI是狗屁数学啊,有数学公式吗?照你这么说,intel/AMD/也是搞数学搞模型的了

J***A
发帖数: 1511
339
对,正是这样, 我因为对软件数据结构那些了解甚微, 但我估摸着很多搞工程和物理
的, 还在纠结怎么建这个数学模型, 或者怎么挑选已有的数学模型, 等有了模型以
后, 当然会有很好的辅助工具不需要人肉计算了。
当然也许其实也有模型库了? 给出数据,电脑就可以自动给建模了? 这点想做到也不
难, 所以人工智能真可怕, 大批失业的前兆......

不仅是 fail rate,每个管子门,其速度特性有散布,这些散步最终对 CPU 速度主频
影响多大,也得靠数学模型。

【在 t*******r 的大作中提到】
: 不仅是 fail rate,每个管子门,其速度特性有散布,这些散步最终对 CPU 速度主频
: 影响多大,也得靠数学模型。
:
: failure

t*******r
发帖数: 22634
340
光有模型不够,那模型还得最终弄成 practically computable for the whole system
within resource limit。否则这玩意儿俩月才算出结果,INTC 说,圣诞节都过了。

【在 J***A 的大作中提到】
: 对,正是这样, 我因为对软件数据结构那些了解甚微, 但我估摸着很多搞工程和物理
: 的, 还在纠结怎么建这个数学模型, 或者怎么挑选已有的数学模型, 等有了模型以
: 后, 当然会有很好的辅助工具不需要人肉计算了。
: 当然也许其实也有模型库了? 给出数据,电脑就可以自动给建模了? 这点想做到也不
: 难, 所以人工智能真可怕, 大批失业的前兆......
:
: 不仅是 fail rate,每个管子门,其速度特性有散布,这些散步最终对 CPU 速度主频
: 影响多大,也得靠数学模型。

相关主题
[葩论] 论“有限小灶策略”小学数学有比赛项目吗
给初中学生家长的一些建议 (转载)请教:怎样说服孩儿他爸多关心孩子的学习?
小学数学上超前班有什么好处吗?学校早晚要教的东西,早早学会了又如何?
进入Parenting版参与讨论
J***A
发帖数: 1511
341
那天我还跟ld说我们家需要个impact driver, (事实ld后来装书架手都磨坏了)这样
买家具连我都能自己装了。结果ld说那个impact driver 力气太大, 用不好估计直接
就装坏了, 螺丝木屑乱飞:)
其实数学物理哪个先有都不重要了:) 能融会贯通了然于胸, 该用哪个用哪个不用纠
结用的是什么。

我上面用 impact driver 只是个比方。也就是说,现代 li-ion battery driven
impact driver,并没有降低对活塞和气缸之间...敲错了........

【在 t*******r 的大作中提到】
: 我上面用 impact driver 只是个比方。
: 也就是说,现代 li-ion battery driven impact driver,并没有降低对活塞和气缸之
: 间...敲错了...螺丝和木头之间...螺丝和木头之间的关系的理解。否则要么是漫天都
: 是谁的螺丝在飞,要么满地都是谁的木头在裂。但对手工解方程算微积分这种体力活的
: 要求大大降低了。

t*******r
发帖数: 22634
342
AMC 8 / 10 网上有答案有解释,kick start 以后不花父母多少事。
其实也就是四/五年级数学费父母的事比较多点,六年级开始学校分快车道,娃觉得学
校数学课很有趣。而且我娃不会 explain,老师课上会教。回家娃自己加强一下
problem solving 半小时即可。除非你家是竞赛娃,否则父母的事儿不会太多。
我现在都开始把重心往小娃移了。以及大娃的 science 阅读。

【在 k****r 的大作中提到】
: 问题是amc8/10不是老师给你推的,是你们这样负责任的家长自己在推. 给家长找多少事
: 啊.
: 我妈老说,孩子大点你就轻松了,看来这是没完了.

J***A
发帖数: 1511
343
现在工业界qualification technology时选数学模型还不都是time driven的, 不需要
物理背景, 甚至不需要是对数据fit的最好的, 只求模型本身简单...... 那些超级复
杂的具有物理意义的模型基本都只有research阶段发paper用

光有模型不够,那模型还得最终弄成 practically computable for the whole system
within resource limit。否则这玩意........

【在 t*******r 的大作中提到】
: 光有模型不够,那模型还得最终弄成 practically computable for the whole system
: within resource limit。否则这玩意儿俩月才算出结果,INTC 说,圣诞节都过了。

t*******r
发帖数: 22634
344
有力矩可调的型号,另外 trigger 也不要拉太狠。
当然装一书桌也不用屠龙刀就是了。

【在 J***A 的大作中提到】
: 那天我还跟ld说我们家需要个impact driver, (事实ld后来装书架手都磨坏了)这样
: 买家具连我都能自己装了。结果ld说那个impact driver 力气太大, 用不好估计直接
: 就装坏了, 螺丝木屑乱飞:)
: 其实数学物理哪个先有都不重要了:) 能融会贯通了然于胸, 该用哪个用哪个不用纠
: 结用的是什么。
:
: 我上面用 impact driver 只是个比方。也就是说,现代 li-ion battery driven
: impact driver,并没有降低对活塞和气缸之间...敲错了........

t*******r
发帖数: 22634
345
公司里,一般 theoretical scienter 总是建议高大上的模型,然后 computational
scienter 立马敲桌子曰你丫有估算需要跑多少 FPU 指令没?Engineer 跳进来说你两
别吵了,我们一堆烂 code,要改你们自己改,改完以后记得贿赂 testing 部门,我不
管你们。

system

【在 J***A 的大作中提到】
: 现在工业界qualification technology时选数学模型还不都是time driven的, 不需要
: 物理背景, 甚至不需要是对数据fit的最好的, 只求模型本身简单...... 那些超级复
: 杂的具有物理意义的模型基本都只有research阶段发paper用
:
: 光有模型不够,那模型还得最终弄成 practically computable for the whole system
: within resource limit。否则这玩意........

J***A
发帖数: 1511
346
呵呵 , 虽然不同的公司下属部门和名称略有不同, 不过十分属实:)

公司里,一般 theoretical scienter 总是建议高大上的模型,然后 computational
scienter 立马敲桌子曰你丫有估算需要跑多少 FPU 指令........

【在 t*******r 的大作中提到】
: 公司里,一般 theoretical scienter 总是建议高大上的模型,然后 computational
: scienter 立马敲桌子曰你丫有估算需要跑多少 FPU 指令没?Engineer 跳进来说你两
: 别吵了,我们一堆烂 code,要改你们自己改,改完以后记得贿赂 testing 部门,我不
: 管你们。
:
: system

l*****8
发帖数: 16949
347
哈,这个是潮水的特点,杀条蚯蚓要上屠龙刀,说个小学生概念都能祭出编译原理到机
器学习各种高大上的理论。装书架哪要冲击钻啊,那玩意儿是打混凝土墙的。就买个便
宜的普通小电钻就行了。

【在 J***A 的大作中提到】
: 那天我还跟ld说我们家需要个impact driver, (事实ld后来装书架手都磨坏了)这样
: 买家具连我都能自己装了。结果ld说那个impact driver 力气太大, 用不好估计直接
: 就装坏了, 螺丝木屑乱飞:)
: 其实数学物理哪个先有都不重要了:) 能融会贯通了然于胸, 该用哪个用哪个不用纠
: 结用的是什么。
:
: 我上面用 impact driver 只是个比方。也就是说,现代 li-ion battery driven
: impact driver,并没有降低对活塞和气缸之间...敲错了........

t*******r
发帖数: 22634
348
主要是工具太多了找起来费劲,屠龙刀也能杀鸡不是?但反过来不行。
另外用电钻拧木螺丝的话,只能上很小功率的电钻,(impact driver 就没有这个问题
,随便啪啪啪)。,否则很容易把螺丝头给直接拧滑了,就这样也得记得用足够的力量
顶上。
这种我也有一个,black & decker 装普通 AA 电池的电动螺丝刀 (可以用 AA 镍氢电
池),保证任何时候都有电池,免维护。
http://www.blackanddecker.com/products/power-tools/portable-pow

【在 l*****8 的大作中提到】
: 哈,这个是潮水的特点,杀条蚯蚓要上屠龙刀,说个小学生概念都能祭出编译原理到机
: 器学习各种高大上的理论。装书架哪要冲击钻啊,那玩意儿是打混凝土墙的。就买个便
: 宜的普通小电钻就行了。

l*****8
发帖数: 16949
349
我家里这种工具也是一堆。以前黑色星期5总想买点啥,结果家里一堆。不过一般用电
池的不至于吧螺丝头拧滑了,没那么大的劲。
你这样的impact我也有,当时冲着"impact"去的,结果木头上拧螺丝还行,水泥墙上钻
洞就别想了。

【在 t*******r 的大作中提到】
: 主要是工具太多了找起来费劲,屠龙刀也能杀鸡不是?但反过来不行。
: 另外用电钻拧木螺丝的话,只能上很小功率的电钻,(impact driver 就没有这个问题
: ,随便啪啪啪)。,否则很容易把螺丝头给直接拧滑了,就这样也得记得用足够的力量
: 顶上。
: 这种我也有一个,black & decker 装普通 AA 电池的电动螺丝刀 (可以用 AA 镍氢电
: 池),保证任何时候都有电池,免维护。
: http://www.blackanddecker.com/products/power-tools/portable-pow

t*******r
发帖数: 22634
350
哥们您不能拿手枪打坦克吧,水泥墙一般上突击步枪。
当然我家也没突击步枪,不常用。我偶尔水泥墙打洞的话,打手枪连续打十几分钟也能
搞出一小洞,还不至于精尽人亡。

【在 l*****8 的大作中提到】
: 我家里这种工具也是一堆。以前黑色星期5总想买点啥,结果家里一堆。不过一般用电
: 池的不至于吧螺丝头拧滑了,没那么大的劲。
: 你这样的impact我也有,当时冲着"impact"去的,结果木头上拧螺丝还行,水泥墙上钻
: 洞就别想了。

相关主题
AMC 8 成绩 2016我来说一下天才的鉴定方法
请教小孩数学教育问题[华盛顿邮报] 9 - 14 岁是数学兴趣的关键时期?
小学数学书推荐?怎么推小学数学好?小学一年级的孩子。
进入Parenting版参与讨论
l*****8
发帖数: 16949
351
这个我也有。当时在网上买的,没看见实货,看到impact就买回来了。后来又买了一把
真正的impact.
问题是那么小的电钻要impact干嘛?就好像说穿甲弹,结果是给手枪用的。

【在 t*******r 的大作中提到】
: 哥们您不能拿手枪打坦克吧,水泥墙一般上突击步枪。
: 当然我家也没突击步枪,不常用。我偶尔水泥墙打洞的话,打手枪连续打十几分钟也能
: 搞出一小洞,还不至于精尽人亡。

J***A
发帖数: 1511
352
哈哈 我还是找个小电钻好了:)
不handy的用屠龙刀自残的可能性太大了:) 整不好家具和墙都让我给拆了:)

哈,这个是潮水的特点,杀条蚯蚓要上屠龙刀,说个小学生概念都能祭出编译原理到机
器学习各种高大上的理论。装书架哪要冲击钻啊,那玩意儿是打混凝土墙的。就买个便
宜的普通小电钻就行了。

【在 l*****8 的大作中提到】
: 哈,这个是潮水的特点,杀条蚯蚓要上屠龙刀,说个小学生概念都能祭出编译原理到机
: 器学习各种高大上的理论。装书架哪要冲击钻啊,那玩意儿是打混凝土墙的。就买个便
: 宜的普通小电钻就行了。

t*******r
发帖数: 22634
353
impact 的目的是基本不会拧滑螺丝头,不用压很大的力,几啪迅速解决问题。如果有
卡住,也不容易拧断螺丝。Mistake free。

【在 l*****8 的大作中提到】
: 这个我也有。当时在网上买的,没看见实货,看到impact就买回来了。后来又买了一把
: 真正的impact.
: 问题是那么小的电钻要impact干嘛?就好像说穿甲弹,结果是给手枪用的。

J***A
发帖数: 1511
354
我想弄个照片墙, 就是挂一堆照片在墙上, 有打孔的好推荐吗? 给个link或照片!
不要屠龙刀啊! 能切萝卜的菜刀就行:)

哥们您不能拿手枪打坦克吧,水泥墙一般上突击步枪。当然我家也没突击步枪,不常用
。我偶尔水泥墙打洞的话,打手枪连续打十几分钟也能搞出一小洞,还不至于精尽人亡


【在 t*******r 的大作中提到】
: 哥们您不能拿手枪打坦克吧,水泥墙一般上突击步枪。
: 当然我家也没突击步枪,不常用。我偶尔水泥墙打洞的话,打手枪连续打十几分钟也能
: 搞出一小洞,还不至于精尽人亡。

t*******r
发帖数: 22634
355
dry wall?用 3M 墙壁贴,不用打洞了。
dry wall 打洞虽然不是个事儿,但太不 handy 的也有打在 110v 电线上的小概率事件。



【在 J***A 的大作中提到】
: 我想弄个照片墙, 就是挂一堆照片在墙上, 有打孔的好推荐吗? 给个link或照片!
: 不要屠龙刀啊! 能切萝卜的菜刀就行:)
:
: 哥们您不能拿手枪打坦克吧,水泥墙一般上突击步枪。当然我家也没突击步枪,不常用
: 。我偶尔水泥墙打洞的话,打手枪连续打十几分钟也能搞出一小洞,还不至于精尽人亡
: 。

J***A
发帖数: 1511
356
什么叫dry wall , 完全不懂......
等我Google 下, 因为这照片墙是我想要的, 我家ld 属于我不把房子拆了, 家里布
置装饰什么一概没兴趣的, 所以我得负责查清楚操作准备好冰箱贴, 然后ld操作,他
好歹还算很handy? 不过看了你们这么多专业工具, 我家只有个有两螺丝刀的小tool
set 以后有点不确定了......
3m墙壁贴记下了, 等再去查查.......

dry wall?用 3M 墙壁贴,不用打洞了。dry wall 打洞虽然不是个事儿,但太不
handy 的也有打在 110v 电线上的小概率事件。

【在 t*******r 的大作中提到】
: dry wall?用 3M 墙壁贴,不用打洞了。
: dry wall 打洞虽然不是个事儿,但太不 handy 的也有打在 110v 电线上的小概率事件。
:
: !

l*****8
发帖数: 16949
357
如果只是挂照片,只要钉子不打在stub上,用个锤子敲一下就进去了,除了锤子啥工具
都不用。就怕画重挂不牢,必须把钉子打到stub上,那就要上电钻了。

tool

【在 J***A 的大作中提到】
: 什么叫dry wall , 完全不懂......
: 等我Google 下, 因为这照片墙是我想要的, 我家ld 属于我不把房子拆了, 家里布
: 置装饰什么一概没兴趣的, 所以我得负责查清楚操作准备好冰箱贴, 然后ld操作,他
: 好歹还算很handy? 不过看了你们这么多专业工具, 我家只有个有两螺丝刀的小tool
: set 以后有点不确定了......
: 3m墙壁贴记下了, 等再去查查.......
:
: dry wall?用 3M 墙壁贴,不用打洞了。dry wall 打洞虽然不是个事儿,但太不
: handy 的也有打在 110v 电线上的小概率事件。

t*******r
发帖数: 22634
358
还有 dry wall anchor

【在 l*****8 的大作中提到】
: 如果只是挂照片,只要钉子不打在stub上,用个锤子敲一下就进去了,除了锤子啥工具
: 都不用。就怕画重挂不牢,必须把钉子打到stub上,那就要上电钻了。
:
: tool

r******s
发帖数: 1388
359
Stud。知道这个词的原意就不会混。

【在 l*****8 的大作中提到】
: 如果只是挂照片,只要钉子不打在stub上,用个锤子敲一下就进去了,除了锤子啥工具
: 都不用。就怕画重挂不牢,必须把钉子打到stub上,那就要上电钻了。
:
: tool

J***A
发帖数: 1511
360
多谢!明白了, 初步打算只挂小照片:)
也许挂着挂着就洗个大的当画挂上去了, 到时再找stub 买电钻:)

如果只是挂照片,只要钉子不打在stub上,用个锤子敲一下就进去了,除了锤子啥工具
都不用。就怕画重挂不牢,必须把钉子打到stub上,那就要上电钻了。

【在 l*****8 的大作中提到】
: 如果只是挂照片,只要钉子不打在stub上,用个锤子敲一下就进去了,除了锤子啥工具
: 都不用。就怕画重挂不牢,必须把钉子打到stub上,那就要上电钻了。
:
: tool

相关主题
请问,AMC 8应该从几年级开始练习呀?有明天考AMC 8的吗?
数学教育 一家之言 番外篇求科普: 奥数 vs. Math Olympiad vs. Math Count vs. Math circle vs. 超前学数学
小学升初中,儿子数学跳级的一点经验9岁, 如何报名考amc10
进入Parenting版参与讨论
J***A
发帖数: 1511
361
你们说一个东西我都要Google是什么东西, 可见我这工科学的多悲催....

还有 dry wall anchor

【在 t*******r 的大作中提到】
: 还有 dry wall anchor
t*******r
发帖数: 22634
362
stud finder

【在 J***A 的大作中提到】
: 多谢!明白了, 初步打算只挂小照片:)
: 也许挂着挂着就洗个大的当画挂上去了, 到时再找stub 买电钻:)
:
: 如果只是挂照片,只要钉子不打在stub上,用个锤子敲一下就进去了,除了锤子啥工具
: 都不用。就怕画重挂不牢,必须把钉子打到stub上,那就要上电钻了。

d**********h
发帖数: 2795
363
潮水兄向来是战略上的猛士,战术上的保守派 :)

【在 l*****8 的大作中提到】
: 哈,这个是潮水的特点,杀条蚯蚓要上屠龙刀,说个小学生概念都能祭出编译原理到机
: 器学习各种高大上的理论。装书架哪要冲击钻啊,那玩意儿是打混凝土墙的。就买个便
: 宜的普通小电钻就行了。

t*******r
发帖数: 22634
364
属实。
很多时候我如果不怕麻烦拿出 1000+ inch-lb 扭矩的 impact driver,
真正的目的就是为了不拧花螺钉头。或者我就走另一个极端,用 20
inch-lb 慢速低扭矩 power screw driver,真正的目的还是为了避免拧
花螺钉头。当然外加省点力。
确实我是战术上的极端保守派没错。

【在 d**********h 的大作中提到】
: 潮水兄向来是战略上的猛士,战术上的保守派 :)
t*******r
发帖数: 22634
365
其实我更大的问题是,大娃的 interest 从哪里来,为啥板门店谈判只能搞成
每天半小时的数学加料,而娃版 medical 以前就不用谈判。
我觉得除了数学内容相对枯燥的因子之外,我认为另一个重要的因子就是,你
曾经说过的 “normative vs positive”。
或者联系实际的说,数学在超过半小时加料之后,对娃而言,本质上是 positive
而不是 normative,也就是说,对大娃内心而言,那不是必须的。
而娃版 medical,对娃而言,本质上更接近 normative,因此更容易上小时数。
当然六年级时,娃连 non-fiction reading 都不干了。我现在想是不是有遇到
瓶劲期的可能。当然另一个原因是娃太想看 Lab Rat 那个 junk TV,但总得试试
能不能跟娃版 medical 泛读给平衡一下,不试不知道。我目前只知道靠数学多半
不行,数学做多了更枯燥,搞不好可能更想看 Lab Rat。目前数学我觉得半小时
就差不多了。

everything。

【在 Y********d 的大作中提到】
:
: 我印象中,有“数学是自然科学的哲学”一说?
: 这个大概也是版上的父母都热衷于数学的原因吧。

Y********d
发帖数: 1478
366
同意,什么事情到最后都是一个normative的问题:为什么,好不好。
如果你娃喜欢medical加电视剧,我推荐CSI和Dr.House。我看电视剧,最喜欢看这一类
的。
看完了,绝对燃起对医学和科学的双重兴趣。
当然对于六年级娃是不是口味太重了一点,你把握就是了。

【在 t*******r 的大作中提到】
: 其实我更大的问题是,大娃的 interest 从哪里来,为啥板门店谈判只能搞成
: 每天半小时的数学加料,而娃版 medical 以前就不用谈判。
: 我觉得除了数学内容相对枯燥的因子之外,我认为另一个重要的因子就是,你
: 曾经说过的 “normative vs positive”。
: 或者联系实际的说,数学在超过半小时加料之后,对娃而言,本质上是 positive
: 而不是 normative,也就是说,对大娃内心而言,那不是必须的。
: 而娃版 medical,对娃而言,本质上更接近 normative,因此更容易上小时数。
: 当然六年级时,娃连 non-fiction reading 都不干了。我现在想是不是有遇到
: 瓶劲期的可能。当然另一个原因是娃太想看 Lab Rat 那个 junk TV,但总得试试
: 能不能跟娃版 medical 泛读给平衡一下,不试不知道。我目前只知道靠数学多半

t*******r
发帖数: 22634
367
你这个是 PG13?
其实要引导兴趣,那还不如 PBS : Your Inner Fish 系列,或者 PBS Frontline 有关
superbug 的,或者 PBS NOVA 有关 Neanderthals 的,等等 documentaries 更直接
有效。不过那些娃都看过了,我觉得她的问题在于读娃版 medical book 要上一点深度
,这样才能 kill time。另外实际上我也要 cut down 娃的 screen time。

【在 Y********d 的大作中提到】
: 同意,什么事情到最后都是一个normative的问题:为什么,好不好。
: 如果你娃喜欢medical加电视剧,我推荐CSI和Dr.House。我看电视剧,最喜欢看这一类
: 的。
: 看完了,绝对燃起对医学和科学的双重兴趣。
: 当然对于六年级娃是不是口味太重了一点,你把握就是了。

t*******r
发帖数: 22634
368
另外比较娱乐一些的比如 mythbusters 系列,她看了有三遍不止。还有几个厨师的 TV
series。反正市面上容易搞到的,图书馆有的,她愿意看的都翻了个底朝天,老实说
除了上一点点深度,我老手上半年前就彻底没货了。

【在 Y********d 的大作中提到】
: 同意,什么事情到最后都是一个normative的问题:为什么,好不好。
: 如果你娃喜欢medical加电视剧,我推荐CSI和Dr.House。我看电视剧,最喜欢看这一类
: 的。
: 看完了,绝对燃起对医学和科学的双重兴趣。
: 当然对于六年级娃是不是口味太重了一点,你把握就是了。

Y********d
发帖数: 1478
369

TV
我先把你提到的这几个系列mark了,看来跟着大娃爸妈混就是有好处啊:-)
她要是真那么喜欢medical, 告诉她数理化生物基础都要好呀,将来做个医生很不错哦
!确实不一定非要爬藤,我总算明白了你的非藤情节。

【在 t*******r 的大作中提到】
: 另外比较娱乐一些的比如 mythbusters 系列,她看了有三遍不止。还有几个厨师的 TV
: series。反正市面上容易搞到的,图书馆有的,她愿意看的都翻了个底朝天,老实说
: 除了上一点点深度,我老手上半年前就彻底没货了。

t*******r
发帖数: 22634
370
我觉得这不是她喜欢 medical,而是 curiosity to herself。
不过我主要纠结的,是大娃不要看 TV 玩游戏太多,找个办法给她 kill time。我同时
还要把小娃适当拉开,防止大娃她 drag down 小娃。而不是纠结啥藤校医生。

【在 Y********d 的大作中提到】
:
: TV
: 我先把你提到的这几个系列mark了,看来跟着大娃爸妈混就是有好处啊:-)
: 她要是真那么喜欢medical, 告诉她数理化生物基础都要好呀,将来做个医生很不错哦
: !确实不一定非要爬藤,我总算明白了你的非藤情节。

相关主题
9岁, 如何报名考amc10给初中学生家长的一些建议 (转载)
算不算有数学天分小学数学上超前班有什么好处吗?
[葩论] 论“有限小灶策略”小学数学有比赛项目吗
进入Parenting版参与讨论
Y********d
发帖数: 1478
371
期待你多讲讲小娃的指导,我们这些小娃父母也可以多些参考。
同时记得及时反馈大娃的进展,期待着一个小小科学家的诞生。

【在 t*******r 的大作中提到】
: 我觉得这不是她喜欢 medical,而是 curiosity to herself。
: 不过我主要纠结的,是大娃不要看 TV 玩游戏太多,找个办法给她 kill time。我同时
: 还要把小娃适当拉开,防止大娃她 drag down 小娃。而不是纠结啥藤校医生。

Y********d
发帖数: 1478
372
再赞一下这几个形象生动的比喻,总算理解了你对人肉计算器不屑的深刻原因。
还有看样子你也不全是nerd,至少还能够让螺丝和木头漫天飞舞:-)

【在 t*******r 的大作中提到】
: 我上面用 impact driver 只是个比方。
: 也就是说,现代 li-ion battery driven impact driver,并没有降低对活塞和气缸之
: 间...敲错了...螺丝和木头之间...螺丝和木头之间的关系的理解。否则要么是漫天都
: 是谁的螺丝在飞,要么满地都是谁的木头在裂。但对手工解方程算微积分这种体力活的
: 要求大大降低了。

t*******r
发帖数: 22634
373
钻个螺丝打个眼,那只能算 handy 水平九九乘法表的水准。
handy 水平 AMC 8 把是上房顶拉电线。
handy 水平 AMC 10 是下 crawl space 换水管。
handy 水平 AMC 12 是卸轮子换刹车 rotor 还有水冷点火传感器电子系统。
handy 水平 AIME 是把发动机大卸八块。
等等等等。
大家对号入座吧。

【在 Y********d 的大作中提到】
: 再赞一下这几个形象生动的比喻,总算理解了你对人肉计算器不屑的深刻原因。
: 还有看样子你也不全是nerd,至少还能够让螺丝和木头漫天飞舞:-)

Y********d
发帖数: 1478
374

结论: handy 水平 IMO 是找一个这些都能轻松完成的partner。
新题:请填空
humor 水平 AMC 8 是
humor 水平 AMC 10 是
humor 水平 AMC 12 是
humor 水平 AIME 是

【在 t*******r 的大作中提到】
: 钻个螺丝打个眼,那只能算 handy 水平九九乘法表的水准。
: handy 水平 AMC 8 把是上房顶拉电线。
: handy 水平 AMC 10 是下 crawl space 换水管。
: handy 水平 AMC 12 是卸轮子换刹车 rotor 还有水冷点火传感器电子系统。
: handy 水平 AIME 是把发动机大卸八块。
: 等等等等。
: 大家对号入座吧。

l*****8
发帖数: 16949
375
乱分类。卸轮子换刹车rotor都太简单,比你的AMC10简单。换水管有时还真不容易。

【在 t*******r 的大作中提到】
: 钻个螺丝打个眼,那只能算 handy 水平九九乘法表的水准。
: handy 水平 AMC 8 把是上房顶拉电线。
: handy 水平 AMC 10 是下 crawl space 换水管。
: handy 水平 AMC 12 是卸轮子换刹车 rotor 还有水冷点火传感器电子系统。
: handy 水平 AIME 是把发动机大卸八块。
: 等等等等。
: 大家对号入座吧。

J***A
发帖数: 1511
376
其实对很多人,比如我, 除了第一项, 其它都是一个难度imo.......不会、 不会 再
学也不会........

乱分类。卸轮子换刹车rotor都太简单,比你的AMC10简单。换水管有时还真不容易。

【在 l*****8 的大作中提到】
: 乱分类。卸轮子换刹车rotor都太简单,比你的AMC10简单。换水管有时还真不容易。
y*******g
发帖数: 1395
377
看你每天12小时泡在这里
天天泡,已经泡了三个月
每小时都在发帖状态下
根本就是全职主妇吧
我并不是每天都来发帖的,至少最近一个月就根本没来
而且以前也不是每天每小时都在发的
估计文科生的工作不好找

【在 J***A 的大作中提到】
: 其实对很多人,比如我, 除了第一项, 其它都是一个难度imo.......不会、 不会 再
: 学也不会........
:
: 乱分类。卸轮子换刹车rotor都太简单,比你的AMC10简单。换水管有时还真不容易。

x***1
发帖数: 999
378
AMC8 相当于登三山五岳:黄山、庐山、雁荡山,泰山、华山、 嵩山、恒山、衡山。
AMC10相当于过五湖四海:洞庭湖、鄱阳湖、太湖、巢湖、洪泽湖,黄海、渤海、南海
,贝加尔海。
AMC12相当于跨五洋七洲:大西洋、太平洋、印度洋、北冰洋、南冰洋,亚洲、欧洲、
非洲、拉丁美洲、北美洲、大洋州、南极洲。
AIME相当于上珠穆朗玛峰。

【在 Y********d 的大作中提到】
:
: 结论: handy 水平 IMO 是找一个这些都能轻松完成的partner。
: 新题:请填空
: humor 水平 AMC 8 是
: humor 水平 AMC 10 是
: humor 水平 AMC 12 是
: humor 水平 AIME 是

y*******g
发帖数: 1395
379
这些地方我父母全去过
其实我们这代人,表面上留学美国,实际上啥都没享受到
赶上了千军万马过独木桥的高考
赶上了千军万马的托福GRE留学大潮
赶上了IT泡沫大学知识全部报废要重新来过
赶上了推娃爬藤这么一个重担
我们父辈多轻松啊,啥操心事儿都木有

【在 x***1 的大作中提到】
: AMC8 相当于登三山五岳:黄山、庐山、雁荡山,泰山、华山、 嵩山、恒山、衡山。
: AMC10相当于过五湖四海:洞庭湖、鄱阳湖、太湖、巢湖、洪泽湖,黄海、渤海、南海
: ,贝加尔海。
: AMC12相当于跨五洋七洲:大西洋、太平洋、印度洋、北冰洋、南冰洋,亚洲、欧洲、
: 非洲、拉丁美洲、北美洲、大洋州、南极洲。
: AIME相当于上珠穆朗玛峰。

u*****a
发帖数: 6276
380
我靠。
那拿诺贝尔奖,还不得成去太阳上度假了!

【在 x***1 的大作中提到】
: AMC8 相当于登三山五岳:黄山、庐山、雁荡山,泰山、华山、 嵩山、恒山、衡山。
: AMC10相当于过五湖四海:洞庭湖、鄱阳湖、太湖、巢湖、洪泽湖,黄海、渤海、南海
: ,贝加尔海。
: AMC12相当于跨五洋七洲:大西洋、太平洋、印度洋、北冰洋、南冰洋,亚洲、欧洲、
: 非洲、拉丁美洲、北美洲、大洋州、南极洲。
: AIME相当于上珠穆朗玛峰。

相关主题
请教:怎样说服孩儿他爸多关心孩子的学习?请教小孩数学教育问题
学校早晚要教的东西,早早学会了又如何?小学数学书推荐?
AMC 8 成绩 2016我来说一下天才的鉴定方法
进入Parenting版参与讨论
t*******r
发帖数: 22634
381
属实,相对太阳而言,珠峰就是屎。

【在 u*****a 的大作中提到】
: 我靠。
: 那拿诺贝尔奖,还不得成去太阳上度假了!

x***1
发帖数: 999
382
别打击啊,这不就是完成黄教授的作业嘛。

【在 u*****a 的大作中提到】
: 我靠。
: 那拿诺贝尔奖,还不得成去太阳上度假了!

u*****a
发帖数: 6276
383
I drive through the streets, and I care not a damn;
The people they stare, and they ask who I am;
And if I should chance to run over a cad,
I can pay for the damage if ever so bad.
So pleasant it is to win AIME, heigh ho!
So pleasant it is to win AIME.
J***A
发帖数: 1511
384
全职主妇是我的人生理想啊:) 可惜现阶段来说太奢侈了。
ld 事业刚起步,我还得替他分担, 让他能更有底气的工作, 等他小有所成了, 我绝
对功成身退做欢乐的家煮:)

看你每天12小时泡在这里天天泡,已经泡了三个月每小时都在发帖状态下根本就是全职
主妇吧我并不是每天都来发帖的,至少最近一个月就根本没来而且以前也不是每天每小
时都在发的估计文科生的........

【在 y*******g 的大作中提到】
: 看你每天12小时泡在这里
: 天天泡,已经泡了三个月
: 每小时都在发帖状态下
: 根本就是全职主妇吧
: 我并不是每天都来发帖的,至少最近一个月就根本没来
: 而且以前也不是每天每小时都在发的
: 估计文科生的工作不好找

y*******g
发帖数: 1395
385
好好分担吧,
一辆烂车,住apartment,只能买学区不做要求的condo,上下班单程不堵车40分钟的美
好生活,
period
这是我本次露面的最后一贴,bye bye

【在 J***A 的大作中提到】
: 全职主妇是我的人生理想啊:) 可惜现阶段来说太奢侈了。
: ld 事业刚起步,我还得替他分担, 让他能更有底气的工作, 等他小有所成了, 我绝
: 对功成身退做欢乐的家煮:)
:
: 看你每天12小时泡在这里天天泡,已经泡了三个月每小时都在发帖状态下根本就是全职
: 主妇吧我并不是每天都来发帖的,至少最近一个月就根本没来而且以前也不是每天每小
: 时都在发的估计文科生的........

y*******g
发帖数: 1395
386
(帖子删了? 怕我找出那第一套?)
我只要掌握了你没钱买好房子,开不起好车新车,对学区无要求,对交通也能忍受到40
分钟的这种经济惨状的证据,就足矣。。
说一句刺你自尊心的话,我们家投资房已经两套了,年底准备再买第三套投资房, 现
在正在到处看房。
这是第一套的房租(如图),他们每次都是连交一个季度的,唯独这次交两个月的,事
先跟我打了一声招呼。 而第二套的房租,是今天交钱,每月15日。
希望我别听到你连google hangout哪个是对方发的,哪个是我发的,都不会辨别的弱智
台词。
说清楚我的立场了,bye bye了。。。

【在 J***A 的大作中提到】
: 全职主妇是我的人生理想啊:) 可惜现阶段来说太奢侈了。
: ld 事业刚起步,我还得替他分担, 让他能更有底气的工作, 等他小有所成了, 我绝
: 对功成身退做欢乐的家煮:)
:
: 看你每天12小时泡在这里天天泡,已经泡了三个月每小时都在发帖状态下根本就是全职
: 主妇吧我并不是每天都来发帖的,至少最近一个月就根本没来而且以前也不是每天每小
: 时都在发的估计文科生的........

1 (共1页)
进入Parenting版参与讨论
相关主题
请问,AMC 8应该从几年级开始练习呀?给初中学生家长的一些建议 (转载)
数学教育 一家之言 番外篇小学数学上超前班有什么好处吗?
小学升初中,儿子数学跳级的一点经验小学数学有比赛项目吗
有明天考AMC 8的吗?请教:怎样说服孩儿他爸多关心孩子的学习?
求科普: 奥数 vs. Math Olympiad vs. Math Count vs. Math circle vs. 超前学数学学校早晚要教的东西,早早学会了又如何?
9岁, 如何报名考amc10AMC 8 成绩 2016
算不算有数学天分请教小孩数学教育问题
[葩论] 论“有限小灶策略”小学数学书推荐?
相关话题的讨论汇总
话题: 数学话题: 竞赛话题: moems话题: aime话题: amc